You are on page 1of 150

LƯU GIANG NAM – TRẦN BÁ ĐẠT

VÕ THÀNH ĐẠT – LƯƠNG VĂN KHẢI – PHẠM THỊ HỒNG NHUNG


Đội Huấn luyện viên Trường đông Toán học miền Nam 2017

TRƯỜNG ĐÔNG TOÁN HỌC


MIỀN NAM 2017
NHỮNG BÀI TOÁN HAY VÀ KHÓ
LƯU GIANG NAM - TRẦN BÁ ĐẠT
VÕ THÀNH ĐẠT - LƯƠNG VĂN KHẢI - PHẠM THỊ HỒNG NHUNG
Đội HLV Trường đông Toán học miền Nam 2017

TRƯỜNG ĐÔNG TOÁN HỌC


MIỀN NAM 2017
Những bài toán hay và khó

Tháng 12/ 2017


Mục lục

1 ĐỀ BÀI 9
1.1 Đại số . . . . . . . . . . . . . . . . . . . . . . . . . . . . . . . . . 9
1.2 Hình học . . . . . . . . . . . . . . . . . . . . . . . . . . . . . . . . 12
1.3 Phương trình hàm - Dãy số . . . . . . . . . . . . . . . . . . . . . . 14
1.4 Số học . . . . . . . . . . . . . . . . . . . . . . . . . . . . . . . . . 17
1.5 Tổ hợp . . . . . . . . . . . . . . . . . . . . . . . . . . . . . . . . . 19

2 HƯỚNG DẪN GIẢI 25


2.1 Đại số . . . . . . . . . . . . . . . . . . . . . . . . . . . . . . . . . 25
2.2 Hình học . . . . . . . . . . . . . . . . . . . . . . . . . . . . . . . . 51
2.3 Phương trình hàm - Dãy số . . . . . . . . . . . . . . . . . . . . . . 76
2.4 Số học . . . . . . . . . . . . . . . . . . . . . . . . . . . . . . . . . 109
2.5 Tổ hợp . . . . . . . . . . . . . . . . . . . . . . . . . . . . . . . . . 121
LỜI NÓI ĐẦU

Nét đăc trưng của các chương trình Gặp gỡ Toán học và Trường đông Toán
học miền Nam chính là đội Huấn luyện viên. Là các sinh viên trưởng thành từ
phong trào Toán Olympic, chúng tôi quay lại giúp đỡ các bạn học sinh chuyên
Toán với tất cả nhiệt huyết, đam mê còn nguyên vẹn từ ngày xưa, ngày mà
chúng tôi ăn ngủ cùng những bài toán, nói những câu chuyện chỉ có những bài
toán, giải những bài toán mỗi ngày và đọc những bài viết về phương pháp giải
toán mỗi đêm. Hơn ai hết, chúng tôi hiểu rõ những gì mà các học sinh chuyên
Toán trải qua, và chúng tôi muốn chia sẻ những khó khăn ấy cùng các bạn.
Tập san này ra đời không ngoài mục đích đó. Chúng tôi muốn đem những
gì có ở Trường đông Toán học miền Nam tới thật nhiều các bạn học sinh, để các
bạn, đặc biệt là những bạn chưa có điều kiện tham gia chương trình, đúng như
tinh thần Bring Math to Everyone của thầy Trần Nam Dũng, đúng với những
giá trị cốt lõi mà Gặp gỡ Toán học và Trường đông Toán học miền Nam. Trong
tập san này, các bài toán được chia theo lĩnh vực, đề bài được đưa ra trước để
các bạn học sinh suy nghĩ, sau đó là phần hướng dẫn giải. Có những bài sẽ có
lời giải đầy đủ, có những bài chỉ có phần hướng dẫn sơ lược, và có những bài
sẽ có thêm phần nhận xét và mở rộng. Xuất hiện ở đây không chỉ có những bài
toán được dùng trong các bài giảng của các thầy mà còn là những bài toán được
thảo luận sau giờ học, trong giờ giải lao của các bạn học sinh.
Các biên tập viên của tập san này chính là các Huấn luyện viên của Trường
đông Toán học miền Nam năm nay:

• Bạn Lưu Giang Nam (sinh viên khoa Toán - Tin học trường ĐH Khoa học
Tự nhiên, ĐHQG Tp. HCM) biên tập phần Phương trình hàm - Dãy số.
• Bạn Trần Bá Đạt (sinh viên khoa Toán - Tin học trường ĐH Sư phạm Tp.
HCM) biên tập phần Tổ hợp.
• Bạn Võ Thành Đạt (sinh viên khoa Toán - Tin học trường ĐH Khoa học Tự
8 MỤC LỤC

nhiên, ĐHQG Tp. HCM) biên tập phần Bất đẳng thức - Đa thức.

• Bạn Lương Văn Khải (sinh viên khoa Toán - Tin học trường ĐH Khoa học
Tự nhiên, ĐHQG Tp. HCM) biên tập phần Hình học.

• Bạn Phạm Thị Hồng Nhung (cựu học sinh trường THPT chuyên Lê Quý
Đôn, tỉnh Bà Rịa - Vũng Tàu) biên tập phần Số học.

Chúng tôi xin gửi lời cảm ơn tới thầy Trần Nam Dũng (ĐH Khoa học Tự
nhiên, ĐHQG Tp. HCM), thầy Trần Quang Hùng (THPT chuyên Khoa học Tự
nhiên, ĐH Khoa học Tự nhiên, ĐHQG Hà Nội), thầy Nguyễn Song Minh (Titan
Education Hà Nội), thầy Võ Quốc Bá Cẩn (Archimedes Academy, Hà Nội), thầy
Lê Phúc Lữ (FPT Software Tp. HCM) đã cung cấp các bài giảng. Cảm ơn các bạn
Ngô Hoàng Anh, Phạm Hoàng Minh (học sinh chuyên Toán trường Phổ thông
Năng khiếu, ĐHQG Tp. HCM), bạn Nguyễn Minh Uyên (học sinh chuyên Toán
trường THPT chuyên Thoại Ngọc Hầu, An Giang), bạn Lư Thương Thương (học
sinh chuyên Toán trường THPT chuyên Lê Hồng Phong, Tp. HCM) đã nhiệt tình
hỗ trợ ban biên tập. Cảm ơn đơn vị tổ chức chương trình là Công ty cổ phần
Giáo dục Titan - Titan Education đã tạo mọi điều kiện để chúng tôi hoàn thành
được tập san này.
Chúng tôi rất mong nhận được sự đóng góp của bạn đọc để những ấn phẩm
sau được hoàn thiện hơn. Mọi đóng góp xin gửi qua hộp thư ở fanpage chường
trình https://www.facebook.com/gapgotoanhoc/. Những ý kiến của các bạn
sẽ là những kinh nghiệm lớn cho chúng tôi trong những lần biên tập sách tiếp
theo.
Cảm ơn tất cả các bạn!
Chương 1

ĐỀ BÀI

1.1 Đại số
Bài 1. (Đề tiêu thụ bài giảng trường đông miền Nam) Cho a, b, c là ba số thực sao
cho (a − b)(b − c)(c − a) 6= 0. Tìm giá trị nhỏ nhất của biểu thức
 
2 2 2
 1 1 1
P = a + b + c + ab + bc + ca + + .
(a − b)2 (b − c)2 (c − a)2

Bài 2. Cho a, b, c là các số dương thoả mãn abc = 1. Chứng minh rằng

a b c 3 2
√ +√ +√ ≥ .
a+1 b+1 c+1 2

Bài 3. (Đề đề nghị trường đông miền Nam)

1. Cho a, b, c là ba số thực. Đặt s = a2 + bc − ab, r = b2 + ca − bc, t =


c2 + ab − ca. Chứng minh rằng sr + ts + rt = a3 b + b3 c + c3 a. Từ đó suy ra
2
(a2 + b2 + c2 ) ≥ 3 (a3 b + b3 c + c3 a).

2. Cho x, y, z là các số thực dương thoả mãn điều kiện xy + yz + zx = 1.


Chứng minh rằng

√ x2 y 2 z 2
3− 3+ + + ≥ (x + y + z)2 .
y z x
10 ĐỀ BÀI

Bài 4. (Đề đề nghị trường đông miền Nam) Cho a, b, c là ba số thực dương. Xét
bất đẳng thức
 2 n  2 n  2 n
a + bc b + ca c + ab
+ + ≥ an + b n + c n .
b+c c+a a+b

1. Chứng minh minh bất đẳng thức trên với n = 1.

2. Với n = 2 thì bất đẳng thức trên còn đúng không? Nếu có, hãy chứng
minh. Nếu không, hãy chỉ ra phản ví dụ.

Bài 5. (Đề chính thức trường đông Trung Trung Bộ) Cho a, b, c là các số thực
dương thoả mãn điều kiện a + b + c = 3. Chứng minh rằng
 
9 a c b
≥2 + + + 3.
abc c b a

Bài 6. Tìm số thực k nhỏ nhất sao cho bất đẳng thức

xyz + 2 + k (x − 1)2 + (y − 1)2 + (z − 1)2 ≥ x + y + z


 

đúng với mọi x, y, z không âm.


Bài 7. (Đề đề nghị trường đông miền Nam) Cho a, b, c là ba cạnh của một tam
giác, chứng minh bất đẳng thức sau:

a2 b(a − b) + b2 c(b − c) + c2 a(c − a) ≥ 0.

Bài 8. (Đề đề nghị trường đông miền Nam) Cho a, b, c là các số thực không âm
thoả mãn điều kiện a + b + c = 3. Chứng minh rằng

a b c 1
+ 3 + 3 ≥ .
b3 + 16 c + 16 a + 16 6

Bài 9. Chứng minh rằng với mọi số thực a, b, c, ta đều có

(b + c − a)2 (c + a − b)2 (a + b − c)2 ≥ (b2 + c2 − a2 )(c2 + a2 − b2 )(a2 + b2 − c2 ).

Bài 10. Cho a, b, c, d là các số thực dương. Chứng minh rằng


r r
ab + ac + ad + bc + bd + cd 3 abc + bcd + cda + dab
≥ .
6 4
1.1 Đại số 11

Bài 11. Tìm tất cả các đa thức P (x) ∈ R[x] thoả mãn

P (x)P (2x2 ) = P (x3 + x).

Bài 12. Cho đa thức P (x) ∈ R[x] và P (x) ≥ 0 với mọi x ∈ R. Chứng minh rằng
đa thức P (x) có thể biểu diễn dưới dạng

P (x) = (A(x))2 + (B(x))2

trong đó A(x), B(x) cũng là các đa thức có hệ số thực.


Bài 13. Cho đa thức

f (x) = xn + an−2 xn−2 + an−3 xn−3 + . . . + a1 x + a0 ∈ R[x].

n!
Chứng minh rằng tồn tại i ∈ {1, 2, . . . , n} sao cho |f (i)| ≥ .
Cni
Bài 14. Cho số nguyên n ≥ 3 và P (x) ∈ R[x] thoả mãn

P (x) = xn + an−3 xn−3 + an−4 xn−4 + . . . + a1 x + a0 .

Biết ít nhất một số trong các số a0 , a1 , . . . , an−3 khác 0. Chứng minh rằng P (x)
không thể có toàn nghiệm thực.
Bài 15. Cho n là số nguyên dương chẵn. Một đa thức monic bậc n có n nghiệm
thực (không nhất thiết phân biệt). Giả sử y p
là số thựcpdương thoả với mọi số
thực t < y, ta có P (t) > 0. Chứng minh rằng P (0) − P (y) ≥ y.
n n

Bài 16. Tìm các đa thức f (x), g(x) hệ số nguyên thoả mãn

f (g(x)) = x2017 + 2018x + 1.


n n−1
Bài 17. Cho đa thức hệ số nguyên P (x) = an x + an−1 x + . . . + a1 x + a0 (an 6=
ai
0). Giả sử tồn tại m sao cho m ≥ max + 2 và P (m) là số nguyên tố.
i∈0,n an
Chứng minh rằng P (x) bất khả quy trên Z.
Bài 18. Cho số nguyên dương n. Tìm tất cả đa thức P thoả mãn đồng thời hai
điều kiện sau:

i. P (x) = an xn + an−1 xn−1 + . . . + a1 x + a0 trong đó {a0 , a1 , . . . , an } =


{0, 1, 2, . . . , n}.

ii. Đa thức P có n nghiệm thực phân biệt.


12 ĐỀ BÀI

Bài 19. Tìm tất cả các đa thức hệ số nguyên f sao cho n|m ⇒ f (n)|f (m) với
mọi các số nguyên dương m, n.
Bài 20. (Đề chính thức ngày 1 trường đông miền Nam) Cho đa thức monic P (x)
bậc n > 1 (tức P có hệ số của bậc lớn nhất bằng 1) có n nghiệm thực x1 , x2 , ...xn
phân biệt khác 0. Chứng minh rằng:

1 1 1 (−1)n+1
+ + ... + = .
x1 P 0 (x1 ) x2 P 0 (x2 ) xn P 0 (xn ) x1 x2 ...xn

1.2 Hình học


Bài 1. (Trần Quang Hùng) Cho 4ABC. điểm P di chuyển trên cạnh P C. Q, R
lần lượt là hai điểm đối xứng với P qua CA, AB. Lấy điểm M nằm trên đường
tròn ngoại tiếp 4AQR sao cho AM k BC. Chứng minh đường thẳng P M đi
qua một điểm cố định khi P di chuyển trên cạnh P C.
Bài 2. (Trần Quang Hùng) Cho 4ABC có đường cao AH. Giả sử đường tròn
đường kính BC tiếp xúc đường tròn nội tiếp 4ABC. Chứng minh rằng AH +
BC = AB + AC.
Bài 3. (Trần Quang Hùng) Cho 4ABC có hai đường cao BE và CF . Đường
tròn bàng tiếp góc A là (Ia ). Hai tiếp tuyến chung trong của (AEF ) và (Ia ) cắt
BC tại P và Q. Chứng minh rằng BP = CQ.
Bài 4. (Trần Quang Hùng) Cho đường tròn (O1 ) và (O2 ) ngoài nhau có AB
là một tiếp tuyến cung ngoài và CD là một tiếp tuyến chung trong. Gọi P =
AB ∩ CD, Q = AD ∩ BC. Chứng minh rằng P Q ⊥ O1 O2 .
Bài 5. (Trần Quang Hùng) Cho tứ giác ABCD nội tiếp đường tròn tâm O và
một điểm P thuộc cung CD không chứa A và B. Gọi E = P A ∩ BD, G =
BP ∩ AC, H = AP ∩ CD, F = BP ∩ CD, Q = EF ∩ HG. Chứng minh rằng P Q
luôn đi qua một điểm cố định khi P di chuyển.
Bài 6. (Trần Quang Hùng) CCho tam giác ABC nội tiếp đường tròn tâm O cố
định có hai điểm BC cố định, A di chuyển trên (O). Gọi AD, BE, CF là ba
đường cao của tam giác cắt nhau tại H. DE, DF lần lượt cắt HB, HC tại Q, R.
Gọi M là trung điểm QR. Chứng minh rằng HM đi qua điểm cố định.
Bài 7. (Trần Quang Hùng) Cho 4ABC nội tiếp đường tròn (O) cố định có B, C
cố định và A di động trên (O). Gọi E, F lần lượt là điểm đồi xứng của B, C qua
CA, AB. Gọi M = CE ∩ AB, N = BF ∩ AC. Chứng minh rằng đường thẳng
qua A vuông góc với M N đi qua một điểm cố định.
1.2 Hình học 13

Bài 8. (Nguyễn Minh Hà - Tạp chí Mathley) Cho tam giác ABC nhọn, BE, CF
là các đường cao. M là trung điểm của BC. N là giao điểm của AM và EF .
Gọi X là hình chiếu của N lên BC. Y, Z theo thứ tự là hình chiếu của X trên
AB, AC .Chứng minh rằng N là trực tâm tam giác AY Z.
Bài 9. (Moscow Olympiad) Cho 4ABC có đường trung tuyến AM . Đường cao
BE cắt đường trung tuyến AM tại P . Lấy Q sao cho QE ⊥ AM, CQ ⊥ AB.
Chứng minh rằng AQ ⊥ CP .
Bài 10. (Đề chính thức ngày 1 trường đông miền Nam) Cho tam giác ABC nội
tiếp đường tròn (O). Tiếp tuyến tại B, C của đường tròn (O) cắt nhau tại T . Gọi
M, N lần lượt là các điểm thuộc tia BT, CT sao cho BM = BC = CN . Đường
thẳng M N cắt CA, AB theo thứ tự tại E, F ; BE giao CT tại P, CF giao BT tại
Q. Chứng minh rằng AP = AQ.
Bài 11. (Đề chính thức ngày 2 trường đông miền Nam) Cho hai đường tròn (O1 )
và (O2 ) tiếp xúc ngoài tại M . Một đường thẳng cắt (O1 ) tại A, B và tiếp xúc với
(O2 ) tại E (B nằm giữa A và E). Đường thẳng EM cắt (O1 ) tại điểm J khác
M . C là một điểm thuộc cung M J không chứa A, B của (O1 ) (C khác M và J).
Kẻ tiếp tuyến CF với đường tròn (O2 ) (F là tiếp điểm) sao cho các đoạn thẳng
CF và M J không cắt nhau. Gọi I là giao điểm của các đường thẳng CJ và EF ,
K là giao điểm khác A của đường thẳng AI và đường tròn (O1 ). Chứng minh
rằng:

1. Tứ giác M CF I là tứ giác nội tiếp và JA2 = JI 2 = JM.JE.


2. CI là phân giác ngoài tại C của tam giác ABC.
3. K là tâm đường tròn ngoại tiếp của tam giác BCI.

Bài 12. (Đề đề nghị trường đông Trung Trung Bộ) Cho tam giác ABC. Đường
tròn (K) bất kỳ tiếp xúc đoạn thẳng AC, AB lần lượt tại E, F . (K) cắt đoạn
thẳng BC tại M, N sao cho N nằm giữa B và M . F M giao EN tại I. Đường
tròn ngoại tiếp các tam giác IF N và IEM cắt nhau tại J khác I. Chứng minh
rằng IJ đi qua A và KJ vuông góc IJ.
Bài 13. (Đề đề nghị trường đông miền Nam) Cho (I) là đường tròn nội tiếp tam
giác nhọn ABC, tiếp xúc BC, CA, AB ở D, E, F . (O) là đường tròn ngoại tiếp
tam giác ABC. Lấy D0 đối xứng với D qua EF .

1. Chứng minh rằng AD0 , BC, OI đồng quy.


2. Gọi H, J lần lượt là trực tâm các tam giác ABC và AEF . Gọi R là giao
HJ và ID, T là giao D0 J và OI. Chứng minh rằng D0 , R, T, I đồng viên.
14 ĐỀ BÀI

Bài 14. (Trần Quang Hùng) Cho tam giác ABC, P là điểm bất kỳ. A1 là hình
chiếu của P lên BC.A2 là trung điểm AA1 .A2 P cắt BC tại A3 .A4 đối xứng A1
qua A3 . Chứng minh rằng P A4 luôn đi qua một điểm cố định.
Bài 15. (Tây Ninh 2017) Cho năm điểm A, B, C, D và E cùng nằm trên một
đường tròn. Gọi M , N , P và Q lần lượt là hình chiếu vuông góc của E xuống
các đường thẳng AB, BC, CD và DA. Gọi hình chiếu vuông góc của E xuống
các đường thẳng M N , N P , P Q, QM lần lượt là R, S, T và U cùng nằm trên
một đường thẳng.

1.3 Phương trình hàm - Dãy số


Bài 1. (Đề chính thức ngày 2 trường đông miền Nam) Tìm các hàm f : R → R
thoả mãn:

f (x2 ) + f (xy) = f (x)f (y) + yf (x) + xf (x + y), ∀ xy ∈ R.

Bài 2. (Đề tiêu thụ bài giảng trường đông miền Nam) Cho hàm số :f : N → N
thoả
f (f (n)) + f (n) = 6n + 4.

1. Tính f (2017)

2. Tìm tất cả các hàm f thoả mãn.

Bài 3. (Đề tiêu thụ bài giảng trường đông miền Nam) Cho dãy số
(
x1 = x2 = 97
(xn ) : q  .
xn+2 = xn+1 xn + x2n+1 − 1 (x2n − 1) ∀n ≥ 1

1. Chứng minh 2 + 2xn là số chính phương.



2. Chứng minh 2 + 2 + 2xn là số chính phương,

Bài 4. (Đề tiêu thụ bài giảng trường đông miền Nam) Xét dãy số nguyên {an }∞
n=1
thoả mãn:
−1 a2n+1 1
a1 = 2; a2 = 7; < an+2 − ≤ .
2 an 2
Chứng minh rằng với mọi n > 1,ta có an là số lẻ.
1.3 Phương trình hàm - Dãy số 15

Bài 5. Tìm f : R → R thoả mãn:

f (x3 + f (y)) = y + f 3 (x) ∀ x, y ∈ R.

Bài 6. Tìm tất cả các hàm f : R → R thoả mãn:

f (xf (x + y)) = f (yf (x)) + x2 , ∀x, y ∈ R.

Bài 7. (Đề tiêu thụ bài giảng trường đông miền Nam) Tìm tất cả hàm số f : R →
R thoả:
f (f (y) + x2 + 1) + 2x = y + (f (x + 1))2 , ∀x, y ∈ R.
Bài 8. Tìm tất cả các hàm f (x) : [1; +∞) → [1; +∞) thoả

x ≤ f (x) ≤ 2x + 2
xf (x + 1) = (f (x))2 − 1

Bài 9. Tìm tất cả các hàm f : R → R thoả mãn:

f (yf (x) − x) = f (x)f (y) + 2x, ∀x, y ∈ R.

Bài 10. Tìm f: R+ → R+ thoả mãn:

f (x + y) + f (x).f (y) = f (xy) + f (x) + f (y).

Bài 11. (Đề tiêu thụ bài giảng trường đông miền Nam) Tìm các hàm f : Q2 → Q
thoả mãn
f (x, y) + f (y, z) + f (z, x) = f (0, x + y + z).
với mọi x, y, z ∈ Q.
Bài 12. Tìm f : R ⇒ R thoả:

f (x)f (yf (x) − 1) = x2 f (y) − f (x), ∀x, y ∈ R

Bài 13. Cho f : Q2 → R+ với Q2 = {(x, y)|x, y ∈ Q}. Giả sử f thoả mãn các
điều kiện sau:

1. f (a, b, c) = f (a, c)f (b, c), ∀a, b, c ∈ Q.

2. f (c, ab) = f (c, a)f (c, b), ∀a, b, c ∈ Q.

3. f (a, 1 − a) = 1, ∀a ∈ Q.
16 ĐỀ BÀI

Chứng minh rằng f (a, a) = f (a, −a) = 1 và f (a, b)f (b, a) = 1.


Bài 14. Cho hàm số f : N? × N? → N? (với N? = N ∪ {0}) thoả mãn các điều
kiện sau:

1. f (0, x) = x + 1

2. f (x + 1, 0) = f (x, 1)

3. f (x + 1, y + 1) = f (x, f (x + 1, y))

Tính f (1, 2017), f (2, 2017), f (3, 2017), f (4, 2017).


Bài 15. Cho dãy số (xn ) được xác định bởi:

x1 = 4
xn+1 = x2n − 2, ∀ ∈ N∗

Tính
xn+1
lim
n→+∞ x1 x2 x3 ...xn

Bài 16. Cho dãy số (un ) được xác định bởi công thức truy hồi :

 u1 = α ∈ R
3
u + 9un − 6
un+1 = n2 , ∀α ∈ N∗
3un − 6un + 7

Tìm α để dãy số (un ) có giới hạn hữu hạn khi và tìm giới hạn của dãy số trong
các trường hợp đó.
Bài 17. Cho hai dãy số (an ), (bn ) được xác định bởi hệ thức truy hồi:

a1 = 3, b1 = 2, an+1 = a2n + 2b2n , bn+1 = 2an bn , ∀n ∈ N∗

Tìm
2n
p √
2n
lim bn ; lim a1 a2 ...an
n→+∞ n→+∞

Bài 18. Cho dãy số (xn ) được xác định bởi hệ thức truy hồi:

 x1 = a ≥ 1
x2n − 2xn 2
xn+1 = , ∀n ∈ N∗
[xn ]2

Chứng minh dãy số (xn ) đã cho có giới hạn hữu hạn khi n → +∞.
1.4 Số học 17

Bài 19. Tìm tất cả các hàm số f : R → R thoả mãn:

f (yf (x + y) + f (x)) = 4x + 2yf (x + y), ∀x, y ∈ R.

Bài 20. Đặt I = [0, 1] và G = {(x, y)|x, y ∈ I}. Tìm tất cả các hàm số f : G → I
thoả mãn với mọi x, y, z ∈ I ta có:

1. f (f (x, y), z) = f (x, f (y, z)),

2. f (x, 1) = x, f (1, y) = y,

3. f (zx, zy) = z k f (x, y).

với k là một số thực dương không liên quan đến x, y, z.

1.4 Số học
Bài 1. (Đề chính thức ngày 2 trường đông miền Nam) Biết rằng với dãy nguyên
dương 1 < k1 < k2 < ... < kn và dãy nguyên tương ứng s1 , s2 , ..., sn , với mọi số
nguyên dương N đều tồn tại i ∈ {1, 2, ..., n} sao cho N ≡ si (mod ki ).

1. Tìm dãy {kn } và {sn } thoả mãn khi k1 = 2 và khi k1 = 3.


n
X 1
2. Chứng minh rằng > 1.
i=1
ki

3. Tìm n nhỏ nhất để có các dãy TMĐK đề bài.

Bài 2. (Đề tiêu thụ bài giảng trường đông miền Nam) Tìm n ∈ N và n > 1 sao
.
cho 3n − 1 .. n3 .
Bài 3. (Đề tiêu thụ bài giảng trường đông miền Nam) Chứng minh rằng tồn tại
vô số số nguyên tố dạng 2n k + 1 với k nguyên dương và n ≥ 2017.
Bài 4. (Đề tiêu thụ bài giảng trường đông miền Nam) Tìm số nguyên dương n
.
nhỏ nhất sao cho 1n + 2n + . . . + 2016n 6 .. 2017.
Bài 5. (Đề tiêu thụ bài giảng trường đông miền Nam) Cho 9 số nguyên dương
phân biệt d1 , d2 , . . . , d9 và đa thức P (x) = (x + d1 )(x + d2 ) . . . (x + d9 ). Chứng
18 ĐỀ BÀI

minh rằng có số N nguyên dương sao cho ∀x ≥ N thì P (x) có ước nguyên tố
lớn hơn 20.
Bài 6. (Đề tiêu thụ bài giảng trường đông miền Nam) Cho P (x) ∈ Z[x]. Biết
a1 , a2 , . . . , an là các số nguyên thoả mãn: ∀x ∈ Z, ∃i ∈ {1; 2; . . . ; n} sao cho
. .
P (x) .. ai . Chứng minh rằng ∃j : P (x) .. aj ∀x ∈ Z.
Bài 7. (Đề tiêu thụ bài giảng trường đông miền Nam) Tìm các số nguyên dương
n sao cho với mọi số nguyên dương k, tồn tại số tự nhiên a sao cho a3 + a − k
chia hết cho n.
Bài 8. (Đề tiêu thụ bài giảng trường đông miền Nam) Tìm đa thức P (x) hệ số
nguyên biết với mọi số nguyên tố p, a, b nguyên dương thì ab ≡ 1 (mod p) ⇒
P (a).P (b) ≡ 1 (mod p).
Bài 9. (Đề tiêu thụ bài giảng trường đông miền Nam) An và Bảo cùng nhau chơi
một trò chơi: họ lần lượt viết các số tuỳ thích lên bảng thành một dòng, mỗi
người 3 số, An viết trước. Sau đó Bảo "nhường" An điền dấu + hoặc − tuỳ ý vào
giữa các số đã viết. An thắng nếu kết quả trên bảng không chia hết cho bất cứ
số tự nhiên nào từ 11 đến 18. Bảo thắng nếu xảy ra trường hợp ngược lại. An
nói rằng mình kiểm soát nhiều hơn, nên chắc chắn chiến thắng. Bạn có đồng ý
không? Tại sao?
Bài 10. (Đề đề nghị trường đông Trung Trung Bộ) Giả sử N∗ phân hoạch thành 3
dãy tăng {an }, {bn }, {cn } thoả mãn:

i. can = bn + 1

ii. an+1 > bn

iii. cn+1 cn − (n + 1)cn+1 − ncn chẵn.

Chứng minh rằng an = n2 .


Bài 11. (Nguyễn Song Minh) Tìm các đa thức thoả điều kiện: a2 − b2 ∈ Q thì
P (a) − P (b) ∈ Q.
Bài 12. (Nguyễn Song Minh) Tìm tất cả đa thức P (x) hệ số nguyên thoả mãn

2n | P (3n ) ∀n ∈ N∗ .

Bài 13. (Nguyễn Song Minh) Tìm các đa thức P (x) hệ số nguyên, a, b nguyên
dương và a > b sao cho:

P (n) | an − bn ∀n ∈ N∗ .
1.5 Tổ hợp 19

Bài 14. (Nguyễn Song Minh) Tìm tất cả các đa thức P (x) hệ số nguyên sao cho

(P (n); P (2017n )) = 1 ∀n ∈ Z+ .

Bài 15. Cho số nguyên dương d. Gọi f (d) là số nguyên dương nhỏ nhất có đúng
.
d ước nguyên dương. Chứng minh rằng f (2k+1 ) .. f (2k ) ∀k ∈ N.

1.5 Tổ hợp

Bài 1. (Đề đề nghị trường đông miền Nam) 23 người bạn muốn cùng nhau chơi
bóng đá. Họ sẽ phải chọn ra một người làm trọng tài và 22 người còn lại chia
làm hai đội đá với nhau. Họ muốn chia sao cho tổng cân nặng của mỗi đội là
bằng nhau. Giả sử cân nặng của từng người trong số 23 người là các số nguyên
dương và với bất kì cách chọn trọng tài nào thì họ cũng có thể chia thành hai
đội mà tổng cân nặng của mỗi đội bằng nhau. Chứng minh rằng 23 người này
có cân nặng bằng nhau.
Bài 2. (Đề đề nghị trường đông miền Nam) Bạn An chơi trò chơi xếp hình với
luật chơi như sau: Cho một hình vuông 4 × 4 chia thành 16 ô, có 15 mảnh ghép
và một ô trống. Trong mỗi bước chơi, An sẽ được phép trượt các mảnh ghép vào
ô trống để thu được hình mới. Bạn An sẽ thắng nếu sau hữu hạn bước trượt, An
thu được hình như sau:

Hỏi An có thể chiến thắng nếu hình ban đầu là hình sau hay không?
20 ĐỀ BÀI

Bài 3. (Đề đề nghị trường đông miền Nam) Cho một dãy vô hạn a1 , a2 , .., an , ...
với a1 = 1. Biết rằng với mỗi n > 1 thì:

• Nếu ước lẻ lớn nhất của n đồng dư với 1 module 4 thì an = an−1 + 1.

• Nếu ước lẻ lớn nhất của n đồng dư với 3 module 4 thì an = an−1 − 1.

1. Chứng minh rằng trong dãy số đó mỗi số nguyên dương xuất hiện vô số
lần.

2. Đặt bn = mini∈N (ai = n). Tìm một công thức tính bn theo n.

Bài 4. Cho lục giác đều ABCDEF cạnh 1 được điền các số như hình vẽ.

0
F B
2 1

0
1 C
E

1
D
1.5 Tổ hợp 21

Có một con ếch ở vị trí A nhảy xung quanh các đỉnh của đa giác với độ dài các
bước nhảy nguyên. Gọi m là số cách nhảy của ếch sao cho tổng các số nó nhảy
qua là 2017. Chứng minh rằng m không là số chính phương.
Bài 5. Cho hai dung dịch A và B thoả mãn:

i. Số đo khối lượng của 1 lít A bằng số đo thể tích của 1kg B.

ii. p lít dung dịch A nặng bằng q lít dung dịch B với p, q là số nguyên tố phân
biệt.

Người ta chia các dung dịch A và B vào các bình giống nhau chứa 1 lít dung
dịch và vỏ bình nặng 1kg. Chứng minh rằng có duy nhất 1 cách để ghép các
bình cùng loại A hoặc B với nhau sao cho tổng khối lượng thuộc (2017, 2018).
Bài 6. Có 2020 người đến một buổi tiệc được chia vào các phòng khác nhau sao
cho:

i. Không người nào trong một phòng quen biết tất cả các người trong phòng
đó

ii. Trong nhóm 3 người bất kì thuộc cùng một phòng, luôn tồn tại ít nhất 2
người không quen biết nhau.

iii. Với bất kì một nhóm 2 người nào trong một phòng mà không quen biết
lẫn nhau, tồn tại đúng một người trong cùng phòng đó quen biết cả hai
người này.

1. Chứng minh rằng trong mỗi phòng, mỗi người có số người quen bằng
nhau.

2. Tìm số phòng lớn nhất.

Bài 7. Cho tập hợp A = {1, 2, . . . , 2n} với n nguyên dương. Một hoán vị các
phần tử của A được gọi là đẹp nếu như có ít nhất hai phần tử hơn kém nhau n
đơn vị đứng cạnh nhau. Chứng minh rằng số hoán vị đẹp nhiều hơn số hoán vị
không đẹp
Bài 8. Cho n là một số nguyên dương và S là tập hợp các điểm (x, y) trên mặt
phẳng với x, y không âm và x + y < n. Các điểm trong S được tô màu đỏ hoặc
xanh theo qui luật, nếu (x, y) là màu đỏ thì (x0 , y 0 ) được tô màu đỏ với x ≤ x0 và
y ≤ y 0 . Đặt A là số cách chọn n điểm xanh mà hoành độ x của nó khác nhau và
22 ĐỀ BÀI

đặt B là số cách chọn n điểm xanh mà tung độ y của chúng khác nhau. Chứng
minh rằng A = B
Bài 9. Cho một dãy n tấm bìa đặt sấp ở trên bàn được đánh số từ 1 tới n. Mỗi
lần cho phép thay đổi trạng thái của k tấm bìa liên tiếp từ sấp thành ngửa và
ngược lại

1. Chứng minh rằng có thể chuyển hết tấm bìa từ sấp sang ngửa khi và chỉ
.
khi m .. k

2. Nếu n không chia hết cho k, tìm số bìa tối đa có thể chuyển sang ngửa.

Bài 10. Trong một bảng vuông n×n, ta đặt những chiếc đèn lên các ô của bảng,
mỗi ô một đèn. Ở mỗi lần thay đổi, ta được phép chọn một đèn làm gốc và thay
đổi trạng thái của đèn đó và tất cả các đèn khác cùng hàng cùng cột với nó từ
tắt sang bật và ngược lại. Với trạng thái ban đầu là bất kì, ta có thể đưa tất cả
đèn về trạng thái bật được hay không với

1. n = 6.

2. n = 2017.

Bài 11. Cho một bảng 5 × 5 được tô trắng đen xen kẽ, các ô ở góc được tô đen.
Trên mỗi ô đen có các đồng xu đen và trên các ô trắng có các đồng xu trắng.
Các đồng xu có thể di chuyển đến các ô bên cạnh. A và B cùng chơi một trò chơi
như sau: Đầu tiên, A khởi động trò chơi bằng cách lấy một đồng xu đen ra khỏi
bảng rồi di chuyển một đồng xu trắng vào ô trống. Sau đó, B di chuyển một
đồng xu đen vào ô trống. Các lượt sau đó, A sẽ di chuyển một đồng xu trắng
vào ô trống và B di chuyển đồng xu đen vào ô trống. Trò chơi kết thúc khi một
trong hai người không thể di chuyển được theo luật trên, và người còn lại là
người chiến thắng. Hỏi có chiến thuật thắng hay không, nếu có hãy chỉ ra ai là
người thắng?
Bài 12. Cho lưới tam giác đều như hình vẽ, trong đó mỗi cạnh có chứa n điểm
(không tính hai đầu mút của cạnh), các đoạn thẳng song song với cạnh tam
giác lớn được nối với nhau. Đếm số tam giác đều có đỉnh là các điểm trong lưới
đã cho
1.5 Tổ hợp 23

B C

Bài 13. Cho 2018 bóng đèn đang ở trạng thái sáng được xếp liên tiếp nhau trên
một đường thẳng. Hai người cùng chơi trò chơi như sau: Ở mỗi lượt chơi của
mình, mỗi người sẽ chọn một bóng đèn sáng, sau đó đổi trạng thái của bóng
đèn đó cùng với 4 bóng đèn phía sau nó.

1. Chứng minh rằng trò chơi sẽ dừng lại sau hữu hạn bước.

2. Ai có thể luôn là người chiến thắng? Hãy đưa ra chiến thuật thắng ấy.

Bài 14. Cho 33 điểm khác nhau nằm bên trong một hình vuông√có cạnh là 4. Vẽ
33 đường tròn nhận các điểm này làm tâm, có cùng bán kính 2. Chứng minh
rằng tồn tại một đường tròn trong số chúng chứa ít nhất 3 điểm trong số 33
điểm nói trên.
Bài 15. Có 2010 que diêm trên bàn. A và B cùng chơi trò chơi theo lượt như
sau: Đến lượt của mình, họ sẽ lấy đi 1, 3, 4, 5 hoặc 7 que diêm. Người lấy que
diêm cuối cùng sẽ chiến thắng. Nếu A chơi trước, hỏi người nào sẽ có chiến
thuật thắng?
24 ĐỀ BÀI
Chương 2

HƯỚNG DẪN GIẢI

2.1 Đại số
Bài 1

Cho a, b, c là ba số thực sao cho (a − b)(b − c)(c − a) 6= 0. Tìm giá trị nhỏ
nhất của biểu thức
 
2 2 2
 1 1 1
P = a + b + c + ab + bc + ca + + .
(a − b)2 (b − c)2 (c − a)2

Lời giải. Ta có
2 1
a2 + b2 + c2 + ab + bc + ca = (a + b + c)2 + (a − b)2 + (b − c)2 + (c − a)2

3 6
1
(a − b)2 + (b − c)2 + (c − a)2


6
 
1  2 2 2
 1 1 1
⇒P ≥ (a − b) + (b − c) + (c − a) + +
6 (a − b)2 (b − c)2 (c − a)2
Giả sử a > b > c. Đặt x = a − b, y = b − c
 
1 2 2 2
 1 1 1
P ≥ x + y + (x + y) + +
6 x2 y 2 (x + y)2
 
1 2 2
 1 1 1
= x + xy + y + +
3 x2 y 2 (x + y)2
26 HƯỚNG DẪN GIẢI

x
Đặt t =
y  
1 2  1 1
P ≥ t +t+1 + +1
3 t2 (t + 1)2
 2(t2 + t) + 1
 
1 2
= t +t+1 +1
3 (t2 + t)2
Đặt z = t2 + t
z z
1

(z + 1)2

(z + 1)3 27. .
P ≥ (z + 1) = ≥ 2 2 =9
3 z 2 3z 2 3z 2 4

b=0
Dấu "=" xảy ra ⇔ .
a = −c
9
Vậy giá trị nhỏ nhất của P là .
4
Nhận xét.

1. Ta có thể tổng quát bài toán như sau: Cho x, y, z là các số thực đôi một phân
biệt. Với những giá trị nào của k thì biểu thức:
 
 2 2 2
 1 1 1
P = x + y + z + k(xy + yz + zx) + +
(x − y)2 (y − z)2 (z − x)2

đạt giá trị nhỏ nhất và hãy tìm giá trị nhỏ nhất đó.
Chứng minh tương tự như lời giải.
2. Một số bài toán tương tự:
(a) (VMO 2008) Cho x, y, z là các số thực không âm đôi một phân biệt.
Chứng minh rằng:
 
1 1 1
(xy + yz + zx) + + ≥ 4.
(x − y)2 (y − z)2 (z − x)2

(b) (Đào Hải Long) Cho x, y, z là các số thực đôi một phân biệt. Chứng
minh rằng:
 
2 2 2 1 1 1 9
(x + y + z ) 2
+ 2
+ 2
≥ .
(x − y) (y − z) (z − x) 2
2.1 Đại số 27

Bài 2

Cho a, b, c là các số dương thoả mãn abc = 1. Chứng minh rằng



a b c 3 2
√ +√ +√ ≥ .
a+1 b+1 c+1 2
√ √
x 3 2 2
Lời giải. Xét f (x) = √ − ln x − với x > 0.
x+1 8 2

x + 2 3 2
Ta có: f 0 (x) = 3 − . Giải phương trình f 0 (x) = 0 trên (0, +∞) ta
2(x + 1) 2 8x
được nghiệm duy nhất x = 1.

x 0 1 +∞

f 0 (x) − 0 +

+∞ +∞

f (x)

Từ bảng biến thiên ta được f (x) ≥ 0 với x > 0. Suy ra


f (a) + f (b) + f (c) ≥ 0
√ √
a b c 3 2 3 2
⇔√ +√ +√ − (ln a + ln b + ln c) − ≥ 0.
a+1 b+1 c+1 8x 2
Do abc = 1 nên ln a + ln b + ln c = 0. Từ đó ta được điều phải chứng minh.
Nhận xét.

1. Công thức ln xy = ln x + ln y giúp ta chuyển điều kiện tích thành tổng để áp


dụng phương pháp tiếp tuyến.
2. Một số bài toán tương tự:
(a) Cho a, b, c > 0 thoả mãn abc = 1. Chứng minh rằng
√ √ √ √
a2 + 1 + b2 + 1 + c2 + 1 ≤ 2(a + b + c).
28 HƯỚNG DẪN GIẢI

(b) (Moldova TST 2011) Cho các số dương x1 , x2 , . . . , xn thoả mãn x1 x2 . . . xn =


1. Chứng minh rằng
1 1 1 n
+ + ... + ≥ .
x1 (x1 + 1) x2 (x2 + 1) xn (xn + 1) 2

Bài 3

1. Cho a, b, c là ba số thực. Đặt s = a2 + bc − ab, r = b2 + ca − bc, t =


c2 + ab − ca. Chứng minh rằng sr + ts + rt = a3 b + b3 c + c3 a. Từ đó
2
suy ra (a2 + b2 + c2 ) ≥ 3 (a3 b + b3 c + c3 a).

2. Cho x, y, z là các số thực dương thoả mãn điều kiện xy + yz + zx = 1.


Chứng minh rằng
√ x2 y 2 z 2
3− 3+ + + ≥ (x + y + z)2 .
y z x

Lời giải.

1. Bằng tính toán trực tiếp ta sẽ kiểm tra được sr + ts + rt = a3 b + b3 c + c3 a,


sau đó áp dụng bất đẳng thức: (r + t + s)2 ≥ 3(sr + ts + rt) ∀r, s, t ∈ R
2. Áp dụng bất đẳng thức ở câu 1 kết hợp với bất đẳng thức Cauchy Schwarz
ta được
 2 2
y2 z2
 3
y3 z3 2x3 2y 3 2z 3
  
x x
+ + ≥3 √ +√ +√ ≥3 + +
y z x yz zx xy y+z z+x x+y
3(x2 + y 2 + z 2 )2
≥ = 3(x2 + y 2 + z 2 )2
xy + yz + zx
x2 y 2 z 2 √ 2 √ 
≥ 3(x + y 2 + z 2 ) = 3 (x + y + z)2 − 2 .

⇒ + +
y z x
Ta quy về chứng minh
√  √
3 (x + y + z)2 − 2 ≥ (x + y + z)2 − 3 + 3

√ 
3 − 1 (x + y + z)2 − 3 ≥ 0


(luôn đúng theo bất đẳng thức Cauchy-Schwarz).


Từ đây ta được điều phải chứng minh.
2.1 Đại số 29

Nhận xét.

1. Bất đẳng thức ở câu a còn được gọi là bất đẳng thức Vasc, là một bổ đề mạnh
hay được sử dụng trong chứng minh bất đẳng thức.

2. Bất đẳng thức ở câu b chính là đề chọn đội tuyển Iran năm 2010.

3. Ngoài ra ta có thể tổng quát bất đẳng thức ở câu 1 như sau: Xét bất đẳng
thức sau với các biến thực a, b, c:
X X X X X
m a4 + n a2 b2 + p a3 b + g ab3 − (m + n + p + g) a2 bc ≥ 0.
cyc cyc cyc cyc cyc


m>0
Khi đó bất đẳng thức này đúng nếu .
3m(m + n) ≥ p2 + pg + g 2
Chứng minh: xem trong [1].

Bài 4

Cho a, b, c là ba số thực dương. Xét bất đẳng thức


 2 n  2 n  2 n
a + bc b + ca c + ab
+ + ≥ an + b n + c n (2.1)
b+c c+a a+b

1. Chứng minh minh bất đẳng thức (2.1) với n = 1.

2. Với n = 2 thì bất đẳng thức (2.1) còn đúng không? Nếu có, hãy chứng
minh. Nếu không, hãy chỉ ra phản ví dụ.

Lời giải.

1. Với n = 1, bất đẳng thức (2.1) trở thành

a2 + bc b2 + ca c2 + ab
+ + ≥a+b+c
b+c c+a a+b

(a + b)(a + c) (b + a)(b + c) (c + a)(c + b)


⇔ + + ≥ 2(a + b + c)
b+c c+a a+b
30 HƯỚNG DẪN GIẢI

Áp dụng bất đẳng thức AM-GM ta được

(a + b)(a + c) (b + a)(b + c)
+ ≥ 2(a + b)
b+c c+a
(b + a)(b + c) (c + a)(c + b)
+ ≥ 2(b + c)
c+a a+b
(c + a)(c + b) (a + b)(a + c)
+ ≥ 2(c + a)
a+b b+c
Cộng lại theo vế ta được đpcm.

2. Với n = 2, bất đẳng thức (2.1) trở thành


X (a + b)2 (a + c)2 X a(a + b)(a + c)
≥2
sym
(b + c)2 sym
b+c

X (a + b)2 (a + c)2 X
⇔ +2 (a + b)(a + c)
sym
(b + c)2 sym
X (a + b + c)(a + b)(a + c)
≥2 (2.2)
sym
b+c

Đặt x = b + c, y = c + a, z = a + b. Khi đó (2.2) tương đương với

y 2 z 2 z 2 x2 x2 y 2
 
yz zx xy
+ 2 + 2 + 2(xy + yz + zx) ≥ (x + y + z) + +
x2 y z x y z

y 2 z 2 z 2 x2 x2 y 2 xy(x + y) yz(y + z) zx(z + x)


⇔ 2
+ 2 + 2 + xy + yz + zx ≥ + +
x y z z x y
⇔ x4 y 4 + y 4 z 4 + z 4 x4 + x2 y 2 z 2 (xy + yz + zx)
≥ x3 y 3 (zx + yz) + y 3 z 3 (xy + zx) + z 3 x3 (yz + xy) (2.3)
Đặt xy = m, yz = n, zx = p. Khi đó (2.3) tương đương với

m4 + n4 + p4 + mnp(m + n + p) ≥ m3 (n + p) + n3 (m + p) + p3 (m + n).

Đây là bất đẳng thức Schur bậc 4 nên ta có điều phải chứng minh.
2.1 Đại số 31

Nhận xét.

1. Với n = 2 thì (2.1) còn đúng với a, b, c ∈ R thoả (a + b)(b + c)(c + a) 6= 0.


Thật vậy, dựa vô chứng minh ở trên, ta chỉ cần chứng minh bất đẳng thức
Schur bậc 4 đúng với mọi a, b, c ∈ R bằng cách đặt α = m + n − p, β =
α+β α+γ β+γ
m + p − n, γ = n + p − m thì m = , n= , p= . Khi đó
2 2 2
(2.3) tương đương

α2 (β − γ)2 + β 2 (γ − α)2 + γ 2 (α − β)2 ≥ 0 (?)

2. Ta có thể chứng minh được (2.1) còn đúng với n là số thực dương (xem trong
[1]).

Bài 5

Cho a, b, c là các số thực dương thoả mãn điều kiện a + b + c = 3. Chứng


minh rằng  
9 a c b
≥2 + + + 3. (2.4)
abc c b a

Lời giải. Không mất tính tổng quát giả sử a = max{a, b, c}. Bất đẳng thức (2.4)
tương đương

9 ≥ 2 a2 b + b2 c + c2 a + 3abc ⇔ 27 ≥ 6 a2 b + b2 c + c2 a + 9abc
 

⇔ (a + b + c)3 ≥ 6 a2 b + b2 c + c2 a + 9abc


⇔ a3 + b3 + c3 − 3abc ≥ 3 a2 b + b2 c + c2 a − ab2 − bc2 − ca2




⇔ (a + b + c) (a − b)2 + (b − c)2 + (a − c)2 ≥ 6(a − b)(b − c)(a − c) (4)


 

Ta có
1 1
(a − b)(b − c) ≤ (a − b + b − c)2 = (a − c)2
4 4
1
⇒ (a − c)(a − b)(b − c) ≤ (a − c)3
4
3
⇒ 6(a − c)(a − b)(b − c) ≤ (a − c)3
2

1 1
(a − b)2 + (b − c)2 ≥ (a − b + b − c)2 = (a − c)2
2 2
32 HƯỚNG DẪN GIẢI

 3
⇒ (a + b + c) (a − b)2 + (b − c)2 + (a − c)2 ≥ (a + b + c)(a − c)2

2
Ta quy về chứng minh
3 3
(a + b + c)(a − c)2 ≥ (a − c)3
2 2
⇔ (a − c)2 (b + 2c) ≥ 0
(luôn đúng với mọi a, b, c thoả điều kiện đề bài).
Vậy ta có điều phải chứng minh.

Bài 6

Tìm số thực k nhỏ nhất sao cho bất đẳng thức

xyz + 2 + k (x − 1)2 + (y − 1)2 + (z − 1)2 ≥ x + y + z


 
(2.5)

đúng với mọi x, y, z không âm.

Lời giải. Do (2.5) đúng với mọi bộ (x, y, z) không âm nên cũng đúng với (0, t +
1, t + 1), khi đó (2.5) trở thành
2t 2t 1
k≥ 2
, ∀t ≥ 1 ⇒ k ≥ min 2
⇒k≥ √ .
1 + 2t [1;+∞) 1 + 2t 2
1 1
Tiếp theo ta sẽ chứng minh (5) đúng với k = √ . Thật vậy, với k = √ thì (2.5)
2 2
trở thành
1 
xyz + 2 + √ (x − 1)2 + (y − 1)2 + (z − 1)2 ≥ x + y + z

2

Không mất tính tổng quát ta có thể giả sử (y −1)(z −1) ≥ 0, suy ra yz +1 ≥ y +z
hay xyz + x ≥ x(y + z). Từ đây ta quy về chứng minh

(x − 1)2 + (y − 1)2 + (z − 1)2 ≥ 2(y + z − 2)(1 − x)
 2
1 2 x+y−2
⇔ (x − y) + √ − (1 − z) ≥ 0.
2 2
Nên ta có điều phải chứng minh.
2.1 Đại số 33

Nhận xét.

1. Do [(x − 1)(y − 1)] · [(y − 1)(z − 1)] · [(z − 1)(x − 1)] = (x − 1)2 (y − 1)2 (z −
1)2 ≥ 0 nên trong 3 số (x − 1)(y − 1); (y − 1)(z − 1); (z − 1)(x − 1) phải có
ít nhất một số không âm, do vậy ta có thể giả sử (y − 1)(z − 1) ≥ 0.

2. Với k = 2 ta được bài toán của thầy Trần Nam Dũng trong cuộc thi Hello
IMO 2007: Cho x, y, z là các số thực dương. Chứng minh rằng ta luôn có bất
đẳng thức
xyz + 2(x2 + y 2 + z 2 ) + 8 ≥ 5(x + y + z).

Bài 7

Cho a, b, c là ba cạnh của một tam giác, chứng minh bất đẳng thức sau:

a2 b(a − b) + b2 c(b − c) + c2 a(c − a) ≥ 0.

Lời giải. Do a, b, c là ba cạnh của một tam giác nên tồn tại các số dương x, y, z
sao cho a = y + z, b = z + x, c = x + y. Khi đó bất đẳng thức cần chứng minh
trở trành

3 3 3 x2 y 2 z 2
xy + yz + zx ≥ xyz(x + y + z) ⇔ + + ≥x+y+z
y z x

(x − y)2 (y − z)2 (z − x)2


⇔ + + ≥ 0.
y z x

Vậy ta có điều phải chứng minh.

Nhận xét.

1. Phép thế trong lời giải được gọi là phép thế Ravi, thường được sử trong các
bài toán chứng minh bất đẳng thức có điều kiện "3 cạnh tam giác". Hình vẽ
sau chỉ ra sự tồn tại của các số x, y, z.
34 HƯỚNG DẪN GIẢI

x
x
b
c

z
y

y z
a

2. Bài tập tương tự:

(a) (Moldova TST 2006): Cho a, b, c là độ dài ba cạnh của một tam giác.
Chứng minh rằng
 
2 b 2 c
 
2 a
 
a −1 +b −1 +c − 1 ≥ 0.
c a b

(b) (IMO 1961): Cho a, b, c là ba cạnh của một tam giác với diện tích S.
Chứng minh rằng √
a2 + b2 + c2 ≥ 4 3S.

Bài 8

Cho a, b, c là các số thực không âm thoả mãn điều kiện a + b + c = 3. Chứng


minh rằng
a b c 1
3
+ 3 + 3 ≥ .
b + 16 c + 16 a + 16 6

Lời giải. Trước hết xin phát biểu không chứng minh một BĐT quen thuộc.
Bổ đề 1. Với x, y, z là các số thực không âm, ta có
4
xy 2 + yz 2 + zx2 + xyz ≤ (x + y + z)3 .
27

Chứng minh BĐT trên rất dễ dàng xin nhường lại cho bạn đọc.
2.1 Đại số 35

Quay lại bài toán. Ta có

ab3
X  X  X
3 a a a
− = − =
16 b3 + 16 16 b3 + 16 16(b3 + 16)

Do đó bất đẳng thức cần chứng minh tương tương

ab3 bc3 ca3 1


3
+ 3 + 3 ≤ .
b + 16 c + 16 a + 16 3

Mặt khác áp dụng bất đẳng AM-GM ta được

ab3 ab3 ab3 ab2


= ≤ √ = .
b3 + 16 b3 + 8 + 8 3
3 8.8.b3 12

Áp dụng tương tự ta sẽ được

ab3 bc3 ca3 ab2 + bc2 + ca2


+ + ≤ .
b3 + 16 c3 + 16 a3 + 16 12

Tới đấy áp dụng Bổ đề 1 ta được


4
ab2 + bc2 + ca2 ≤ ab2 + bc2 + ca2 + abc ≤ (a + b + c)3 = 4.
27

Từ đó ta được điều phải chứng minh. Đẳng thức xảy ra khi (a, b, c) là hoán vị
của (0, 1, 2).

Nhận xét.

1. Chuyển bất đẳng thức cần chứng minh từ dạng "≥" sang "≤" được gọi là kỹ
thuật AM-GM ngược dấu.

2. Bổ đề 1 là một bổ đề quen thuộc hay được sử dụng.

3. Bài tập tương tự (USA National Mathematical Olympiad 2017): Cho a, b, c, d


là các số thực không âm thoả mãn a + b + c + d = 4. Tìm giá trị nhỏ nhất
của
a b c d
3
+ 3 + 3 + 3 .
b +4 c +4 d +4 a +4
36 HƯỚNG DẪN GIẢI

Bài 9

Chứng minh rằng với mọi số thực a, b, c, ta đều có

(b+c−a)2 (c+a−b)2 (a+b−c)2 ≥ (b2 +c2 −a2 )(c2 +a2 −b2 )(a2 +b2 −c2 ). (2.6)

Lời giải. Nếu a2 + b2 + c2 = 0 thì ta sẽ được a = b = c = 0. Khi đó bất đẳng thức


luôn đúng nên ta có thể giả sử a2 + b2 + c2 > 0. Khi đó (2.6) tương đương

(a2 +b2 +c2 )(b+c−a)2 (c+a−b)2 (a+b−c)2 ≥ (a2 +b2 +c2 )(b2 +c2 −a2 )(c2 +a2 −b2 )(a2 +b2 −c2 ).


3(a2 + b2 + c2 ) ≥ (a + b + c)2
nên ta quy về chứng minh

(a + b + c)2 (b + c − a)2 (c + a − b)2 (a + b − c)2

≥ 3(a2 + b2 + c2 )(b2 + c2 − a2 )(c2 + a2 − b2 )(a2 + b2 − c2 ) (2.7)


Mặt khác ta có

(a2 +b2 +c2 )(b2 +c2 −a2 )(c2 +a2 −b2 )(a2 +b2 −c2 ) = 2 a4 b4 + b4 c4 + c4 a4 − a8 + b8 + c8
 

(a + b + c)(b + c − a)(c + a − b)(a + b − c) = 2 a2 b2 + b2 c2 + c2 a2 − a4 + b4 + c4


 

Do đó, (2.7) tương đương


2
2 a2 b 2 + b 2 c 2 + c 2 a2 − a4 + b 4 + c 4 ≥ 3 2 a4 b4 + b4 c4 + c4 a4 − a8 + b8 + c8
    

⇔ a8 + b8 + c8 + 2a2 b2 c2 (a2 + b2 + c2 ) ≥ a2 b2 + b2 c2 + c2 a2 a2 + b2 + c2
 

Đây là bất đẳng thức Schur bậc 4 nên từ đây ta có điều phải chứng minh.
Nhận xét.

1. Đẳng thức (x+y+z)(y+z−x)(z+x−y)(x+y−z) = 2 x2 y 2 − x4 thường


P P
được sử dụng trong các bài toán có biểu thức (y +z −x)(z +x−y)(x+y −z),
được gọi là đẳng thức Heron.

2. Bài tập tương tự: Cho a, b, c > 0. Chứng minh rằng

abc(ab + bc + ca) ≥ (a2 + b2 + c2 )(a + b − c)(c + a − b)(b + c − a).


2.1 Đại số 37

Bài 10

Cho a, b, c, d là các số thực dương. Chứng minh rằng


r r
ab + ac + ad + bc + bd + cd 3 abc + bcd + cda + dab
≥ . (2.8)
6 4

Lời giải. Đặt S2 = ab + ac + ad + bc + bd + cd, S3 = abc + bcd + cda + dab. Xét


đa thức
P (x) = (x − a)(x − b)(x − c)(x − d) = x4 − (a + b + c + d)x3 + S2 x2 − S3 x + abcd.
Ta có P (a) = P (b) = P (c) = P (d) nên theo định lý Rolle, tồn tại α, β, γ dương
sao cho P 0 (α) = P 0 (β) = P 0 (γ) = 0 nên
P 0 (x) = 4 (x − α) (x − β) (x − γ) = 4x3 −4 (α + β + γ) x2 +4 (αβ + βγ + γα) x−4αβγ
(do P 0 (x) có hệ số của bậc cao nhất bằng 4).
Mặt khác ta lại có P 0 (x) = 4x3 − 3(a + b + c + d)x2 + 2S2 x − S3 nên từ đó ta được
4x3 −4 (α + β + γ) x2 +4 (αβ + βγ + γα) x−4αβγ = 4x3 −3(a+b+c+d)x2 +2S2 x−S3 .
Đồng nhất hệ số hai vế, suy ra
S2 = 2 (αβ + βγ + γα)
S3 = 4αβγ.
Khi đó (2.8) trở thành
r
αβ + βγ + γα p
≥ 3 αβγ.
3
Điều này luôn đúng theo bất đẳng thức AM-GM nên ta được điều phải chứng
minh.
Nhận xét.

1. Bài tập tương tự (VMO 1996): Cho các số thực không âm a, b, c, d thoả mãn
2(ab + ac + ad + bc + bd + cd) + abc + bcd + cda + dab = 16.
Chứng minh rằng
2
a + b + c + d ≥ (ab + bc + ca + ad + ac + bd).
3
38 HƯỚNG DẪN GIẢI

2. Ta có bài toán tổng quát (Bất đẳng thức Maclaurin): Cho số nguyên n ≥ 2
P Qk
và các số thực không âm x1 , x2 , x3 , . . . , xn . Đặt Sk = xi j
1≤i1 <i2 <...<ik ≤n j=1
  k1
Sk
và dk = , k = 1, n. Chứng minh rằng
Cnk

d1 ≥ d2 ≥ d3 ≥ . . . ≥ dn .

Chứng minh: xem trong [7].

Bài 11

Tìm tất cả các đa thức P (x) ∈ R[x] thoả mãn

P (x)P (2x2 ) = P (x3 + x). (2.9)

Lời giải. Xét các trường hợp:

TH1: P là đa thức hằng. Trong trường hợp này ta tìm được các đa thức thoả
mãn yêu cầu bài toán là

P (x) = 0; P (x) = 1.

TH2: P không là đa thức hằng. Từ (2.9) dễ thấy P (0) ∈ {0, 1}.

• Nếu P (0) = 0, tồn tại đa thức G(x) ∈ R[x] và k ∈ Z+ thoả mãn

P (x) = xk G(x) và G(0) 6= 0.

Khi đó từ (2.9) ta được


k k
xk G(x) 2x2 G 2x2 = x3 + x G(x3 + x).


k
⇔ 2x2 G(x)G(2x2 ) = (x2 + 1)k G(x3 + x).
Từ đây suy ra G(0) = 0, mâu thuẫn.
• Nếu P (0) = 1, đặt deg P = m với m ∈ Z+ . Từ (2.9) ta cũng được
1
hệ số cao nhất của P là m . Gọi x1 , x2 , . . . , xm là n nghiệm phức của
2
2.1 Đại số 39

đa thức P . Theo định lý Viet ta có |x1 x2 . . . xm | = 2m . Suy ra tồn tại


m0 ∈ {1, 2, . . . , m} sao cho |xm0 | ≥ 2.
Ta sẽ chứng minh nếu số phức z có |z| ≥ 2 thì |z 2 + 1| > 1. Thật vậy,
ta có |z 2 + 1| ≥ |z 2 | − 1 = |z|2 − 1 ≥ 4 − 1 > 1.
Xét dãy số 
z0 = xm0
(zn ) : 3
zn = zn−1 + zn−1 ∀n ∈ Z+
Áp dụng nhận xét trên ta sẽ chứng được bằng quy nạp rằng

|zn | > |zn−1 | ≥ 2 ∀n ∈ Z+ .

Mặt khác ta lại có zn là nghiệm của P (x) với mọi n ∈ N. Suy ra P (x)
có vô số nghiệm, mâu thuẫn.

vậy các đa thức cần tìm thoả mãn đề bài là P (x) = 0; P (x) = 1.

Nhận xét.

1. Ta có bất đẳng thức về modun của số phức như sau: Với z1 , z2 ∈ C thì
|z1 | + |z2 | ≥ |z1 + z2 | ≥ |z1 | − |z2 |.

2. Ta có định lý quan trọng sau: Giả sử f, g, h là các đa thức với hệ số thực thoả
mãn điều kiện deg(f ) + deg(g) = deg(h) và thoả mãn một trong hai điều
kiện sau:

(i) deg(f ) 6= deg(g)


(ii) deg(f ) = deg(g) và f ∗ + g ∗ 6= 0, trong đó f ∗ , g ∗ là hệ số cao nhất của
các đa thức f và g tương ứng.

Khi đó với mọi số nguyên dương n tồn tại nhiều nhất một đa thức P(x) có
bậc n và thoả mãn phương trình P (f (x))P (g(x)) = P (h(x)).
Chứng minh: xem trong [4].
Áp dụng định lý này ta có một số bài toán tương tự:

(a) (VMO 1990) Tìm tất cả các đa thức P ∈ R[x] thoả mãn P (x)P (2x2 ) =
P (2x3 + x).
(b) Tìm tất cả các đa thức P ∈ C[x] thoả mãn P (x)P (−x) = P (x2 ).
40 HƯỚNG DẪN GIẢI

Bài 12

Cho đa thức P (x) ∈ R[x] và P (x) ≥ 0 với mọi x ∈ R. Chứng minh rằng đa
thức P (x) có thể biểu diễn dưới dạng

P (x) = (A(x))2 + (B(x))2

trong đó A(x), B(x) cũng là các đa thức có hệ số thực.

Lời giải. Ta bỏ qua trường hợp đơn giản là P (x) ≡ C (với C là hằng số). Do
P (x) ≥ 0 với mọi x ∈ R nên đa thức P (x) có bậc bằng 2n và có thể phân tích
được dưới dạng tích các nhân tử bậc hai không âm, nghĩa là:
n
Y
(ai x + bi )2 + c2i
 
P (x) =
i=1

trong đó ai , bi , ci ∈ R với mọi i = 1, 2, . . . , n.


Từ hằng đẳng thức
p2 + q 2 r2 + s2 = (pr + qs)2 + (ps − qr)2
 

ta được nhận xét sau: Tích của hai biểu thức có dạng (u(x))2 + (v(x))2 cũng là
một biểu thức có dạng đó. Từ đó bằng quy nạp ta sẽ thu được điều phải chứng
minh.

Bài 13
Cho đa thức

f (x) = xn + an−2 xn−2 + an−3 xn−3 + . . . + a1 x + a0 ∈ R[x].

n!
Chứng minh rằng tồn tại i ∈ {1, 2, . . . , n} sao cho |f (i)| ≥ .
Cni

Lời giải. Đặt g(x) = f (x) − xn . Áp dụng công thức nội suy Lagrange ta được
 
 
 
n n n  n
X  Y x − j  X 1 Y 
g(x) = g(i) · = g(i) · · (x − j).
 
n
i − j
   Q
(i − j) j=1
 
i=1 j=1 i=1  
j6=i j=1 j6=i
j6=i
2.1 Đại số 41

Ta có
n
Y
(i − j) = (i − 1)(i − 2) . . . [i − (i − 1)][i − (i + 1)] . . . (i − n)
j=1
j6=i

(−1)n−i n!
= (−1)n−i (i − 1)!(n − i)! = .
iCni
Nên  
n i n
n−i i · g(i) · Cn
X  Y 
g(x) = (−1) · · (x − j)

i=1
n! 
j=1
j6=i

So sánh hệ số của xn−1 ở 2 vế ta được


n n
X
n−i i · g(i) · Cni X i · (f (i) − in ) · Cni
(−1) · =0⇔ (−1)n−i · =0
i=1
n! i=1
n!
n
X n
X
⇔ Cni (−1)n−i in+1 = iCni (−1)n−i f (i).
i=1 i=1
n!
Giả sử |f (i)| < ∀i = 1, 2, . . . , n thì suy ra
Cni
!
n n n
X X X n(n + 1)!
iCni (−1)n−i f (i) = iCni |f (i)| < i · n! =

2


i=1 i=1 i=1

X n n(n + 1)!
i n−i n+1
⇒ Cn (−1) i < .


i=1
2
Mặt khác ta lại có
n
X n(n + 1)!
Cni (−1)n−i in+1 = .
i=1
2

Thật vậy xét h(x) = xn − (x − 1)(x − 2) . . . (x − n). Áp dụng công thức nội suy
Lagrange ta được
   
n n n n+1 n
X  Y x − j X  n−i i · Cni Y 
h(x) = h(i) ·  = (−1) · · (x − j)
i=1

j=1
i − j 
i=1
 n! j=1

j6=i j6=i
42 HƯỚNG DẪN GIẢI

So sánh hệ số của xn−1 ở 2 vế ta được


n
Cni (−1)n−i in+1
P
i=1
1 + 2 + ... + n =
n!
n
Cni (−1)n−i in+1
P
n
n(n + 1) i=1 n(n + 1)! X i
⇔ = ⇔ = Cn (−1)n−i in+1 .
2 n! 2 i=1

Vậy điều giả sử là sai, từ đó ta được điều phải chứng minh.


Nhận xét. Một trong những ứng dụng của công thức nội suy Lagrange là tính
tổng và chứng minh bất đẳng thức liên quan đến đa thức.
Một số bài toán tương tự:
n n
(−1)k · Cnk · k n và (−1)k · Cnk · k n+2 .
P P
1. Cho số nguyên dương n. Tính
k=0 k=0

2. (VMO 1977) Cho n + 1 số nguyên đôi một phân biệt x0 , x1 , . . . , xn . Xét các
đa thức P (x) = xn + an−1 xn−1 + . . . + a1 x + a0 ∈ R[x]. Chứng minh rằng
n!
max |P (xi )| ≥ n .
i∈0,n 2

Bài 14

Cho số nguyên n ≥ 3 và P (x) ∈ R[x] thoả mãn

P (x) = xn + an−3 xn−3 + an−4 xn−4 + . . . + a1 x + a0 .

Biết ít nhất một số trong các số a0 , a1 , . . . , an−3 khác 0. Chứng minh rằng
P (x) không thể có toàn nghiệm thực.

Lời giải. Gọi x1 , x2 , . . . , xn là các nghiệm của P (x). Giả sử xi ∈ R với mọi i =
1, 2, . . . , n, khi đó theo định lý Viet ta có
n
X
xi = 0
i=1

n n
!2
X X X X
xi xj = 0 ⇔ x2i = xi −2 xi xj = 0
1≤i<j≤n i=1 i=1 1≤i<j≤n
2.1 Đại số 43

nên xi = 0 với mọi i = 1, 2, . . . , n, tức là P (x) chỉ có nghiệm thực duy nhất là
0, dẫn tới a0 = a1 = . . . = an−3 (mâu thuẫn với giả thiết). Vậy điều giả sử là sai
nên ta có điều phải chứng minh.

Bài 15

Cho n là số nguyên dương chẵn. Một đa thức monic bậc n có n nghiệm


thực (không nhất thiết phân biệt). Giả sử y là sốpthực dương
p thoả với mọi
số thực t < y, ta có P (t) > 0. Chứng minh rằng n P (0) − n P (y) ≥ y.

Lời giải. Gọi x1 , x2 , . . . , xn là các nghiệm của P (x). Nếu tồn tại i ∈ {1, 2, . . . , n}
sao cho xi < y thì P (xi ) > 0 (mâu thuẫn vì xi là nghiệm của P (x)). Vậy ta có
0 < y ≤ xi i = 1, n .
Mặt khác ta có P (x) = (x − x1 ) (x − x2 ) . . . (x − xn ) và n chẵn nên

P (0) = (−1)n x1 x2 . . . xn = x1 x2 . . . xn > 0 (do n là số chẵn)

P (y) = (y − x1 ) (y − x2 ) . . . (y − xn ) = (−1)n (x1 − y) (x2 − y) . . . (xn − y)


= (x1 − y) (x2 − y) . . . (xn − y) ≥ 0.
Điều cần chứng minh trở thành
h p in
y + n (x1 − y) (x2 − y) . . . (xn − y) ≤ x1 x2 . . . xn .

Áp dụng BĐT Holder ta được

x1 x2 . . . xn = (y + x1 − y) (y + x2 − y) . . . (y + xn − y)
h√ p in h p in
≥ n y n + n (x1 − y) (x2 − y) . . . (xn − y) = y + n (x1 − y) (x2 − y) . . . (xn − y) .

Đây chính là điều phải chứng minh.

Bài 16

Tìm các đa thức f (x), g(x) hệ số nguyên thoả mãn

f (g(x)) = x2017 + 2018x + 1. (2.10)


44 HƯỚNG DẪN GIẢI

Lời giải. Ta bỏ qua trường hợp đơn giản khi f hoặc g là các đa thức hằng. Gọi
m = deg f và n = deg g. Từ giả thiết ta được mn = 2017.

TH1: m > 1; n > 1. Trong (2.10), đạo hàm hai vế, ta được

g 0 (x)f 0 (g(x)) = 2017x2016 + 2018 (2.11)


..

 2017 6 . 2


.
Mặt khác ta có 2018 .. 2 nên theo tiêu chuẩn Eisenstein ta có 2017x2016 +
.

2018 6 .. 22


2018 bất khả quy trên Z[x]. Rõ ràng điều này mâu thuẫn với (2.11).
2017
ci xi với a, b, c1 , c2 , . . . , c2017 ∈
P
TH2: m = 1; n = 2017. Đặt f (x) = ax+b và g(x) =
i=0
Z và a, c2017 6= 0. Khi đó (2.10) trở thành
2017
!
X
a ci xi + b = x2017 + 2018x + 1.
i=0

Đồng nhất hệ số hai vế ta được:



ac2017 = 1; aci = 0 i = 2, 2016 ; ac1 = 2018; ac0 + b = 1.

Từ đây ta sẽ tìm được các đa thức sau thoả mãn đề bài:

f (x) = x + b; g(x) = x2017 + 2018x + 1 − b

f (x) = −x + b; g(x) = −x2017 − 2018x + b − 1.


2017
ki xi và g(x) = sx+t với s, t, k1 , k2 , . . . , k2017 ∈
P
TH3: n = 1, m = 2017. Đặt f (x) =
i=0
Z và s, k2017 6= 0. Làm tương tự như TH2 ta sẽ được s = ±1.

• Nếu s = 1, ta có
g(x − t) = x
f (g(x − t)) = (x − t)2017 + 2018(x − t) + 1.
Suy ra

f (x) = (x − t)2017 + 2018(x − t) + 1; g(x) = x + t.


2.1 Đại số 45

• Nếu s = −1, tương tự như trên, ta cũng tìm được

f (x) = −(x − t)2017 − 2018(x − t) + 1; g(x) = −x + t.

Vậy tất cả các cặp đa thức f, g cần tìm là

f (x) = x + b; g(x) = x2017 + 2018x + 1 − b

f (x) = −x + b; g(x) = −x2017 − 2018x + b − 1


f (x) = (x − t)2017 + 2018(x − t) + 1; g(x) = x + t
f (x) = −(x − t)2017 − 2018(x − t) + 1; g(x) = −x + t.

Bài 17

xn + a
Cho đa thức hệ số nguyên P (x) = an n−1 x
n−1
+ . . . + a1 x + a0 (an 6= 0).
ai
Giả sử tồn tại m sao cho m ≥ max + 2 và P (m) là số nguyên tố.
i∈0,n an
Chứng minh rằng P (x) bất khả quy trên Z.

Lời giải. Giả sử tồn tại các đa thức Q, H khác hằng thoả mãn P (x) = Q(x)H(x).
Do P (m) là số nguyên tố nên |Q(m)| = 1 hoặc |H(m)| = 1. Giả sử |Q(m)| =
1, gọi deg Q = k và xi (i ∈ 1, k) là các nghiệm của Q(x). Khi đó Q(x) =
Qk k
Q
a (x − xi ) với a ∈ Z \ {0}, suy ra 1 = |Q(m)| = |a| |m − xi |.
i=1 i=1

Do |a| ≥ 1 nên tồn tại j ∈ {1, 2, . . . , k} sao cho |m − xj | ≤ 1. Mà |m − xj | ≥


|m| − |xj | nên suy ra |xj | ≥ m + 1 > 1.
n
ai xij = 0, suy ra
P
Mặt khác, do xj cũng là nghiệm của P (x) nên
i=0
n−1
n−1 n−1
!
|xj |n − 1 M |xj |n
X a X
n i i ai i
X
|xj | = x ≤ an |xj | ≤ M |xj |i =M· <


i=0
an j i=0 i=0
|xj | − 1 |xj | − 1

⇒ |xj | < 1 + M ≤ 1 + m − 2 = m − 1
Tới đây ta được một điều mâu thuẫn nên điều giả sử ban đầu là sai, tức là P (x)
bất khả quy trên Z.
46 HƯỚNG DẪN GIẢI

Nhận xét. Qua bài 17 và bài 18, ta sẽ phát biểu lại một số tiêu chuẩn để chứng
minh đa thức bất khả quy
n
ai xi (i = 0, n)
P
1. Tiêu chuẩn 1 (tiêu chuẩn Eisenstenin): Cho đa thức P (x) =
i=0
với ai ∈ Z. Khi đó nếu tồn tại số nguyên tố p thoả mãn đồng thời các điều
kiện:

(i) Tất cả các hệ số trừ an chia hết cho p.


(ii) a0 không chia hết cho p2 .

Khi đó P (x) bất khả quy trên Z.


n
ai xi (i =
P
2. Tiêu chuẩn 2 (tiêu chuẩn Eisenstenin suy rộng): Cho đa thức P (x) =
i=0
0, n) với ai ∈ Z. Khi đó nếu tồn tại số nguyên tố p thoả mãn đồng thời các
điều kiện:

(i) an không chia hết cho p.


(ii) a0 không chia hết cho p2 .
(iii) a0 , a1 , . . . , an−k chia hết cho p

Khi đó nếu P (x) = H(x)G(x) thì một trong hai đa thức H(x), G(x) có bậc
nhỏ hơn k.
n
ai xi (i = 0, n) với
P
3. Tiêu chuẩn 3 (tiêu chuẩn Perron): Cho đa thức P (x) =
i=0
ai ∈ Z và a0 6= 0. Khi đó, nếu

|an−1 | > |a0 | + |a1 | + . . . + |an−2 | + |an |,

thì P (x) bất khả quy trên Z.

4. Tiêu chuẩn 4 (tiêu chuẩn đánh giá về nghiệm): Cho P ∈ Z[x] và m ∈ Z thoả
mãn |P (m)| là 1 hoặc là một số nguyên tố sao cho P (x) không có nghiệm
thuộc {z ∈ C : |z − m| ≤ 1}.
Khi đó P (x) bất khả quy trên Z.
2.1 Đại số 47

Bài 18

Cho số nguyên dương n. Tìm tất cả đa thức P thoả mãn đồng thời hai điều
kiện sau:

i. P (x) = an xn + an−1 xn−1 + . . . + a1 x + a0 trong đó {a0 , a1 , . . . , an } =


{0, 1, 2, . . . , n}.

ii. Đa thức P có n nghiệm thực phân biệt.

Lời giải. Từ giả thiết suy ra an 6= 0.


Gọi x1 , x2 , . . . , xn là n nghiệm thực phân biệt của đa thức P . Theo định lý Viete,
ta có 
Pn an−1

 ε 1 = xi = −
an



 i=1


 . . .
n Q n a1

xj = (−1)n−1
P
εn−1 =


 i=1 j=1 an

 j6=i
 n a0
xi = (−1)n
 Q
 εn =


i=1 an
Do {a0 , a1 , . . . , an } = {0, 1, 2, . . . , n} nên P (x) > 0 với x > 0. Suy ra xi ≤ 0, ∀i =
1, n.

εi > 0
Nếu xi < 0, ∀i = 1, n thì , ∀i = 1, n, nên suy ra ai 6= 0, ∀i = 0, n − 1
εi < 0
(mâu thuẫn với giả thiết).
Vậy đa thức P có một nghiệm bằng 0, giả sử xn = 0, từ đó ta cũng có a0 = 0.

• Với n = 1: ta tìm được đa thức duy nhất thoả mãn yêu cầu bài toán là
P (x) = x.
• Với n = 2: ta tìm được tất cả đa thức thoả mãn yêu cầu bài toán là P (x) =
x2 + 2x; P (x) = 2x2 + x.
• Với n = 3: ta tìm được tất cả đa thức thoả mãn yêu cầu bài toán là P (x) =
x3 + 3x2 + 2x; P (x) = 2x3 + 3x2 + x.
• Với n ≥ 4
Đặt Q (x) = an xn−1 + an−1 xn−2 + . . . + a2 x + a1 thì ta có Q(x) có n − 1
nghiệm âm phân biệt x1 , x2 , . . . , xn−1 và P (x) = xQ(x).
48 HƯỚNG DẪN GIẢI

Đặt ti = −xi , ∀i = 1, n − 1, suy ra

ti > 0, ∀i = 1, n − 1
n−1
X an−1
ti = (2.12)
i=1
an
n−1 n−1
X Y a2
tj = (2.13)
i=1 j=1
an
j6=i

n−1
Y a1
ti = (2.14)
i=1
an

Từ (2.13) và (2.14) ta được


n−1
X 1 a2 a1 a2
= : = (2.15)
t
i=1 i
an an a1

Từ (2.12) và (2.15) ta được


n−1
! n−1
!
X X 1 an−1 a2
ti = · (2.16)
i=1
t
i=1 i
an a1

Theo bất đẳng thức AM-GM ta được


n−1
! n−1 !
X X1
ti ≥ (n − 1)2 .
i=1
t
i=1 i

Do {a0 , a1 , . . . , an } = {0, 1, 2, . . . , n} nên

an−1 a2 n (n − 1)
· ≤
an a1 2
n
Từ đây ta suy ra n − 1 ≤ , suy ra n ≤ 2 (mâu thuẫn vì n ≥ 4).
2

Kết luận:

• Nếu n = 1 thì tất cả đa thức thoả mãn yêu cầu bài toán là P (x) = x.
2.1 Đại số 49

• Nếu n = 2 thì tất cả đa thức thoả mãn yêu cầu bài toán là P (x) = x2 +
2x; P (x) = 2x2 + x.
• Nếu n = 3 thì tất cả đa thức thoả mãn yêu cầu bài toán là P (x) = x3 +
3x2 + 2x; P (x) = 2x3 + 3x2 + x.
• Nếu n ≥ 4 thì không tồn tại đa thức nào thoả mãn yêu cầu bài toán.

Bài 19

Tìm tất cả các đa thức hệ số nguyên f sao cho n|m ⇒ f (n)|f (m) với mọi
các số nguyên dương m, n.

Lời giải. Gọi f ∈ Z[x] là đa thức thoả mãn điều kiện đề bài và a ∈ Z \ {0} là hệ
số cao nhất của f . Đặt deg f = k (k ∈ N).
Với mỗi n nguyên, ta thấy f (n)|f (dn), ∀d ∈ Z nên
k
X
f (n) | Cki (−1)i f ((d − i)n) = k!ank .
i=0

Suy ra
f (n)|k! f (n) − ank , ∀n ∈ Z.

(2.17)
 
Mà deg f (x) > deg f (x) − axk nên tồn tại n0 ∈ N đủ lớn sao cho

|f (n)| > f (n) − ank , ∀n > n0 . (2.18)
Từ (2.17) và (2.18) suy ra
f (n) = ank , ∀n > n0 .
Vậy tất cả các đa thức thoả mãn điều kiện đề bài là f (n) = ank (k ∈ N).
Nhận xét.
k
Cki (−1)i f ((d − i)n) = ak!nk bằng công thức nội
P
1. Chứng minh đẳng thức
i=0
suy Lagrange.
2. Ta có một bài toán "họ hàng" của bài toán trên như sau: Tìm tất cả các đa
thức f với hệ số nguyên sao cho f (n)|f (m) ⇒ n|m.
Chứng minh: xem trong [9].
50 HƯỚNG DẪN GIẢI

Bài 20

Cho đa thức monic P (x) bậc n > 1 (tức P có hệ số của bậc lớn nhất bằng
1) có n nghiệm thực x1 , x2 , ...xn phân biệt khác 0. Chứng minh rằng:

1 1 1 (−1)n+1
+ + ... + = .
x1 P 0 (x1 ) x2 P 0 (x2 ) xn P 0 (xn ) x1 x2 ...xn

Lời giải. Đặt Pi (x) là đa thức bậc n − 1 có dạng:


Y
Pi (x) = (x − xj )
j6=i

Khi đó ta có:
Pi (xj ) = 0, ∀j 6= i

n
X
0
P (x) = Pi (x)
i=1

Ngoài ra ta có:
P 0 (xi ) = Pi (xi ).
Xét đa thức:
n
X Pi (x)
Q(x) = − 1.
i=1
P 0 (xi )

Vì các đa thức Pi có bậc là n − 1 nên Q cũng có bậc là n − 1. Thay các giá trị xi
vào ta có:
n
X Pi (xj ) Pj (xj )
Q(xj ) = 0
−1= 0 −1=0
i=1
P (xj ) P (xj )

Vậy ta có Q có n nghiệm. Điều đó dẫn đến Q ≡ 0. Cuối cũng chỉ cần đống nhất
hệ số sẽ có kết quả như mong muốn.

1. Đồng nhất hệ số bậc cao nhất ta có:

1 1 1
+ + ... + = 0.
P 0 (x 1) P 0 (x 2) P 0 (x n)
2.2 Hình học 51

2. Đồng nhất hệ số bậc tự do ta có:

(−1)n−1 x2 x3 ...xn (−1)n−1 x1 x3 ...xn (−1)n−1 x1 ...xn−1 xn


+ + ... + = 1.
P 0 (x1 ) P 0 (x2 ) P 0 (xn )
Hay tương đương:

1 1 1 (−1)n+1
+ + ... + = .
x1 P 0 (x1 ) x2 P 0 (x2 ) xn P 0 (xn ) x1 x2 ...xn

2.2 Hình học


Bài 1

Cho 4ABC. điểm P di chuyển trên cạnh BC. Q, R lần lượt là hai điểm
đối xứng với P qua CA, AB. Lấy điểm M nằm trên đường tròn ngoại tiếp
4AQR sao cho AM k BC. Chứng minh đường thẳng P M đi qua một điểm
cố định khi P di chuyển trên cạnh BC.

A M
Q

R O

B C
P

Lời giải. Gọi giao điểm của hai đường thẳng BR và CQ là N . Biến đổi góc, ta
được RN
\ Q = 180 − 2BAC
[ ⇒ N thuộc đường tròn ngoại tiếp 4AQR.
52 HƯỚNG DẪN GIẢI

Gọi giao điểm thứ hai của hai đường tròn ngoại tiếp các tam giác N BC và N QR
là G. Dễ dàng chứng minh 4GBR ∼ 4GCQ (g.g), dẫn tới

GB RB BP
= = .
GC QC CP

Điều này cho ta GP là phân giác trong góc BGC.


\
Gọi O là tâm ngoại tiếp 4ABC. Để ý rằng BGC\ = BN
\ C = 180o − QAR
[ =
o
180 − BOC
\ ⇒ O thuộc đường tròn ngoại tiếp 4BGC ⇒ GO là phân giác
\ ⇒ G, P, O thẳng hàng.
trong góc BGC
Gọi T là giao điểm của hai đường thẳng AG và QR. Bằng biến đổi góc, ta được
[ = RT
AGR [ G. Từ đây dẫn tới 4GQR ∼ 4GCB, mà GT, GP lần lượt là các
phân giác của góc RAQ,
[ BGC\ ⇒ RT [ G = BP
\ G ⇒ AM
\ G = BP
\ G. Mà giả thiết
cho ta AM k BC ⇒ G, P, M thẳng hàng, tức là G, P, O, M thẳng hàng. Vậy BM
đi qua điểm O cố định.

Bài 2

Cho 4ABC có đường cao AH. Giả sử đường tròn đường kính BC tiếp xúc
đường tròn nội tiếp 4ABC. Chứng minh rằng AH + BC = AB + AC.

Lời giải. Đặt ID = r(D ∈ BC) là bán kihs đường tròn nội tiếp 4ABC. Vì
đường tròn nội tiếp 4ABC tiếp xúc trong đường tròn đường kính BC (bạn
đọc hãy thử giải thích tại sao hai đường tròn đó không thể tiếp xúc ngoài) nên
BC
IM = − r.
2
2.2 Hình học 53

B H D M C

a+b+c
Lời giải. Đặt ID = r, BC = a, CA = b, AB = c, p = . Từ đó, ta có
2
BC |DB − DC|
được ID = − r, M D = .
2 2
Ta có  2  2
2 2 2 BC 2 DB − DC
IM = ID + DM ⇒ −r =r +
2 2
BC 2 (DB − DC)2
⇔ − BC · r =
4 4
2
(DB + DC) (DB − DC)2
⇔ − BC · r =
4 4
DB · DC (p − b)(p − c)
⇔r= = .
BC a
Lại có
4S 2 4p(p − a)(p − b)(p − c)
AH 2 = 2
=
BC a2
S
4. (p − a)r
= r = AH(p − a) ⇔ AH = p − a.
a
54 HƯỚNG DẪN GIẢI

Vậy, ta có điều phải chứng minh.

Nhận xét. Các bài hình học có nội dung tính toán đôi lúc sẽ gây khó khăn cho các
bạn học sinh, vì nó đòi hỏi sự kết hợp nhuần nhuyễn giữa khả năng nhìn hình và
kĩ năng tính toán. Các cách tính toán thường dùng để xử lí các bài toán hình học
gồm có:

• Đặt hệ trục toạ độ.

• Tính toán dựa trên độ dài các cạnh.

• Dùng định lí sin, hệ thức cos, định lí Menelaus và định lí Ceva dạng lượng
giác.

Sau đây là một số bài tập làm thêm cho các bạn.

1. Xét một họ các tam giác cân có tính chất sau; Chúng có đáy nằm trên một
đường thẳng d cố định, có đỉnh A thuôc đáy là một điểm cố định và có bán
kính đường tòn nội tiếp bằng r không đổi. Chứng minh rằng cạnh bên không
đi qua A của các tam giác này luôn tiếp xúc một đường tròn cố định.

2. Cho tam giác ABC có BD, CE là hai đường phân giác D ∈ AC, E ∈ AB>
[ = 60o .
Giả sử ID = IE, chứng minh 4ABC cân tại A hoặc BAC

3. Cho 4ABC có 2BC = AB + AC. Gọi O, I lần lượt là tâm đường tròn ngoại
tiếp và nội tiếp 4ABC. Chứng minh rằng 4AIO vuông tại I.

4. (VMO 2009) Cho 2 điểm cố định A, B và điểm C di động trên mặt phẳng
ˆ = a (0 < a < 180) không đổi cho trước. Hình chiếu của tâm
sao cho ACB
đường tròn nội tiếp I của tam giác ABC xuống ba cạnh AB, BC, CA lần
lượt là D, E, F . AI và BI cắt EF lần lượt tại M, N .

(a) Chứng minh độ dài M N không đổi.


(b) Chứng minh đường tròn (DM N ) luôn đi qua một điểm cố định.

5. (Trung Quốc MO 2008) Cho tam giác nhọn không cân ABC có I là tâm
đường tròn nội tiếp. Gọi E là tiếp điểm của I trên cạnh BC, đoạn thẳng AE
cắt (I) tại điểm thứ hai là D khác E. Trên đường thẳng AE lấy điểm F sao
cho CE = CF . Đường thẳng BD cắt đường thẳng CF tại K. Chứng minh
rằng KF = CE.
2.2 Hình học 55

6. (IMO 2001) Cho tam giác ABC có A b = 60o và AP, BQ lần lượt là các phân
giác trong góc A, b Biết rằng AB + BP = AQ + QB, tính các góc còn lại
b B.
của tam giác ABC.

Bài 3

Cho 4ABC có hai đường cao BE và CF . Đường tròn bàng tiếp góc A là
(Ia ). Hai tiếp tuyến chung trong của (AEF ) và (Ia ) cắt BC tại P và Q.
Chứng minh rằng BP = CQ.

A
E
F
H

D C
B M G N

Lời giải. Bài toán này đã xuất hiện trong đề thi Olympiad hình học toàn Liên
bang Nga mở rộng (Sharygin Olympiad 2017). Gọi J là tâm đường tròn ngoại
56 HƯỚNG DẪN GIẢI

tiếp 4AEF , S là giao của hai tiếp tuyến chung trong, (I), (K) lần lượt là đường
tròn nội tiếp các tam giác ABC, SP Q; (I), (K), (Ia ) lần lượt tiếp xúc với BC tại
D.D0 , T . Gọi L là điểm đối xứng với T qua Ia . Ta có JA k Ia L và S là tâm vị tự
trong của (J) và (Ia ) nên A, S, L thẳng hàng.
Lại có A là tâm vị tự ngoài của (I) và (Ia ), đồng thời ID k Ia L nên A, D, L thẳng
hàng. Do đó A, D, S, L thẳng hàng. Mặt khác, S là tâm vị tự ngoài của (K) và
(Ia ), đồng thời KD0 k Ia L nên S, D0 , L thẳng hàng. Suy ra D ≡ D0 . Từ đó ta có
DP = T Q. Mà DB = T C nên BP = CQ.
Nhận xét. Ngoài lời giải trên, tác giả Nguyễn Văn Linh còn có phần mở rộng rất
đáng chú ý cho bài toán này, xin giới thiệu cùng bạn đọc.
Trong lời giải bài toán trên chỉ dùng tới dữ kiện JA ⊥ BC, do đó ta mở rộng bài
toán như sau:
Bài toán. Cho tam giác ABC với (Ia ) là đường tròn bàng tiếp góc A.b Một đường
tròn bất kì qua B, C cắt AC, AB lần lượt tại E, F . Tiếp tuyến chung trong của
(AEF ) và (Ia ) cắt BC tại P, Q. Chứng minh rằng BP = CQ.
b bằng đường tròn nội tiếp (I), ta được bài
Nếu thay đường tròn bàng tiếp góc A
toán tương tự.
Bài toán. Cho tam giác ABC với (I) là đường tròn nội tiếp. Một đường tròn bất
kì qua B, C cắt AC, AB lần lượt tại E, F . Tiếp tuyến chung trong của (AEF ) và
(I) cắt BC tại P, Q. Chứng minh rằng BP = CQ.

Để ý rằng ở bài toán ban đầu, đường tròn đường kính P Q tiếp xúc với đường tròn
(AEF ) và (O). Vậy ta có bài toán sau:
Bài toán. Cho tam giác ABC nội tiếp đường tròn (O), ngoại tiếp đường tròn (I).
Một đường tròn tâm J bắt kì qua B, C cắt AC, AB tại E, F . Tiếp tuyến chung
ngoài của (AEF ) và (I) cắt BC tại P, Q. Chứng minh rằng (J, JP ) tiếp xúc với
(O) và (AEF ).

Tới đây ta thay đường tròn nội tiếp (I) bằng đường tròn bàng tiếp (Ia ) của góc
Ab với P 0 , Q0 là giao của tiếp tuyến chung trong của (AEF ) và (Ia ) với BC thì
(J, JP 0 ) vẫn tiếp xúc với (O), (AEF ) lần lượt tại M, K. Do đó P 0 ≡ P, Q0 ≡ Q. Ta
thu được bài toán sau:
Bài toán. Cho tam giác ABC ngoại tiếp đường tròn (I), (Ia ) là tâm đường tròn
bàng tiếp góc A.
b Một đường tròn bất kì qua B, C cắt AC, AB lần lượt tại E, F .
Một tiếp tuyến chung ngoài của (AEF ) và (I) cắt BC tại P . Chứng minh rằng P
nằm trên tiếp tuyến chung trong của (AEF ) và (Ia ).
2.2 Hình học 57

Bài 4

Cho đường tròn (O1 ) và (O2 ) ngoài nhau có AB là một tiếp tuyến cung
ngoài và CD là một tiếp tuyến chung trong. Gọi P = AB ∩ CD, Q =
AD ∩ BC. Chứng minh rằng P Q ⊥ O1 O2 .

Lời giải. Ta sẽ chứng minh AC, BD, O1 O2 đồng quy. Gọi Y = P Q ∩ BD ⇒


YB XB XB
(BD, Y X) = −1. Mặt khác, ta có 4O1 AC ∼ 4P BD ⇒ = = ·
YD XD MP
MP AX CM AX
= · = ⇒ 4O1 AX ∼ 4P BY ⇒ O \2P Q = X\1 OM ⇒
XD AM CX CX

1 P O2 = 90 . Từ đây, ta có điều phải chứng minh.
O\

O1
X
O2

Q
D
Y
B

Bài 5

Cho tứ giác ABCD nội tiếp đường tròn tâm O và một điểm P thuộc cung
CD không chứa A và B. Gọi E = P A ∩ BD, G = BP ∩ AC, H = AP ∩
CD, F = BP ∩ CD, Q = EF ∩ HG. Chứng minh rằng P Q luôn đi qua một
điểm cố định khi P di chuyển.
58 HƯỚNG DẪN GIẢI

D
A

H
R Q P

O F
G

B C

Lời giải. Gọi P Q cắt (O) tại điểm thứ hai


 là R và tiếp tuyến
 tại P cắt CD tại X.
P C B
Áp dụng định lí Pascal cho bộ 6 điểm , ta có ba điểm E, G,
D P A
X thẳng hàng. Suy ra, A(XQ, GE) = −1 ⇒ P (P R, BA) = −1 ⇒ AP BR điều
hoà. Từ đó, ta kết luận P Q đi qua giao điểm của hai tiếp tuyến tại A và B. Bài
toán chứng minh hoàn tất.
Nhận xét.

1. Định lí Pascal có thể mở rộng cho một đường conic bất kì (không nhất thiết
phải là đường tròn).
2. Một số bài tập ứng dụng định lí Pascal:
(a) Tứ giác ABCD nội tiếp có tiếp tuyến tại A và B cắt nhau ở N1 , tại B
và C cắt nhau ở N2 , tại C và D cắt nhau ở N10 , tại D và A cắt nhau
ở N20 ; các đường thẳng AD và BC cắt nhau ở M1 , các đường thẳng
AB và CD cắt nhau ở M2 ; AC và BD cắt nhau ở P . Chứng minh bốn
điểm M1 , N1 , P, N10 thẳng hàng và bốn điểm M2 , N2 , P, N20 thẳng hàng.
(b) Cho một đường tròn với hai dây AB và CD không song song. Đường
vuông góc với AB kẻ từ A cắt đường vuông góc với CD kẻ từ C và từ D
lần lượt tại M, P . Đường vuông góc với AB kẻ từ B cắt đường vuông
góc với CD kẻ từ C và D lần lượt tại Q và N . Chứng minh rằng các
đường thẳng AD, BC, M N đồng quy và các đường thẳng AC, BD, P Q
cũng đồng quy.
2.2 Hình học 59

(c) (IMO Shortlist 2011) Cho ABC là một tam giác với đường tròn nội
tiếp tâm I và đường tròn ngoại tiếp (C). D và E là giao điểm thứ hai
của (C) với các tia AI và BI tương ứng. DE cắt AC tại điểm F , và cắt
BC tại điểm G. P là giao điểm của đường thẳng đi qua F song song
với AD và đường thẳng qua G song song với BE. Giả sử rằng K là
giao điểm của các tiếp tuyến của (C) tại A và B. Chứng minh rằng ba
đường thẳng AE, BD và KP song song hoặc đồng 0quy.

Bài 6

Cho tam giác ABC nội tiếp đường tròn tâm O cố định có hai điểm BC cố
định, A di chuyển trên (O). Gọi AD, BE, CF là ba đường cao của tam giác
cắt nhau tại H. DE, DF lần lượt cắt HB, HC tại Q, R. Gọi M là trung điểm
QR. Chứng minh rằng HM đi qua điểm cố định.

Lời giải. Trước hết xin phát biểu và chứng minh bổ đề sau.
Bổ đề 2. Cho 4ABC nội tiếp đường tròn (O) có hai tiếp tuyến tại B và C cắt
nhau tại T . Phân giác BE, CF cắt nhau tại I. Khi đó IT chia đôi EF .

M E
F
O

B C

T
60 HƯỚNG DẪN GIẢI

Thật vậy, chứng minh được BF


\ C = F[ \ = EBT
CT , CEB [ . Gọi M là giao điểm
của EF và IT . Ta có:
SIM E SIM E SIBT SIEC SIF B SICT
= · · · ·
SIM F SIBT SIEC SIF B SICT SIM F
IM · IE IB · BT IF · F B IC · IT F B IE IC
= · · · = · · =1
IB · IT IE · EC IC · CT IM · IF EC IF IB
Từ đó, ta có điều phải chứng minh.
S

A
E0

E Y
F0
F
H O
R
M R0
Q M0
X
C
Q0

D
B

D0

T
2.2 Hình học 61

Quay lại bài toán. Gọi D0 , E 0 , F 0 lần lượt là giao điểm thứ hai của ADBE, CF
với đường tròn (O). Tiếp tuyến tại B và F 0 cắt nhau tại X, tiếp tuyến tại F 0 và
E 0 cắt nhau tại S, tiếp tuyến tại E 0 và C cắt nhau tại Y .
Ta có H là tâm đường tròn nội tiếp 4DEF , từ đó suy ra H cũng chính là trực
tâm 4D0 E 0 F 0 . Sử dụng Bổ đề 2 cho 4D0 E 0 F 0 , suy ra H, M, S thẳng hàng. Kết
hợp với tứ giác XSY T ngoại tiếp đường tròn (O), ta có ST, CF 0 , BE 0 đồng quy
tại H, suy ra H, M , T thằng hàng, kết thúc chứng minh.

Bài 7

Cho 4ABC nội tiếp đường tròn (O) cố định có B, C cố định và A di động
trên (O). Gọi E, F lần lượt là điểm đồi xứng của B, C qua CA, AB. Gọi
M = CE ∩AB, N = BF ∩AC. Chứng minh rằng đường thẳng qua A vuông
góc với M N đi qua một điểm cố định.

Hướng dẫn.
62 HƯỚNG DẪN GIẢI
P

F
E
K

M
N

B C

Gọi P = BN ∩ CM . Ta có A là tâm đường tròn nội tiếp 4P BC. Do đó, BAC


[ =
90◦ + BP
\ C
⇒ BP
\ C = const. Suy ra tâm đường tròn ngoại tiếp ∆P BC cố định
2
(gọi tâm đường tròn này là K).
Gọi J là tâm đường tròn bàng tiếp góc P của ∆P BC. Ta chứng minh được
KJ ⊥ M N k d (với d là đường thẳng qua A vuông góc với M N ). Gọi T là
trung điểm cung BC không chứa P của đường tròn (P BC). Suy ra được T là ta
đường tròn (ABC) nên T ≡ O. Từ đó, ta có O là trung điểm AJ. Sử dụng phép
đối xứng tâm O, ta suy ra điều phải chứng minh.
2.2 Hình học 63

Bài 8

Cho tam giác ABC nhọn, BE, CF là các đường cao. M là trung điểm của
BC. N là giao điểm của AM và EF . Gọi X là hình chiếu của N lên BC.
Y, Ztheo thứ tự là hình chiếu của X trên AB, AC .Chứng minh rằng N là
trực tâm tam giác AY Z.

E
N

K
Z
F

B X M C

Lời giải. Xin giới thiệu cách giải của tác giả Nguyễn Minh Hà đăng trên tạp chí
Mathley.
Dễ chứng minh khẳng định bài toán đúng trong trường hợp tam giác ABC cân
tại A. Xét trường hợp tam giác ABC không cân, không mất tổng quát giả sử
AB > AC.
\ = BF
Gọi K là hình chiếu của M trên EF . Vì BEC \ C = 90o nên tứ giác BEF C
nội tiếp đường tròn đường kính BC. Vì M là trung điểm của BC và M K vuông
góc với EF nên K là trung điểm của EF .
64 HƯỚNG DẪN GIẢI

[ = ABC,
Cũng vì tứ giác BEF C nội tiếp nên AEF [ dẫn tới hai tam giác AEF
và ABC đồng dạng. Lưu ý rằng AM, AK tương ứng là trung tuyến của các tam
giác ABC, AEF nên các tam giác AKE, AM B đồng dạng. Điều này kéo theo
\ = AM
AKE \ B.

Mặt khác M
\ KN = M
\ XN = 90o nên tứ giác M N KX nội tiếp, kéo theo
\ = AM
XKE \ C.

Từ hai điều trên suy ra


\ + XKE
AKE \ = AM
\ B + AM
\ C = 180o ,

hay A, K, X thẳng hàng. Từ đó, chú ý hai tam giác AEF, ABC đồng dạng, ta
\ = KAE
thu được XAC \=N \ AF . Điều này dẫn tới
NE XB
= .
NF XC
Mặt khác, do CF, XY cùng vuông góc với AB nên CF k XY . Theo định lý
Thales, ta suy ra
XB YB
= .
XC YF
NE YB
Vậy = , từ đó theo định lý Thales đảo, Y N k BE. Mà BE vuông góc
NF YF
với AC nên Y N cũng vuông góc với AC. Chứng minh tương tự ta cũng có ZN
vuông góc với AB. Vì thế, N là trực tâm của tam giác AY Z.
Nhận xét.

1. Với trường hợp AB = AC, tứ giác M N KX sẽ suy biến thành đoạn thẳng.
2. Các tính chất của đường đối trung là ý tưởng xuyên suốt bài toán này.
NB
• Nếu AN là đường đối trung của tam giác ABC với N ∈ [BC] thì =
NC
AB 2
.
AC 2
• Giao điểm của hai tiếp tuyến tại B và tại C của đường tròn ngoại tiếp
tam giác ABC (nếu có) thuộc đường đối trung xuất phát từ A của tam
giác ABC
2.2 Hình học 65

Bài 9

Cho tam giác ABC có đường trung tuyến AM . Đường cao BE cắt đường
trung tuyến AM tại P . Lấy Q sao cho QE ⊥ AM, CQ ⊥ AB. Chứng minh
rằng AQ ⊥ CP .

L
K
E

P
Q
H

B M C

Lời giải. Gọi H = CQ ∩ BE và K thuộc BC thoả HK ⊥ CP . Ta có P là trực


tâm ∆HKC. Suy ra, P K ⊥ HC||AB. Gọi L thuộc AB sao cho P L||AC. Ta có
LB PB AK PB AC LB AK AC KC
= = . Lại có, = , do đó = = = . Mặt
LA PE KE PE AE LA KE AE KA
LB HQ KC HQ
khác, ∆P AB ∼ ∆EQC ⇒ = . Do đó, = ⇒ AQ k HK ⊥
LA HC KA HC
CP .

Bài 10

Cho tam giác ABC nội tiếp đường tròn (O). Tiếp tuyến tại B, C của đường
tròn (O) cắt nhau tại T . Gọi M, N lần lượt là các điểm thuộc tia BT, CT
sao cho BM = BC = CN . Đường thẳng M N cắt CA, AB theo thứ tự tại
E, F ; BE giao CT tại P, CF giao BT tại Q. Chứng minh rằng AP = AQ.

Lời giải. Gọi AD là phân giác của tam giác ABC. Do B, C đối xứng nhau qua
OT và BM = CN nên M, N đối xứng qua OT , suy ra BC ⊥ M N .
66 HƯỚNG DẪN GIẢI

D C
B

E
F M N
T

BM = 180o − ABC
Ta có F\ [ − CBM \ = 180o − ABC
[ − CAB [ = ACB,[ chú ý góc
[ = BF
đồng vị ABC \ M do đó 4ABC ∼ 4M F B. Từ đó ta chú ý F M ⊥ BC nên
theo định lý Thales
QC BC BM AC DC
= = = = ⇒ QD k BF.
QF FM FM AB DB

Tương tự P D k CE. Từ đó theo định lý Thales và tính chất đường phân giác ta

DQ DQ BF CE CD AB BC CD AB
= . . = . . = . = 1 ⇒ DP = DQ.
DP BF CE DP BC AC BD BD AC

\ = ABC + ACB
[
\ = ADB
Ta lại có ADQ \ + BDQ [ + ABC.
[ Tương tự, ADP\ =
2
BAC
[
+ ACB
[ + ABC. \ = ADP
[ Suy ra ADQ \. Kết hợp với DP = DQ, ta suy ra
2
4ADP = 4ADQ ⇒ AP = AQ, chính là đpcm.
Nhận xét. Bài toán trên được lấy của tác giả Trần Quang Hùng, và cấu hình bài
toán là rất cũ. Nếu áp dụng thành thục định lý sin cũng sẽ dẫn tới kết quả, nhưng
nó đòi hỏi tính toán thật nhanh và chính xác.
2.2 Hình học 67

Tổng quát của bài toán như sau:


Bài toán (Trần Quang Hùng). 4ABC nội tiếp (O).M, N thuộc cung BC sao cho
M N k BC và tia AM nằm giữa hai tia AB, AN . Trên tia BM, CN lấy các điểm
P, Q sao cho BP = BN = CM = CQ. P Q cắt AM, AN tại S, T . CS cắt BP tại
K và BT cắt CQ tại L. Chứng minh rằng AK = AL.

Lời giải của bài toán tổng quát trên có thể được tìm thấy trên fanpage Trường
đông Toán học miền Nam 2017.

Bài 11

Cho hai đường tròn (O1 ) và (O2 ) tiếp xúc ngoài tại M . Một đường thẳng
cắt (O1 ) tại A, B và tiếp xúc với (O2 ) tại E (B nằm giữa A và E). Đường
thẳng EM cắt (O1 ) tại điểm J khác M . C là một điểm thuộc cung M J
không chứa A, B của (O1 ) (C khác M và J). Kẻ tiếp tuyến CF với đường
tròn (O2 ) (F là tiếp điểm) sao cho các đoạn thẳng CF và M J không cắt
nhau. Gọi I là giao điểm của các đường thẳng CJ và EF , K là giao điểm
khác A của đường thẳng AI và đường tròn (O1 ). Chứng minh rằng:

1. Tứ giác M CF I là tứ giác nội tiếp và JA2 = JI 2 = JM.JE.

2. CI là phân giác ngoài tại C của tam giác ABC.

3. K là tâm đường tròn ngoại tiếp của tam giác BCI.

B E
A

O1 M
O2

K
I

J
68 HƯỚNG DẪN GIẢI

Lời giải.

1. Đường thẳng JO1 cắt (O1 ) tại D, ta có JD là phân giác trong góc AJB,
[
[ Dễ thấy JO1 k EO2 (bạn đọc tự chứng
CD là phân giác trong góc ACB.
minh) ⇒ JO\ 1 M = EO2 M ⇒ M BJ = M CI = M F I ⇒ Tứ giác M CIF
\ \ \ [
nội tiếp ⇒ M
\ EF = M\ FJ = M [IJ ⇒ IJ 2 = JM.JE. Ngoài ra, ABJ[ =
[ = BM
BAJ \ E ⇒ JBE
[ = BM \ J ⇒ KA2 = JB 2 = JM, JE.

2. Dễ thấy CD ⊥ IJ, mà CD là phân giác trong góc ACB


[ nên CI là phân
giác ngoài góc ACB.
[

3. Câu 1 cho ta JA = JB = JI ⇒ AIJ


d = IAJd = JBK.
[ Mà JBI d = JIB
d ⇒
KBI = KIB
[ ⇒ KB = KI. Cuối cùng, KCI [ = KAJ[ = KIC [ ⇒ KI =
KC. Vậy KB = KC = KI, và ta có K là tâm ngoại tiếp 4KCI.

Nhận xét. Bài toán này không gây quá nhiều khó khăn cho các bạn khi làm bài.
Bổ đề đơn giản sau đóng vai trò quyết định cho hai câu đầu, và gợi ý cho câu cuối:
Cho hai đường tròn (O1 ) và (O2 ) tiếp xúc ngoài tại M . Một tiếp tuyến chung ngoài
tại C ∈ (O2 ) của (O2 ) cắt (O1 ) tại hai điểm A, B phân biệt. Đường thẳng CM cắt
(O1 ) tại điểm thứ hai D. Khi đó DA = DB.

O1 M O2

D
2.2 Hình học 69

Bài 12

Cho tam giác ABC. Đường tròn (K) bất kỳ tiếp xúc đoạn thẳng AC, AB lần
lượt tại E, F . (K) cắt đoạn thẳng BC tại M, N sao cho N nằm giữa B và
M . F M giao EN tại I. Đường tròn ngoại tiếp các tam giác IF N và IEM
cắt nhau tại J khác I. Chứng minh rằng IJ đi qua A và KJ vuông góc IJ.

F
I

J K

B N M C

Lời giải (Trần Quang Hùng). Gọi J0 là hình chiếu của K lên AI, dễ thấy A, F ,
J0 , K, E thuộc đường tròn đường kính AK. Từ đó ta có góc nội tiếp và góc tạo
bởi tiếp tuyến và dây cung bằng nhau ∠0 F = ∠AEF = ∠F N I ⇒ Tứ giác F J 0 IN
nội tiếp. Tương tự tứ giác EJ0 IM nội tiếp từ đó J0 là điểm chung khác I của
đường tròn ngoại tiếp tam giác IF N và IEM , vậy j 0 ≡ J ⇒ A, I, J thẳng hàng
⇒ KJ⊥IJ.
70 HƯỚNG DẪN GIẢI

Bài 13

Cho (I) là đường tròn nội tiếp tam giác nhọn ABC, tiếp xúc BC, CA, AB ở
D, E, F . (O) là đường tròn ngoại tiếp tam giác ABC. Lấy D0 đối xứng với
D qua EF .

1. Chứng minh rằng AD0 , BC, OI đồng quy.

2. Gọi H, J lần lượt là trực tâm các tam giác ABC và AEF . Gọi R là
giao HJ và ID, T là giao D0 J và OI. Chứng minh rằng D0 , R, T, I
đồng viên.

R
D0

J E

F O
T
I

C
B D
S

Lời giải.
1. • Gọi A0 , B 0 , C 0 lần lượt là điểm chính giữa các cung nhỏ BC, CA, AB
thì 4A0 B 0 C 0 có OI là đường thẳng Euler và có phép vị tự biến
2.2 Hình học 71

4A0 B 0 C 0 thành 4DEF . Do đó OI song song hoặc trùng với đường


thẳng Euler của 4DEF . Mặt khác I là tâm (DEF ) nên OI là đường
thẳng Euler của 4DEF .

• Ta đưa hình vẽ về dạng như sau:

I
K

E F
K0

D0

Với S là giao BC và AD0 , K là trực tâm của 4DEF , K 0 là giao DD0


với (I) và V là giao AI với SD.
D0 K IA
Ta sẽ chứng minh = . Khi đó theo Thales ta sẽ có K thuộc
DK IV
IS.
D0 K DK 0
Do tính đối xứng ta có = .
DK DK
[ = α và bán kính của (I) là R. Ta có IV = R .
Đặt IDK
cos α
R IA cos α
Đồng thời IA = . Suy ra = .
cos EDF
\ IV cos EDF
\
72 HƯỚNG DẪN GIẢI

\ và DK 0 = 2R cos α (Suy từ định lý sin


Dễ thấy DK = 2R cos EDF
0
DK cos α IA
cho (I)). Từ đó = = . (đpcm)
DK cos EDF
\ IV

2. Ta đưa hình vẽ về dạng như sau:

R
D0

K0
J E
N
M

F O
L T
I
K
H

C
B D
S

Ta thấy I và J đối xứng qua EF nên D0 J và DI giao nhau ở N trên EF .


Gọi L là giao DJ và OI thì L là tâm Euler của 4DEF và DKJI là hình
bình hành.
Gọi M là chân đường cao từ D của 4DEF . Ta có kết quả quen thuộc
M, H, J thẳng hàng.
[ = IJK
Ta có HAJ [ = JKM [ =M
\ và AHJ \ JK.
Từ đó 4AHJ ∼ 4KJM nên
AH KJ ID
= = (2.19)
AJ KM KM
2.2 Hình học 73

Ta có
IL 1 IK 1 JD0 ID
= . = . = . (2.20)
IT 2 IT 2 JT 2JT
JN KD DK DK
Theo định lý Thales ta có 0
= 0
= 0
nên IN = .JK
JD KD DK DK 0
2ID.KM
Do đó JK − IN = .
DK 0
Lại có
TJ JK TJ JK DK 0
= ⇒ = =
NT IN IN JK − IN 2KM

IN.DK DK.JK ID.DK


⇒ 2JT = = = .
KM KM KM
kết hợp với (2.19) và DK = IJ ta có
AH.IJ
2JT = .
AJ
Lại dùng Thales cho 4AJH có IR k AH thì thấy 2JT = IR.
IL ID
Kết hợp với (2.40) ta có = nên DJ k RT .
IT IR
Từ đó có
NJ NR NI NR
= ⇒ = ⇒ N I.N R = N T.N D0 .
NT ND NT N D0
Vì vậy D0 , R, I, T đồng viên.

Bài 14

Cho tam giác ABC, P là điểm bất kỳ. A1 là hình chiếu của P lên BC.A2 là
trung điểm AA1 .A2 P cắt BC tại A3 .A4 đối xứng A1 qua A3 . Chứng minh
rằng P A4 luôn đi qua một điểm cố định.

Lời giải. Chúng ta phát biểu và làm bài toán tổng quát hơn.
Bài toán. Cho tam giác ABC, P là điểm bất kỳ. A1 là hình chiếu song song của
P theo phương l cố định lên BC.A2 là trung điểm AA1 .A2 P cắt BC tại A3 .A4 đối
xứng A1 qua A3 . Chứng minh rằng P A4 luôn đi qua một điểm cố định.
74 HƯỚNG DẪN GIẢI

A2
A5
P

B A4 A3 A1 C

Gọi L là hình chiếu song song phương l của A lên BC. Gọi A5 là trung điểm
AL, ta sẽ chứng minh rằng A4 , P, A5 thẳng hàng thật vậy, từ liên hệ tỷ số đơn
và tỷ số kép dễ thấy

A1 (A4 A2 A5 P ) = (LAA5 ) = −1( do P A1 k AL và A5 là trung điểm AL.

A2 (A4 A1 A3 A5 ) = (A4 A1 A3 ) = −1( do A2 A5 k A1 A4 và A3 là trung điểm A1 A4 .


Từ đó A1 (A4 A2 A5 P ) = A2 (A4 A1 A3 A5 ) nên A4 , A5 , P thằng hàng. Ta có điều
phải chứng minh.

Nhận xét. Ngoài lời giải tỷ số kép, có một số cách tiếp cận khác chỉ sử dụng công
cụ hình học lớp 8, tuy vậy với cách làm bằng tỷ số kép, các bạn hoàn toàn có thể
tổng quát bài toán.

Bài 15

Cho năm điểm A, B, C, D và E cùng nằm trên một đường tròn. Gọi M , N ,
P và Q lần lượt là hình chiếu vuông góc của E xuống các đường thẳng AB,
BC, CD và DA. Gọi hình chiếu vuông góc của E xuống các đường thẳng
M N , N P , P Q, QM lần lượt là R, S, T và U cùng nằm trên một đường
thẳng.

Lời giải. Ta dễ dàng chứng minh được bổ đề sau:


2.2 Hình học 75

Bổ đề 3. Cho tam giác ABC nội tiếp đường tròn (O) và M là một điểm bất kì
trên đường tròn đó. Gọi chân các đường vuông góc kẻ từ M lên ba đường thẳng
BC, CA, AB lần lượt là D, F , E. Khi đó D, F , E thẳng hàng (đường thẳng chứa
D, F , E là đường thẳng Simson ứng với điểm M của tam giác ABC).

A
I

C
M
R

Q
U
J
T

E
D

Quay lại bài toán. Gọi I, J lần lượt là hình chiếu vuông góc của E xuống các
đường thẳng AC, BD. Theo bổ đề trên ta thấy:
• M, N, I thẳng hàng (đường thẳng Simson ứng với điểm E của tam giác
ABC). Do đó R thuộc đường thẳng M I.
• N, P, J thẳng hàng (đường thẳng Simson ứng với điểm E của tam giác
BCD). Do đó S thuộc đường thẳng N P .
Mặt khác, ta thấy M, I, Q, E cùng nằm trên đường tròn đường kính AE, nên
từ kết quả trên ta có R, U, T thẳng hàng (đường thẳng Simson ứng với điểm
E của tam giác M IQ). Lại có I, N, P, E cùng nằm trên đường tròn đường kính
CE, nên R, S, T thẳng hàng (đường thẳng Simson ứng với điểm E của tam giác
N P I. Vậy R, S, T, U thẳng hàng.
76 HƯỚNG DẪN GIẢI

2.3 Phương trình hàm - Dãy số


Bài 1

Tìm các hàm f : R → R thoả mãn:

f (x2 ) + f (xy) = f (x)f (y) + yf (x) + xf (x + y), ∀ xy ∈ R (2.21)

Lời giải. Bài này là một bài khá là quen thuộc và đã xuất hiện khá nhiều các
giải trên mạng. Bài viết sẽ trình bày 2 cách giải quen thuộc nhất. Nhưng trức
hết giai đoạn đầu là như nhau:
Thay x = y = 0 vào đề ta có:

2 f (0) = 2
2f (0) = (f (0)) ⇔
f (0) = 0

Với f (0) = 2 ta thay x = 0 vào đề ta có:

4 = 2f (y) + 2y ⇔ f (y) = 2 − y, ∀y ∈ R

Thử lại ta có:



VT = 2 − x2 + 2 − xy = 4 − x2 − xy
VP = (2 − x)(2 − y) + y(2 − x) + x(2 − x − y) = 4 − x2 − xy

Vậy ta có một nghiệm là f (x) = 2 − x, ∀ x ∈ R.


Với f (0) = 0 thì có ba cách xử lí:

• Cách 1: Nghiệm lai.


Thay y = −x ta có:

f (x2 ) + f (−x2 ) = f (x)f (−x) − xf (x) (2.22)

Thay x = −y ta có:

f (y 2 ) + f (−y 2 ) = f (y)f (−y) + yf (−y) (2.23)

Từ (2.22) và (2.23) ta có:

f (−x) = −f (x), ∀x ∈ R
2.3 Phương trình hàm - Dãy số 77

Thay ngược lại (2.22) ta có:

0 = −(f (x))2 − xf (x) ⇔ f (x)(f (x) + x) = 0, ∀x ∈ R

Điều trên dẫn đến

f (x) = 0, ∀x ∈ A (2.24)
f (x) = −x, ∀x ∈ B (2.25)

với A, B ⊂ R và A ∪ B = R.
Ta sẽ đi chứng minh A = B = R. Sử dụng phản chứng, tức tồn tại x0 6=
y0 ∈ R, x0 y0 6= 0 sao cho:

f (x0 ) = 0
f (y0 ) = −y0

Thay y = 0 vào đề ta có:

f (x2 ) = xf (x), ∀x ∈ R (2.26)

Lần lượt thay x = x0 và x = y0 vào (2.26) ta có được kết quả f (x20 ) = 0 và


f (y02 ) = −y02 .
Thay x = x0 , y = y0 và đề, sau đó đổi thứ tự và thu gọn ta có được:

y02 = x0 y0 + (x0 − y0 )f (x0 + y0 )

Nếu f (x0 + y0 ) = 0 thì ta có y02 = x0 y0 dẫn đến x0 = y0 hoặc y0 = 0, vô lý.


Nếu f (x0 + y0 ) = −x0 − y0 thì ta có: y02 = x0 y0 + y02 − x20 dẫn đến x0 = y0
hoặc x0 = 0, vô lý.
Vậy A = B = R hay ta có hai nghiệm nữa là f (x) = 0, ∀x ∈ R và
f (x) = −x, ∀x ∈ R.

• Cách 2: Ta vẫn có thể tính được f là hàm lẻ. Khi đó thay y = 0 vào (2.21)
ta có:
f (x2 ) = xf (x), ∀x ∈ R (2.27)
Thay đổi vai trò của x và y trong (2.21) ta có:

f (y 2 ) + f (xy) = f (x)f (y) + xf (y) + yf (x + y), ∀x, y ∈ R (2.28)


78 HƯỚNG DẪN GIẢI

Lấy (2.21) trừ (2.28) vế theo vế ta có:

f (x2 ) − f (y 2 ) = yf (x) − xf (y) + (x − y)f (x + y), ∀x, y ∈ R

Thay (2.27) vào phương trình trên ta có:

(x − y)f (x) + (x − y)f (y) = (x − y)f (x + y), ∀x, y ∈ R

hay
f (x) + f (y) = f (x + y), ∀x 6= y ∈ R (2.29)

Ngoài ra, thay y = −x vào (2.21) ta có:

(f (x))2 = −xf (x), ∀x ∈ R

và thay y = x vào (2.21), sử dụng đẳng thức trên ta có:

xf (x) + xf (x) = −xf (x) + xf (x) + xf (2x) ⇔ f (2x) = 2f (x), ∀x 6= 0 ∈ R

Mà f (x) = 0 nên ta có

f (2x) = 2f (x), ∀x, y ∈ R (2.30)

Từ (2.29) và (2.30) ta có:

f (x) + f (y) = f (x + y), ∀ x, y ∈ R (2.31)

Đến đây ta sẽ sử dụng phương pháp tính bằng hai cách f ((x + 1)2 ) như
sau:

f ((x+1)2 ) = (x+1)f (x+1) = (x+1)(f (x)+f (1)) = xf (x)+f (x)+xf (1)+f (1)

f ((x + 1)2 ) = f (x2 + 2x + 1) = f (x2 ) + 2f (x) + f (1) = xf (x) + 2f (x) + f (1)


Từ hai đẳng thức trên ta suy ra được f (x) = xf (1), ∀x ∈ R.
Từ đây dễ dàng tìm được f (1) = 0 hoặc f (1) = −1 hay ta có 2 nghiệm là
f (x) = 0, ∀x inR hay f (x) = −x, ∀x ∈ R.
2.3 Phương trình hàm - Dãy số 79

Nhận xét. Đây là một bài toán khá dễ chịu, cả về ý tưởng lẫn hướng giải quyết.
Với các giải thứ 2 thì sau khi đã nhận biết được sự công tính và tính chất f (x2 ) =
xf (x) thì một các tự nhiên, ta sẽ suy nghĩ đến đại lượng f ((x + 1)2 ) vì giải pháp
tính bằng 2 cách sẽ hữu hiệu với những gì đã có.
Với cách đầu tiên, rõ ràng là khá nguy hiểm khi có nhiều bạn sẽ suy ra nghiệm
ngay từ phương trình f (x)(f (x) + x) = 0, ∀x ∈ R mà quên đi việc kiểm tra
nghiệm ngoại lai. Sau đây sẽ là một số bài toán tương tự cho cách 1:

1. (MOSP 2002) Tìm tất cả các hàm f : R → R thoả mãn:

f (f (x) + y) = f (x2 − y) + 4f (x)y, ∀x, y ∈ R.

2. (Balkan 2000) Tìm tất cả các hàm f : R → R thoả mãn:

f (xf (x) + f (y)) = (f (x))2 + y, ∀x, y ∈ R.

3. Tìm tất cả các hàm f : R → R thoả mãn:

(f (x + y))2 = f (x)f (x + 2y) + yf (y), ∀x, y ∈ R.

4. Tìm tất cả các hàm f : R → R thoả mãn:

f (x2 + f (y)) = xf (x) + y, ∀x, y ∈ R.

Bài 2

Cho hàm số :f : N → N thoả

f (f (n)) + f (n) = 6n + 4 (2.32)

1. Tính f (2017)

2. Tìm tất cả các hàm f thoả mãn.

Lời giải. Giả sử hàm f thõa mãn yêu cầu bài toán. Với n là số tự nhiên bất kì,ta
xét dãy {xn } như sau: x0 = n và xn+1 = f (xn ).
Thay n bởi xn ta được:

xn+2 + xn+1 − 6xn − 4 = 0


80 HƯỚNG DẪN GIẢI

suy ra :xn = a.(−3)n + b.2n − 1.


Mà do   n 
n −3 1
xn = 2 b+a − n .
2 2
Xét a < 0 thì khi chọn n chẵn và đủ lớn ta có xn tiến về âm vô cùng, vô lí. Tương
tự xét a > 0 và chọn n lẻ và đủ lớn cũng có được xn tiến về âm vô cùng ,vô lí.
Vậy a = 0. Khi đó xn = b.2n − 1 .
Thay n = 0, 1 ta có được f (n) = 2n + 1. Thử lại đúng. Vậy f (n) = 2n + 1 và
f (2017) = 4035.

Nhận xét. Một hàm số có tập xác định trên N luôn được biểu diễn như một dãy
số vì xn = f (n) với f : N → R. Phần việc tìm số hạng tổng quát và tính giá trị rất
quen thuộc với nhưng học sinh chuyên toán.
Bài này tất cả các đoàn đều làm đúng và được trọn vẹn điểm.

Bài 3

Cho dãy số
(
x1 = x2 = 97
(xn ) : q  .
xn+2 = xn+1 xn + x2n+1 − 1 (x2n − 1) ∀n ≥ 1

1. Chứng minh 2 + 2xn là số chính phương.



2. Chứng minh 2 + 2 + 2xn là số chính phương,

Hướng dẫn. Ta chứng minh bằng quy nạp:


√ k 1
• 2+ 3 + √ k ∈ Z ∀k ∈ N
2+ 3
!
1 √ 4Fn 1
• xn = 2+ 3 + √ 4Fn ∀n ∈ N, trong đó Fn là dãy Fi-
2 2+ 3
bonacci. Khi đó ta được
!2
 √ 2(Fn ) 1
2 + 2xn = 2+ 3 + √ 2(Fn ) .
2+ 3
2.3 Phương trình hàm - Dãy số 81
!2
√  √ (Fn ) 1
2 + 2 + 2xn = 2+ 3 + √ (Fn ) .
2+ 3

Bài 4

Xét dãy số nguyên {an }∞


n=1 thoả mãn:

−1 a2 1
a1 = 2; a2 = 7; < an+2 − n+1 ≤ . (2.33)
2 an 2

Chứng minh rằng với mọi n > 1,ta có an là số lẻ.

Lời giải. Hệ thức đã cho ở đề bài là đặc trưng của dãy truy hồi cấp 2. Ta có giữa
a2 1 a2 1
hai số n+2 − và n+2 + có duy nhất 1 số nguyên nên dãy đã cho xác định
an 2 an 2
duy nhất.
Tính một số giá trị đầu:a1 = 2, a2 = 7, a3 = 25, a4 = 89. Nên ta nghĩ đến dãy số
xác định như sau:

a1 = 2, a2 = 7, và an+2 = 3an+1 + 2an . (2.34)

Ta sẽ chứng minh dãy số này thoả mãn đề bài. Bằng quy nạp ta sẽ chứng minh:
a2n+1 − an an+2 = (−1)n .2n−1 .

• Với n = 1,đẳng thức hiển nhiên đúng.

• Giả sử đẳng thức đúng đến k − 1.Hay a2k − ak−1 ak+1 = (−1)k−1 .2k−2 . Ta
phải chứng minh:a2k+1 − ak ak+2 = (−1)k .2k−1 hay

a2k+1 − ak ak+2 = −2(a2k − ak−1 ak+1 )


⇔ a2k+1 − 2ak+1 ak−1 = ak ak+2 − 2a2k
⇔ ak+1 (ak+1 − 2ak−1 ) = ak (ak+2 − 2ak )
ak+1 − 2ak−1 ak+2 − 2ak
⇔ =
ak ak+1

Hiển nhiên đúng do các tỉ số trên đều bằng 3 theo công thức truy hồi.
Đẳng thức được chứng minh.
82 HƯỚNG DẪN GIẢI

Trở lại bài toán. Từ đẳng thức ta có


n−1
a2n+1 (−1)n .2
an+2 − = (2.35)
an an
Ta phải chứng minh

(−1)n .2n−1 1
≤ , hay an ≥ 2n với n > 1 (2.36)

an 2

• Với n = 2,hiển nhiên.

• Giả sử an ≥ 2n ,do an+1 = 3an + 2an−1 , nên an+1 > 2n+1 ,đúng.

Vậy ta có (2.36),hay dãy số này thoả mãn bài toán, từ tính duy nhất ta có dãy
vừa lập chính là dãy trong bài toán.
Giờ ta có:
an+2 = 3an+1 + 2an ⇒ an+2 ≡ an+1 (mod 2) (2.37)
Mà a2 = 7 nên ta có an là số lẻ với mọi n > 1, điều phải chứng minh.

Bài 5

Tìm f : R → R thoả mãn:

f (x3 + f (y)) = y + f 3 (x) (2.38)

với mọi x, y thực.

Lời giải. Ta sẽ chứng minh f (0) = 0. Từ đề bài dễ thấy f là một song ánh. Do
vậy sẽ tồn tại duy nhất số thực b sao cho f (b) = 0. Đặt f (0) = a.
Trong (2.36) cho x = y = 0 :
f (a) = a3 .
Trong (2.36) cho y = a, x = b :

f (b3 + a3 ) = a

Suy ra f (b3 + a3 ) = a = f (0), kéo theo b3 + a3 = 0.


Trong (1) cho x = 0, y = b :

f (0) = b + a3 ⇔ a3 + b = a
2.3 Phương trình hàm - Dãy số 83

Trong (2.36) cho x = 0, y = a :

f (f (a)) = a + f 3 (0) ⇔ f (a3 ) = a + a3

Trong (2.36) tiếp tục cho x = a, y = b :

f (a3 ) = b + f 3 (a) = b + a9

Từ đó suy ra b + a9 = a + a3 .
Giải hệ gồm ba phương trình hai ẩn sau :

b + a9 = a + a3
a3 + b 3 = 0 ⇔ a = b = 0
a3 + b = a

Vậy tóm lại là f (0) = 0. Từ đó dễ dàng suy ra được :

f (f (y)) = y, ∀y ∈ R (2.39)

f (x3 ) = f 3 (x), ∀x ∈ R
Kết hợp hai điều trên với phương trình hàm đã cho ta suy ra được :

f (x + y) = f (x) + f (y), ∀x ∈ R

Khi đó sẽ có :
f (n) = f (1).n, ∀n ∈ Z
Thay vào (2.39) ta có f (1) = ±1.
Nếu f (1) = 1 ta có

f ((x + 1)3 ) = [f (x) + 1]3 = f 3 (x) + 3f 2 (x) + 3f (x) + 1, ∀x ∈ R

Cũng có :

f ((x + 1)3 ) = f (x3 + 3x2 + 3x + 1) = f 3 (x) + 3f (x2 ) + 3f (x) + 1, ∀x ∈ R

So sánh hai kết quả trên thì được :

f (x2 ) = f 2 (x), ∀x ∈ R (2.40)

Điều này cho ta tính chất :

f (x) ≥ 0, ∀x ≥ 0.
84 HƯỚNG DẪN GIẢI

Kết hợp với tính cộng tính của f ta có được kết quả f (x) = f (1)x = x.
Tương tự với f (1) = −1 ta có:

f (x2 ) = −f 2 (x)

hay f (x) ≤ 0 nếu x ≥ 0, từ đó sẽ có f giảm. Khi đó f (x) = f (1)x = −x.


Vậy ta có 2 nghiệm hàm là f (x) = x và f (x) = −x với mọi x ∈ R.

Nhận xét. Đây là một bài toán không mới, nhưng các kỹ thuật của bài toán này
rất hay được sử dụng. Bài toán có thể suy ra được rất nhiều giả thuyết cho f như
f song ánh (f (f (x)) = x, ∀x ∈ R), f cộng tính và f (x3 ) = (f (x))3 ). Từ những
điều đó, theo một các tự nhiên như cách 2 của A1 thì ta nên xét f ((x + 13 ) bằng
hai cách để tìm được những mối liên hệ còn lại (ở đây là tính đơn điệu của hàm
số).

Bài 6

Tìm tất cả các hàm f : R → R thảo mãn:

f (xf (x + y)) = f (yf (x)) + x2 , ∀x, y ∈ R (2.41)

Lời giải. Đầu tiên ta sẽ chứng minh f (0) = 0 ⇔ x = 0. Thật vậy nếu f (0) 6= 0
x
thì thay x = 0 và y = thì ta có
f (0)

f (0) = f (x) , ∀x ∈ R.

Tức là f (x) là hàm hằng nhưng dễ thấy hàm này không thoả, vậy f (0) = 0.
Ngoài ra nếu f (x) = 0 thì thay y = 0 ta có f (0) = f (0) + x2 ⇔ x = 0 ta được
điều phải chứng minh.
Thay y = 0 ta có:
f (xf (x)) = x2 .
Tiếp theo ta sẽ chứng minh f (x) đơn ánh. Thật vậy nếu với hai số a và b bất kì
khác 0 thoả mãn f (a) = f (b) thì thay x = a, y = b − a ta được:

f (af (b)) = f ((b − a) f (a)) + a2 = f ((b − a) f (a)) + f (af (a)) .

Nên
f ((b − a) f (a)) = f (af (b)) − f (af (a)) .
2.3 Phương trình hàm - Dãy số 85

Vì f (a) = f (b) nên

f ((b − a) f (a)) = 0 ⇔ (b − a) f (a) = 0 ⇔ a = b.

Vậy f đơn ánh. Ta có đánh giá:

f (xf (x)) = x2 = (−x)2 = f (−xf (−x))

Vì f là đơn ánh nên xf (x) = −xf (−x) hay f (x) = −f (−x).


Vì f (xf (x)) = x2 nên R+ ∪ {0} ⊂ f (R) và vì f là hàm lẻ nên f (−xf (x)) = −x2
nên R− ⊆ f (R). Điều này chứng tỏ f (R) = R (vì ta đã sẵn có f (R) ⊂ R do f
đơn ánh).
Tiếp theo, vì f là một toàn ánh nên tồn tại u ∈ R sao cho f (u) = 1. Khi đó vì
f (uf (u)) = u2 ⇒ f (u) = u2 ⇒ u2 = 1 ⇒ u = ±1 nên u chỉ có hai giá trị là ±1.

• Nếu u = 1, tức là f (1) = 1 thì khi đó thay x = 1 và y = x − 1 ta có:

f (f (x)) = f (x − 1) + 1

thay x = 1, y = −x − 1 ta có:

f (f (−x)) = f (−x − 1) + 1 ⇒ −f (f (x)) − f (x + 1) + 1

Từ đó ta có:

f (x + 1) = f (x − 1) + 2 ⇒ f (x + 2) = f (x) + 2 ⇒ f (x + 4) = f (x) + 4

Mặt khác thay x = 2 ta có:

f (2f (y + 2)) = f (yf (2)) + 4 = f (2y + 4)

⇒ 2f (y + 2) = 2y + 4 ⇒ f (y) = y.

• Tương tự với u = −1 ta được f (x) = −x.

Thử lại thấy cả hai hàm đều thoả.


86 HƯỚNG DẪN GIẢI

Bài 7

Tìm tất cả hàm số f : R → R thoả:

f (f (y) + x2 + 1) + 2x = y + (f (x + 1))2 , ∀x, y ∈ R (2.42)

Lời giải. Thay x = 0 vào ta có:

f (f (y) + 1) = y + f (1)2 (2.43)

Từ đây ta suy ra f song ánh. Do đó tồn tại k thoả mãn f (k) = 0.


Thay x = 0, y = k vào ta được

f (1) = k + f (1)2 ⇒ k = f (1) − f (1)2 = a − a2

với a = f (1).
f (f (y) + 2) = 2 + y + f (0)2 (2.44)
Từ (2.43) và (2.44) dẫn đến

f (f (y) + 2) − f (f (y) + 1) = 2 + f (0)2 − f (1).

Vì tính song ánh của f nên từ phương trình này ta suy ra f (x + 1) − f (x) =
2 + f (0)2 − f (1)2 với mọi x ∈ R. Khi đó

f (1) − f (0) = 2 + f (0)2 − f (1)2 . (2.45)

Ta cũng có f (2) − f (1) = f (1) − f (0) hay f (2) = 2f (1) − f (0).


Thay x bởi −x ta có:

f (f (y) + x2 + 1) − 2x = y + f (1 − x)2 (2.46)

Từ (2.46) và phương trình đề ta suy ra

f (1 − x)2 + 2x = f (x + 1)2 − 2x. (2.47)

Thay x = 1 vào phương trình (2.47) ta được f (0)2 +4 = f (2)2 = [2f (1) − f (0)]2 .
Do đó f (0)f (1) = f (1)2 − 1. Nhân cả hai vế của (2.45) với f (1)2 = a2 ta suy ra

a3 − a(a2 − 1) = 2a2 + (a2 − 1)2 − a4 ⇔ a = f (1) = 1.

Ta suy ra k = f (1) − f (1)2 = 0 hay f (0) = 0. Khi đó f (x + 1) − f (x) = 1.


2.3 Phương trình hàm - Dãy số 87

Thay x bởi f (x) ta được f (f (x) + 1) = f (f (x)) + 1. Mặt khác, từ (2.43) ta có


f (f (x) + 1) = x + 1 nên ta suy ra f (f (x)) = x.
Thay x = 1 ta có: f (y 2 + 2) = f (y + 1)2 + 1 − 2y. Vì tính toàn ánh của f nên tồn
tại h thoả mãn f (h) = −2x. Ta suy ra

x2 + 2 = f (f (x2 + 2)) = f f (x + 1)2 + 1 + f (h)




= h − 2f (x + 1) + f (f (x + 1) + 1)2 = h − 2 (f (x) + 1) + (x + 2)2 .


Từ đây ta tìm được h = 2f (x) − 4x hay

f (2f (x) − 4x) = −2x (2.48)

P (2x + 1, x) ⇒ f f (2x + 1) + x2 + 1 = f (x + 1)2 + 1




Do đó

f (2x + 1) + x2 + 1 = f f f (2x + 1) + x2 + 1


= f f (x + 1)2 + 1 + f (0)


= f (f (x + 1) + 1)2 − 2f (x + 1) = (x + 2)2 − 2f (x) − 2

Ta tìm được f (2x) + 2f (x) = 4x hay

2f (x) − 4x = −f (2x). (2.49)

Từ (2.48) và (2.49) ta suy ra f (−f (2x)) = −2x = f (f (−2x)). Vì f là đơn ánh


nên
f (−2x) = −f (2x)
hay f (−x) = −f (x)∀x ∈ R. Áp dụng các tính chất của hàm vừa được chứng
minh ở trên, ta sẽ có

(2.47) ⇔ f (x−1)2 +4x = f (x+1)2 ⇔ [f (x) − 1]2 +4x = [f (x) + 1]2 ⇔ f (x) = x.

Vậy f (x) = x, ∀x ∈ R.
88 HƯỚNG DẪN GIẢI

Bài 8

Tìm tất cả các hàm f (x) : [1; +∞) → [1; +∞) thoả

x ≤ f (x) ≤ 2x + 2
(2.50)
xf (x + 1) = (f (x))2 − 1

Lời giải. Ta có :

x + 1 ≤ f (x + 1) ≤ 2x + 4 ∀x ∈ [1, +∞)

⇒ x2 + x ≤ xf (x + 1) = f 2 (x) − 1 ≤ 2x2 + 4x, ∀x ∈ [1, +∞)


 2
1 3
⇒ x+ + ≤ f 2 (x) ≤ 2(x + 1)2 − 2, ∀x ∈ [1, +∞)
4 4
1 √
⇒ x + ≤ f (x) ≤ 2(x + 1), ∀x ∈ [1, +∞)
4
x+1 √
⇒ √ ≤ f (x) ≤ 2(x + 1), ∀x ∈ [1, +∞)
2
Thực hiện liên tiếp quá trình này, ta được :

x+1 √
2n
2
√n ≤ f (x) ≤ 2(x + 1), ∀x ∈ [1, +∞)
2

Với mỗi x thuộc nửa khoảng từ 1 tới dương vô cực, ta cho n → +∞ ta được :

x + 1 ≤ f (x) ≤ x + 1, ∀x ∈ [1, +∞)

Như vậy ta phải có :


f (x) = x + 1, ∀x ∈ [1, +∞)
Thử lại thấy thoả. Đó là đáp số duy nhất của bài toán.

Bài 9

Tìm tất cả các hàm f : R → R thoả mãn:

f (yf (x) − x) = f (x)f (y) + 2x, ∀x, y ∈ R (2.51)


2.3 Phương trình hàm - Dãy số 89

Lời giải. Thay x = y = 0 vào (2.51) ta được f (0) = (f (0))2 nên ta f (0) = 0
hoặc f (0) = 1.

• Trường hợp f (0) = 0. Thay y = 0 vào (2.51), ta được f (x) = 2x, suy ra
f (x) = 2x với mọi x. Thử lại, ta thấy thoả mãn.

• Trường hợp f (0) = 1. Thay y = 0 vào (2.51), ta được

f (−x) = f (x) + 2x, ∀x ∈ R (2.52)

Thay y bởi −y vào (2.51) và sử dụng (2.52), ta được:

f (yf (x) + x) + 2 [yf (x) + x] = f (−yf (x) − x)

= f (x)f (−y) + 2x = f (x) [f (y) + 2y] + 2x


từ đó suy ra
f (yf (x) + x) = f (x)f (y)
Do đó phương trình (2.51) có thể được viết lại thành:

f (yf (x) − x) = f (yf (x) + x) + 2x, ∀x, y ∈ R (2.53)

x
Với mọi x khác 0 mà f (x) 6= 0, thay y = vào (2.53), ta được
f (x)

f (2x) = 1 − 2x (2.54)

Với mọi x khác 0 mà f (x) = 0 ta có f (−x) = f (x) + 2x = 2x 6= 0 nên theo


trên thì f (−2x) = 1 + 2x. Suy ra

f (2x) = f (−2x) − 4x = 1 − 2x (2.55)

Từ (2.54) và (2.55), ta suy ra với mọi x 6= 0 thì f (x) = 1 − 2x. Mà f (0) = 1


nên ta có f (x) = 1 − x, ∀x ∈ R. Thử lại, ta thấy thoả mãn.
Tóm lại, có hai hàm số thoả mãn yêu cầu đề bài là f (x) = −2x và f (x) =
1 − x.
90 HƯỚNG DẪN GIẢI

Bài 10

Tìm f: R+ → R+ thoả mãn:

f (x + y) + f (x).f (y) = f (xy) + f (x) + f (y) (2.56)

Lời giải. Ta thêm biến z ∈ R và thay y bởi y + z ta có:

f (x + y + z) = f (x) + f (y + z) + f (xy + xz) − f (x)f (y + z)

= f (x) + f (y) + f (z) + f (xy) + f (yz) + f (zx) + f (x)f (y)f (z)


−f (x)f (y) − f (y)f (z) − f (z)f (x)
+f (x2 yz) − f (xy)f (xz) − f (x)f (yz)
với mọi x, y, z ∈ R+ .
Tương tự thay x = y còn y = x + z ta có

f (x2 yz) − f (xy)f (xz) − f (x)f (yz) = f (xy 2 z) − f (xy)f (yz) − f (y)f (xz) (2.57)

với mọi x, y, z ∈ R+ .
Thay y = 1 vào (2.57) ta có:

f (x2 z) − f (x)f (xz) − f (x)f (z) = f (xz) − f (x)f (z) − f (1)f (xz)

với mọi x, z ∈ R+ . Điều này có nghĩa là f (x2 z) = (1 − f (1))f (xz) + f (x)f (xz)
với mọi x, z ∈ R+ , vì thế

f (xy) = (1 − f (1))f (y) + f (x)f (y)

với mọi x, y ∈ R+ .
Từ f (xy) = (1 − f (1))f (y) + f (x)f (y) = (1 − f (1))f (x) + f (x)f (y) với mọi
x, y ∈ R+ ta được f (x) = c hoặc f (1) = 1.
Nếu f (x) = c, ta có c + c2 = 3c, vì thế f (x) = 2 với mọi x ∈ R+ .
Ngược lại, f (1) = 1, vì thế f (xy) = f (x)f (y) với mọi x, y ∈ R+ , vì thế f (x + y) =
f (x) + f (y) với mọi x, y ∈ R+ ,
Từ f : R+ → R+ , vì thế ta có f (x) = cx với hằng số c. Thay lại ta có c = 1. Khi
đó đáp án là f (x) = 2 với mọi x ∈ R+ và f (x) = x với mọi x ∈ R+ .
2.3 Phương trình hàm - Dãy số 91

Bài 11

Tìm các hàm f : Q2 → Q thoả mãn

f (x, y) + f (y, z) + f (z, x) = f (0, x + y + z) (2.58)

với mọi x, y, z ∈ Q.

Lời giải. (Đáp án tham khảo của Trần Quốc Nhật Hân từ Diễn đàn Toán học
VMF).

f : Q2 → Q : f (x, y) + f (y, z) + f (z, x) = f (0, x + y + z) (2.59)

Thay x = y = z = 0 từ (2.58) ta có

f (0, 0) = 0

Thay y = z = 0 từ (2.58) ta có

f (x, 0) + f (0, x) = f (0, x) ⇒ f (x, 0) = 0∀x (2.60)

Thay z = 0 từ (2.58) và (2.60) ta có:

f (x, y) + f (0, x) = f (0, x + y) (2.61)

Theo (2.61) ta có

f (0, x + (y + z)) = f (x, y + z) + f (0, x)

Tiếp tục từ (2.61) ta có:

f (0, (x + y) + z) = f (x + y, z) + f (0, x + y)

= f (x + y, z) + f (x, y) + f (0, x) (2.62)


Khi đó:
f (x, y + z) = f (x + y, z) + f (x, y) (2.63)
Thay z = y vào (2.63) ta được:

f (x, 2y) = f (x + y, y) + f (x, y) (2.64)


92 HƯỚNG DẪN GIẢI

Thay z = 2y vào (2.63) ta được:

f (x, 3y) = f (x + y, 2y) + f (x, y) (2.65)


= f (x + 2y, y) + f (x, y) + f (x, y) (2.66)
= f (x + 2y, y) + 2f (x, y) (2.67)

⇒ f (x, (n + 1) y) = f (x + ny, y) + nf (x, y) ∀n ∈ N (2.68)


Sử dụng (2.63) ta có:

f (x, 4y) = f (x + 3y, y) + f (x, 3y) (2.69)


= f (x + 3y, y) + f (x + 2y, y) + 2f (x, y) (2.70)

Ngoài ra cũng từ (2.63) ta có:

f (x, 4y) = f (x + 2y, 2y) + f (x, 2y) (2.71)


= (f (x + 2y + y, y) + f (x, y)) + (f (x + y, y) + f (x, y)) (2.72)
= f (x + 3y, y) + f (x + y, y) + 2f (x, y) (2.73)

Từ (2.70), (2.73) ta được

f (x + 2y, y) = f (x + y, y) ⇒ f (x + y, y) = f (x, y) , ∀x, y (2.74)

Khi đó từ (2.74) ta có:

f (x + ny, y) = f (x, y) ∀n ∈ N

Thay x bởi x − y vào (2.74) ta được:

f (x, y) = f (x − y, y) ⇒ f (x − ny, y) = f (x, y) ∀n ∈ N

⇒ f (x + ny, y) = f (x, y) ∀n ∈ Z. (2.75)


Sử dụng (2.68),(2.74) ta được:

f (x, ny) = nf (x, y) ∀n ∈ N

Thay z = y và y bởi − vào (2.63) ta được:


(2.75)
f (x, 0) = f (x − y, y) + f (x, −y) ⇒ f (x, −y) = −f (x − y, y) = −f (x, y)

⇒ f (x, ny) = nf (x, y) ∀n ∈ Z


2.3 Phương trình hàm - Dãy số 93

y
Thay y bởi ta được:
n
 y    
p y p
f (x, y) = nf x, ∀n ∈ Z ⇒ f x, y = pf x, = f (x, y)
n q q q
⇒ f (x, zy) = zf (x, y) ∀z (2.76)
⇒ f (x, y) = yf (x, 1) ∀y (2.77)
Từ (2.63),(2.74) ta lại có:

f (x, y + z) = f (x, z) + f (x, y) , ∀x, y, z (2.78)


y
Thay y bởi ta vào (2.75) ta có:
m
 n y  y  (2.76) 1  n  1
f x + y, = f x, ⇒ f x + y, y = f (x, y)
m m m m m m
⇒ f (x + zy, y) = f (x, y) , ∀z
Thay x = 0 và chọn z ∈ Q ta được:
(2.77)
f (zy, y) = f (0, y) = yf (0, 1) ∀y, z ⇒ f (x, y) = ay∀x, y.

Vậy ta có f (x, y) = ay, ∀(x, y) ∈ Q2 .


Nhận xét. Đây là bài khó nhất của đêm tiêu thụ bài giảng thứ hai phần Phương
trình hàm và không có đoàn nào làm (!) Cách làm như những bài Phương trình
hàm rời rạc một biến, tuy nhiên khối lượng công việc nhiều hơn hai lần như vậy
vì phải xử lý hai biến và mối liên hệ.
Một số bài toán tương tự:

1. (IMO Longlist 1967) Cho hàm số ϕ(x, y, z) được xác định bởi bộ ba các số
thực x, y, z, thoả mãn có hai hàm số f, g đều được xác định bởi các cặp số
thực thoả mãn:

ϕ(x, y, z) = f (x + y, z) + g(x, y + z)

với mọi x, y, z ∈ R.
Chứng minh rằng tồn tại một hàm số đơn biến thực h sao cho

ϕ(x, y, z) = h(x + y + z)

với mọi số thực x, y, z.


94 HƯỚNG DẪN GIẢI

2. (Olympiad các nước vùng Baltic 1998) Tìm tất cả các hàm số f : Z+ × Z+ →
Z+ thoả mãn các điều kiện sau với mọi số nguyên dương x, y:

(a) f (x, x) = x.
(b) f (x, y) = f (y, x).
(c) (x + y)f (x, y) = yf (x, x + y).

Bài 12

Tìm f : R ⇒ R thoả:

f (x)f (yf (x) − 1) = x2 f (y) − f (x), ∀x, y ∈ R

Lời giải. Thay x = 0 vào đề ta có:

f (0)f (yf (0) − 1) = −f (0)

Nếu f (0) 6= 0 thì ta có f (x) ≡ −1, thay vào đề thì vô lý. Vậy f (0) = 0. Khi đó
thay y = 0 vào đề thì có:

f (x)f (−1) = −f (x), ∀x ∈ R

Suy ra f (−1) = −1.


Bây giờ ta sẽ chứng minh
f (x) = 0 ⇔ x = 0 (2.79)
Chiều ngược đã đúng, ta sẽ chứng minh thêm: Nếu f (x) = 0 thì suy ra x = 0.
Với x thoả f (x) = 0 thì ta thay y = −1 vào đề ta có:

0 = −x2 ⇔ x = 0

Vậy ta có (2.79).
Bây giờ, thay x = y = 1 ta có;

f (1)f (f (1) − 1) = 0.

Do (2.79) nên ta có f (1) = 1. Thay tiếp x = 1 ta có:

f (y − 1) = f (y) − 1.
2.3 Phương trình hàm - Dãy số 95

Khi đó bài toán thành


f (x)f (yf (x)) = x2 f (y) (2.80)
Thay x = y trong (2.80) ta có

f (x)f (f (x)) = x2 , ∀x ∈ R (2.81)

Thay x bởi x − 1 trong (2.81) ta có:

[f (x) − 1][f (f (x)) − 1] = (x − 1)2 ⇔ f (x) + f (f (x)) = 2x.

Thay vào (2.81) ta có f (x) = x, ∀x ∈ R. Thử lại thoả.


Vậy f (x) = x, ∀x ∈ R.

Nhận xét. Mấu chốt của bài toán là chứng minh (2.79). Mọi luaapj luận về sau
đề sử dụng đến (2.79) để được giải quyết đơn giản hơn. Việc tính được f (x − 1) =
f (x) − 1 cũng cho ta thấy hướng đi sẽ thay x bởi x − 1.

Bài 13

Cho f : Q2 → R+ với Q2 = {(x, y)|x, y ∈ Q}. Giả sử f thoả mãn các điều
kiện sau:

1. f (a, b, c) = f (a, c)f (b, c), ∀a, b, c ∈ Q.

2. f (c, ab) = f (c, a)f (c, b), ∀a, b, c ∈ Q.

3. f (a, 1 − a) = 1, ∀a ∈ Q.

Chứng minh rằng f (a, a) = f (a, −a) = 1 và f (a, b)f (b, a) = 1.

Lời giải. Lần lượt thay b = 1 vào điều kiện (a) và (b) ta có:

f (a, c) = f (a, c)f (1, c) ⇔ f (1, c) = 1 (2.82)

f (c, a) = f (c, a)f (c, 1) ⇔ f (c, 1) = 1 (2.83)


(do tập xác định của hàm f là R+ ).
Thay a = b = −1 lần lượt vào điều kiện (1) và (2) ta có:

f (−1, c)2 = f (1, c) = 1 ⇔ f (−1, c) = 1 (2.84)


96 HƯỚNG DẪN GIẢI

f (c, −1)2 = f (c, 1) = 1 ⇔ f (c, −1) = 1 (2.85)


1
Thay a bởi 1 − vào điều kiện (3) ta có:
a
 
1 1
f 1− , =1 (2.86)
a a

1
Thay c = 1 − vào (2.83) ta có:
a
 
1
f 1 − ,1 = 1
a

Khi đó      
1 1 1 1
f 1 − ,a f 1 − , = f 1 − ,1 = 1
a a a a

Do điều kiện (c) nên ta có:  


1
f 1− =1 (2.87)
a
Từ đó    
1 −1
f 1− = f (1 − a, a)f ,a
a a
 
−1
⇒f ,a = 1
a
Khi đó ta có:
  
−1
1 = f (1, a) = f , a f (−a, a) ⇔ f (−a, a) = 1, theo (2.82)


a
 
−1
 1 = f (−1, a) = f , a f (a, a) ⇔ f (a, a) = 1, theo (2.84)


a

Tiếp theo, do f (x, x) = 1, ∀x ∈ Q nên ta có:

f (a, b)f (b, a) = f (a, b)f (a, a)f (b, a)f (b, b) = f (a, ab)f (b, ab)
= f (ab, ab) = 1

Vậy ta có 3 đẳng thức cần chứng minh.


2.3 Phương trình hàm - Dãy số 97

Nhận xét. Nếu nhìn nhận theo phương trình hàm một biến thì ta có thể thấy
rằng, điều kiện (1) cho ta tính nhân tính trái và điều kiện số (2) cho ta tính nhân
tính phải và điều đầu tiên cần chứng minh chính là chứng minh hàm số chẵn trái
nhưng lẻ phải.
Có một các tiếp cận để chứng minh đẳng thức cuối cùng, có phần tự nhiên hơn
nhưng khi làm thì phức tạp hơn như sau:
 
1
Để chứng minh f (a, b)f (b, a) = 1 thì ta cần chứng minh f (b, a) = f , b hoặc
  a
1
f (b, a) = f a, . Ta sẽ chứng minh điều đầu tiên. Ta dễ dàng có được
b
 
−1
f a, =1
a
và  
1
f a, ± =1 (2.88)
a
Sử dụng điều kiện (1) và (2) ta lần lượt có:
   
1 1 1
f (b, a)f ,a = 1 ⇒ f ,a =
b b f (b, a)
     
1 1 1 1 1
f ,a f , =1⇒f ,a =  
b b a b 1 1
f ,
b a
hay  
1 1
f (b, a) = f , .
b a
Từ đó và (2.88) ta có
   
1 1 a 1
f (b, a) = f , =f ,
b a b a
    
a 1 a a a 1
=f , f , =f ,
b a b b b b
     
a 1 1 1
=f , f b, = f a,
b b b b
 
1
Vậy ta có f (b, a) = f a, hay đã có điều cần chứng minh.
b
98 HƯỚNG DẪN GIẢI

Bài 14

Cho hàm số f : N? × N? → N? (với N? = N ∪ {0}) thoả mãn các điều kiện


sau:

1. f (0, x) = x + 1

2. f (x + 1, 0) = f (x, 1)

3. f (x + 1, y + 1) = f (x, f (x + 1, y))

Tính f (1, 2017), f (2, 2017), f (3, 2017), f (4, 2017).

Lời giải. Ta có:

f (1, n) = f (0, f (1, n − 1)) = f (1, n − 1) + 1.

Bằng quy nạp ta có được:

f (1, n) = f (1, 0) + n = n + f (0, 1) = n + 2

Hay f (1, 2017) = 2019.


Tiếp theo:
f (2, n) = f (1, f (2, n − 1)) = f (2, n − 1) + 2.
Bằng quy nạp ta có

f (2, n) = 2n + f (2, 0) = 2n + f (1, n) = 2n + 3.

Hay f (2, 2017) = 4037.


Tiếp theo như quy nạp cho x = 3 như sau:

f (3, n) = f (2, f (3, n − 1)) = 2f (3, n − 1) + 3.

với n ∈ N?
Nếu đặt an = f (3, n) thì ta tính được

a0 = f (3, 0) = f (2, 1) = 5

và dãy truy hồi


an = 2an−1 + 3
2.3 Phương trình hàm - Dãy số 99

Dễ dàng tìm được số hạng tổng quát là an = 2n+3 − 3 với a0 = 5. Vậy ta có


f (3, n) = an = 2n+3 − 3 hay

f (3, 2017) = 22020 − 3

Tiếp tục sử dụng kết quả trường hợp x = 3 cho x = 4 như sau:

f (4, n) = f (3, f (4, n − 1)) = 2f (4,n−1)+3 − 3.

Đặt an = f (4, n) thì ta có

a0 = f (4, 0) = f (3, 1) = 24 − 3 = 13

và dãy truy hồi


an + 3 = 2an−1 +3
Sử dụng quy nạo ta có
2a0 +3 216 2 2
.. .. ..
2. 2. 2.
an + 3 = |2 {z } = |2 {z } = |2 {z }
n lần mũ nlnm n+3 lần mũ

Vậy
22
...
2
f (4, n) = an = |2 {z } −3.
n+3 lần mũ

Thay n = 2017 ta được kết quả là:


22
..
2.
f (4, 2017) = 2| {z } −3.
2020 lần mũ

Nhận xét. Đây là một bài toán rất nỗi tiếng về hàm nhiều biến, có tên gọi là
Phương trình hàm Ackermann, thuộc lớp hàm có thể tính toán được. Trường hợp
x = 4 cũng chính là câu 6 đề IMO 1981. Phương trình hàm Ackermann có thể mở
rộng cho dạng ba biến nhưng vì khối lượng tính toán quá lớn nên phương trình
hàm này rất ít được sử dụng cả trong lý thuyết và ứng dụng.
100 HƯỚNG DẪN GIẢI

Bài 15

Cho dãy số (xn ) được xác định bởi:



x1 = 4
xn+1 = x2n − 2, ∀ ∈ N∗

Tính
xn+1
lim
n→+∞ x1 x2 x3 ...xn

Lời giải. Bằng quy nạp ta chứng minh được xn ≥ 4, ∀n ∈ N∗ ,


Mặt khác:

x2n+1 −4 = (xn −2)2 −4 = x2n (x2n −4) = . . . = (x1 x2 . . . xn )2 (x21 −4) = 12(x1 x2 . . . xn )

Suy ra:
 2
xn+1 4
= 12 +
x1 x2 ..xn (x1 x2 . . . xn )2
Lại có:
xn+1 − xn = x2n − xn − 2 = (xn + 1)(xn − 2) > 0, ∀n ∈ N∗
Do đó: (xn ) là dãy tăng ⇒ xn ≥ 4, ∀n ∈ N∗ .
4 1
Khi đó: 0 ≤ ≤ 2n−1 .
x1 x2 ..xn 4
1
Mà limn→+∞ 2n−1 = 0 nên theo định lí kẹp, ta có:
4
4
lim =0
n→+∞ x1 x2 ..xn

Vậy nên:
s
xn+1 4 √
lim = lim 12 + = 2 3
n→+∞ x1 x2 ..xn n→+∞ (x1 .x2 ...xn )2

Nhận xét. Ý tưởng bài toán trên hoàn toàn giống với một bài toán dãy số trong
kì thi Olympic toán sinh viên toàn quốc năm 2005, chỉ có điều sự khác biệt ở đây
là giá trị x1 = 5.Ý tưởng của bài toán ấy còn được xuất hiện một lần nữa trong
2.3 Phương trình hàm - Dãy số 101

kì thi Olympic toán sinh viên quốc tế năm 2010,với nội dung như sau: Cho dãy số
(xn ) được xác định bởi hệ thức truy hồi :
 √
x1 = 5
xn+1 = x2n − 2, ∀n ∈ N ∗
x1 x2 ...xn
Tìm limn→+∞ .
xn+1

Bài tập bổ sung: Cho dãy số



x0 = a
(xn ) :
xn+1 = 2x2n − 1, ∀n ≥ 0

Tìm các giá trị của a để xn < 0, ∀n ≥ 0.

Bài 16

Cho dãy số (un ) được xác định bởi công thức truy hồi :

 u1 = α ∈ R
u3n + 9un − 6
un+1 = 2 , ∀α ∈ N∗
3un − 6un + 7

Tìm α để dãy số (un ) có giới hạn hữu hạn khi và tìm giới hạn của dãy số
trong các trường hợp đó.

Lời giải. Nếu α = −1 thì un = −1∀n ∈ N∗ .


Xét α 6= −1. Khi đó, ta có:
u3n−1 + 9un−1 − 6
un + 1 = +1
3u2n−1 − 6un−1 + 7

u3n−1 + 3u2n−1 + 3un−1 + 1 (un−1 + 1)3


= =
3u2n−1 − 6un−1 + 7 3u2n−1 − 6un−1 + 7
Lại có:
u3n−1 + 9un−1 − 6
un − 3 = −3
3u2n−1 − 6un−1 + 7
u3n − 9u2n−1 + 27un−1 − 27 (un−1 ˘3)3
= =
3u2n−1 − 6un−1 + 7 3u2n−1 − 6un−1 + 7
102 HƯỚNG DẪN GIẢI

x−3
Xét hàm số f (x) = . Ta có:
x+1
 3
un − 3 un−1 − 3
f (xn ) = =+
un + 1 un−1 + 1
2 n−1 n−1
= [f (xn−1 )]3 = [f (xn−2 )]3 = ... = [f (u1 )]3 = [f (α)]3
Đặt
n−1
β = [f (α)]3
Từ (*) ta có:
un − 3 3 + βn
= βn ⇔ un − 3 = βn un + βn ⇔ un =
un + 1 1 − βn

Vậy:
 3n−1
α−3
3+
α+1
un =  3n−1
α−3
1−
α+1
Biện luận:
α−3
= 1 ⇔ α − 3 = α + 1 ⇔ 0α = 4
α+1
(vô nghiệm)

α − 3 2 2
α + 1 < 1 ⇔ (α − 3) < (α + 1) ⇔ −6α + 9 < 2α + 1 ⇔ α > 1


α − 3
α + 1 > 1 ⇔ −1 6= α < 1

Vậy:

• Nếu α = −1 thì limn→+∞ un = −1.

• Nếu α = 1 thì un = 1∀n ∈ N∗ suy ra limn→+∞ un = 1.

• Nếu α > 1 thì  3n−1


α−3
lim ⇒ lim = 3.
n→+∞ α+1 n→+∞
2.3 Phương trình hàm - Dãy số 103

• Nếu −1 6= α < 1 thì


 −3n−1
α−3
lim = 0 ⇒ lim un = −1.
n→+∞ α+1 n→+∞

Bài 17

Cho hai dãy số (an ), (bn ) được xác định bởi hệ thức truy hồi:

a1 = 3, b1 = 2, an+1 = a2n + 2b2n , bn+1 = 2an bn , ∀n ∈ N∗

Tìm
2n
p √
2n
lim bn ; lim a1 a2 ...an
n→+∞ n→+∞

Lời giải. Ta có:

√ √ √ 2 √ n−1
an + bn 2 = (an−1 + 2bn−1 )2 = (an−2 + 2bn−2 )2 = . . . = (a1 + 2b1 )2
√ n √ n
= (3 + 2 2)2 −1 = ( 2 + 1)2 , ∀n ∈ N∗

Tương tự, ta có: √ √ n


an − bn 2 = ( 2 − 1)2 , ∀n ∈ N∗
Từ hai điều trên suy ra:

1 √ √

n n
 an = [( 2 + 1)2 + ( 2 − 1)2 ]

2
1 √ n √ n
bn = √ [( 2 + 1)2 − ( 2 − 1)2 ]

2 2

Mặt khác: s √
2n ( 2 + 1)2
n
n
p √n

√ < 2 b n < 2 an < 2 + 1
4 2
s √
n
2n ( 2 + 1)2
lim √
n→+∞ 4 2
104 HƯỚNG DẪN GIẢI

2+1
= lim
x→+∞ 1
√ n
(4 2) 2

= 2+1
Theo định lí kẹp ta suy ra:
2n
p √ √
2n
lim bn = lim an = 2+1
x→+∞ n→+∞

Mặt khác:
b2 b3 bn+1 an b n
a1 a2 ..a3 = ... n+1 = n
2b1 2b2 2 2
Do đó:
r

r
√ 2n an b n √ pn n 1
2 + 1)2
2n n 2 2
lim a1 a2 ...an = lim = lim 2
a n . b n . = (
x→+∞ x→+∞ 2n n→+∞ 2n

=3+2 2

r  1
2n 1 1 2n
lim = lim =1
n→+∞ 2n n→+∞ 2n

Nhận xét. Không khó để nhận ra được việc tìm số hạng tổng quá là cần thiết, đặc
biệt sự xuất hiện của a2n + 2b2n và 2an bn có thể tạo ra được hằng đẳng thức.
√ Sau
√n 2n
khi tìm được số hạng tổng quát thì việc tính limn→+∞ an và limn→+∞ bn có
2

nhiều hướng đi, trong đó sử dụng định lý kẹp là hữu hiệu nhất.

Bài 18

Cho dãy số (xn ) được xác định bởi hệ thức truy hồi:

 x1 = a ≥ 1
x2n − 2xn 2
xn+1 = 2
, ∀n ∈ N∗
[xn ]

Chứng minh dãy số (xn ) đã cho có giới hạn hữu hạn khi n → +∞.
2.3 Phương trình hàm - Dãy số 105

Lời giải. Xét các trường hợp:

TH1: a là số nguyên. Khi đó: [a] = a; a = 0. Do đó:

x21 − 2x1 2 a2
x1 = a, x2 = = =1
[x1 ]2 a2

Bằng quy nạp ta chứng minh được: xn = 1, ∀n ∈ N∗ .


Do đó: limx→+∞ xn = 1.

TH2: a không phải là số nguyên. Khi đó:

a2 − 2x1 2 ([a] + a)2 − 2a2


x2 = = (2.89)
[a]2 [a]2
2 2
2[a]2 − ([a] − a)2
 
[a] − a a
= =2− =2− 1− (2.90)
[a]2 [a] [a]

Vì a không phải số nguyên nên 0 < a < 1. Mặt khác a ≥ 1 nên [a] ≥ 1. Từ
a
đó ta có: 0 < < 1.
[a]
Kết hợp với (2.90) ta có: 1 < x2 < 2.
Bằng quy nạp ta chứng minh được:1 < xn < 2, ∀n ∈ N∗ .
Do 1 < xn < 2 nên [xn ] = 1. Suy ra:

xn = xn − [xn ] = xn − 1, ∀n ∈ N∗

Vì thế:
xn+1 = x2n − 2(xn − 1)2 = −x2n + 4xn − 2, ∀n ∈ N∗
Từ đó ta có:
2 n−1
2 − xn+1 = (2 − xn )2 = (2 − xn−1 )2 = ... = (2 − x2 )2

Do đó:
n−1
lim (2 − xn+1 ) = lim (2 − x2 )2 =0
n→+∞ n→+∞

(Vì 0 < 2 − x2 < 1)

Vậy:

• Nếu a ∈ Z+ thì limn→+∞ xn = 1.


106 HƯỚNG DẪN GIẢI

• Nếu a ∈
/ Z, a ≥ 1 thì limn→+∞ xn = 2.

Nhận xét. Một số bài toán dãy số liên quan đến phần nguyên sau sẽ giúp các bạn
tích lũy nhiều phương pháp để giải quyết dạng này tốt hơn:

1. Cho dãy {xn } xác định như sau:



 x1 = 1 
3 , ∀n ≥ 1.
xn+1 = xn
2

Chứng minh rằng dãy {xn } có vô hạn các số chẵn, có vô hạn các số lẻ.

2. Tìm công thức của xn biết x0 = 1 ,


h √ i
xn+1 = xn + xn 5 .

Bài 19

Tìm tất cả các hàm số f : R → R thoả mãn:

f (yf (x + y) + f (x)) = 4x + 2yf (x + y), ∀x, y ∈ R

Lời giải. Thay y = 0 ta có f (f (x)) = 4x, suy ra f song ánh. Khi đó tồn tại a để
f (a) = 0. Khi đó
f (f (0)) = 0 = f (a) ⇒ a = f (0)
do f song ánh.
Mà lại có 4a = f (f (a)) = f (0) = a vậy suy ra a = 0 hay f (0) = 0.
1
Thay x = 0 và y = ta có:
2
    
1 1 1
f f =f
2 2 2
    
1 1 1
Do f song ánh nên f = 1. Mà ta lại có f (1) = f f = 4. = 2 nên
2 2 2
f (2) = f (f (1)) = 4
2.3 Phương trình hàm - Dãy số 107

Thay y = 1 − x vào đề ta có:

f ((1 − x)f (1) + f (x)) = 4x + 2(1 − x)f (1) = 4x + 4 − 4x = 4 = f (2)

Do f đơn ánh và thu gọi vế trái phương trình trên ta có:

2 − 2x + f (x) = 2 ⇔ f (x) = 2x

Thử lại thoả. Vậy f (x) = 2x, ∀x ∈ R.

Bài 20

Đặt I = [0, 1] và G = {(x, y)|x, y ∈ I}. Tìm tất cả các hàm số f : G → I


thoả mãn với mọi x, y, z ∈ I ta có:

1. f (f (x, y), z) = f (x, f (y, z)),

2. f (x, 1) = x, f (1, y) = y,

3. f (zx, zy) = z k f (x, y).

với k là một số thực dương không liên quan đến x, y, z.

Lời giải. Ta sẽ xét hai trường hợp:

• Nếu x ≤ y thì ta có
 
x
f (x, y) = f , 1 y k = xy k−1
y

• Nếu y ≤ x thì ta có
 y
f (x, y) = f 1, xk = xk−1 y
x

Cả hai trường hợp trên đều sử dụng tính chất (3) và (2).
Từ tính chất (1) và 2 kết quả trên, chọn z đủ bé nhưng khác 0 và x ≥ y ta có:
2
f (f (x, y), z) = f (xk−1 y, z) = (xk−1 y)k−1 z = x(k−1) y k−1 z
108 HƯỚNG DẪN GIẢI

f (f (x, y), z) = f (x, f (y, z)) = f (x, y k−1 z = xk−1 y k−1 z


So sánh hay phương trình trên ta có:

2 k=1
(k − 1) = k − 1 ⇔
k=2

• Nếu k = 1 thì ta có f (x, y) = min{x, y}.

• Nếu k = 2 thì ta có f (x, y) = xy.

Nhận xét. Đây thực chất là đề Trung Quốc 1991. Do tính chất mới lạ nên mình
giới thiệu vào cho những ai quan tâm có cơ hội tìm hiểu. Về bài toán, có một số
điều sau:

• Nếu xem cách tác động f vào như phép nhân thì ta có tính chất (1) của hàm
f chính là phép liên hợp ((x.y).z = x.(y.z)). Tính chất số 2 có thể xem như
là điều kiện biên của bài toán. Tính chất 3 chính là tính thuần nhất (hay ta
có thể gọi f là Hàm số thuần nhất, Homogeneous function).

• Bài toán này lại được sử dụng lại vào năm 2004 trong đề chọn đội tuyển
Bulgari, câu 1, ngày 4 như sau: Tìm tất cả các hàm số f : [0, 1] × [0, 1] →
[0, 1] thoả mãn các điều kiện sau với mọi k > 0:

1. f (f (x, y), z) = f (x, f (y, z)),


2. f (x, y) = f (y, x),
3. f (x, 1) = x,
4. f (zx, zy) = z k f (x, y).

với mọi x, y, z ∈ [0, 1].


Bài toán có kết quả như bài Trung Quốc 1991, phần lời giải xin được phép
mời bạn đọc suy nghĩ.
2.4 Số học 109

2.4 Số học
Bài 1

Biết rằng với dãy nguyên dương 1 < k1 < k2 < ... < kn và dãy nguyên tương
ứng s1 , s2 , ..., sn , với mọi số nguyên dương N đều tồn tại i ∈ {1, 2, ..., n} sao
cho N ≡ si (mod ki ).

1. Tìm dãy {kn } và {sn } thoả mãn khi k1 = 2 và khi k1 = 3.


Pn 1
2. Chứng minh rằng i=1 > 1.
ki

3. Tìm n nhỏ nhất để có các dãy TMĐK đề bài.

Lời giải.

1. • Ví dụ 1:
i 1 2 3 4 5
k 2 3 4 6 12
s 1 0 0 4 2
• Ví dụ 2:
i 1 2 3 4 5 6 7 8 9 10 11 12
k 3 4 6 8 9 12 16 18 24 36 48 72
s 1 0 5 6 0 2 7 3 18 33 15 51

2. Chọn N là bội chung của k1 , k2 , . . . , kn . Ta thấy rằng trong tập {1; 2; . . . ; N }


N
có số đồng dư với si theo (mod k)i . Do đó ta có:
ki
n0 n0
X N X 1
≥N ⇔ ≥ 1.
i=1
ki i=1
k i

3. Ta sẽ chứng minh n ≥ 5. Giả sử tồn tại n0 ≤ 4 là số nhỏ nhất để tồn tại 2


dãy số nguyên thoả mãn đề bài.
P 0 1 1 1 1 1
Nếu k1 ≥ 3 thì ni=1 ≤ + + + < 1. Vì thế k1 = 2.
ki 3 4 5 6
Ta cũng có nhận xét là n0 ≥ 3. Ta có 2 trường hợp sau:
110 HƯỚNG DẪN GIẢI

TH1: s1 = 1. Còn lại các số chẵn. Khi đó xét số nguyên dương N chẵn bất
kì:
N si ki
• Nếu ki chẵn thì ≡ (mod ).
2 2 2
N
• Nếu ki lẻ thì ≡ si .2−1 (mod ki ) với 2−1 là phần tử nghịch đảo
2
của 2 theo (mod ki ). Khi đó với i = 2, n0 tồn tại bộ

 ki
0 nếu ki chẵn
ki−1 = 2
k nếu k lẻ
i i

Nếu bộ số trên là phân biệt thì với mọi số nguyên N tồn tại bộ
n0 − 1 số ki0 và s0i tương ứng thoả mãn đề bài. Mâu thuẫn với giả
thiết n0 là số nhỏ nhất.
Vì vậy k10 , k20 , kn0 0 −1 có đúng hai số bằng nhau.
Suy ra (k2 , . . . , kn0 ) = (2m + 1, 4m + 2, k) (k nếu có).

– Với k lẻ ta có (k10 , k20 , kn0 0 −1 ) = (2m + 1, 2m + 1, k) từ đó có

2 1
+ ≥ 1.
2m + 1 k
Do đó m = 1 và k = 3 = 2m + 1 vô lý.
– Với k = 2t với t ≥ 2 ta có (k10 , k20 , kn0 0 −1 ) = (2m + 1, 2m + 1, t)
từ đó có
2 1
+ ≥ 1.
2m + 1 t
Do đó m = 1 và t = 2 hoặc t = 3. Khi t = 3 ta có k = 6 =
4m + 1 vô lý, nên t = 2 và k = 4.
Ta có (k10 , k20 , k30 ) = (2, 3, 3). Rõ ràng khi phân hoạch tập số
nguyên theo mod3 thì mỗi tập đều có số lẻ và số chẵn, nên
bộ nêu trên không thoả mãn.
TH2: s1 = 0. Còn các số lẻ. Ta xét tập các số lẻ trừ đi 1 và bộ (si − 1)
tương ứng. Ta lại có toàn các số chẵn như trường hợp 1.

Vậy n nhỏ nhất là 5.

Nhận xét. Bài toán trên tuy nhìn đề khá khó nhưng chỉ sử dụng một vài kĩ thuật
và nhận xét cơ bản. Câu a. là ví dụ cũng như tiền đề để có ý tưởng của các câu b.
2.4 Số học 111

và c.. Để có các ví dụ này, chúng ta không thể chỉ nâng mod theo lũy thừa của 2
vì khi đó sẽ luôn dư ra ít nhất một số dư. Chú ý: 2, 3, 4, 6, 12 đều là ước của 12.
Quan sát này có gợi cho bạn điều gì không? Khi nhận thấy được quy luật của câu
a. thì kĩ thuật xuống thang của câu c là khá rõ ràng.

Bài 2
.
Tìm n ∈ N và n > 1 sao cho 3n − 1 .. n3 .

Lời giải.

• Gọi p là ước nguyên tố nhỏ nhất của n thì 3n ≡ 1 (mod p). Mà 3p−1 ≡ 1
.
(mod p) nên (n; p − 1) .. ordp (3). Do đó ordp (3) = 1 do cách chọn p. Vì thế
3 ≡ 1 (mod p) ⇒ p = 2.

• Do n chẵn nên v2 (3n − 1) = v2 (3 − 1) + v2 (3 + 1) + v2 (n) − 1 = v2 (n) + 2.Mà


v2 (3n − 1) ≥ 3v2 (n) nên v2 (n) + 2 ≥ 3v2 (n) ⇒ v2 (n) = 1.

• Đặt n = 2m với m lẻ. Nếu m > 1 thì gọi q là ước nguyên tố nhỏ nhất của
.
m. Khi đó (2m; q − 1) .. ordq (3). Từ đó ordq (3) = 1 hoặc ordq (3) = 2. Nhưng
cả 2 trường hợp đều suy ra q = 2 vô lý.
Khi đó m = 1 và n = 2. Thử lại đúng.

Vậy n = 2.

Nhận xét. Đây là một bài toán cơ bản của việc áp dụng bổ đề nâng lũy thừa
LTE. Công thức được áp dụng hoàn toàn có thể được chứng minh bằng cách đặt
n = 2m k với k lẻ và phân tích 3n − 1 thành nhân tử. Thay 3 bằng một số khác,
trong một số trường hợp ta có bài toán tương tự.

Bài 3

Chứng minh rằng tồn tại vô số số nguyên tố dạng 2n k + 1 với k nguyên


dương và n ≥ 2017.

Hướng dẫn. Ta chứng minh 2 ý sau:


112 HƯỚNG DẪN GIẢI
k
• Tập hợp ước nguyên tố của các số có dạng 22 + 1 là vô hạn. Để có điều
n m
đó ta sẽ chứng minh (22 + 1; 22 + 1) = 1 với mọi m 6= n.
n m n m−n
Thật vậy cho m > n và p là ước nguyên tố của 22 +1 thì 22 ≡ (22 )2 ≡
.
1 (mod p) suy ra 22 + 16 .. p.
m

.
• Nếu p là ước nguyên tố của 22 + 1 thì p .. 2n+1 . Ta dễ dàng có điều này
n

bằng cách sử dụng cấp.

Nhận xét. Đây là một bài toán áp dụng tính chất của cấp khá cơ bản. Tuy nhiên
k
việc chỉ ra tập ước nguyên tố của 22 + 1 là vô hạn là rất quan trọng.

Bài 4
.
Tìm số nguyên dương n nhỏ nhất sao cho 1n + 2n + . . . + 2016n 6 .. 2017.

Lời giải. Ta chứng minh bài toán tổng quát sau:


.
Bài toán. Cho p là số nguyên tố lẻ. Chứng minh rằng 1k + 2k + ... + (p − 1)k .. p
.
với mọi k 6 .. p − 1.

Thật vậy, mỗi số nguyên tố p có ít nhất một căn nguyên thuỷ m. Khi đó,
{m; m2 ; . . . ; mp−1 } ≡ {1; 2; . . . ; p − 1}. Vì thế

(mk )p−1
1k + 2k + ... + (p − 1)k = mk + (m2 )k + . . . + (mp−1 )k = mk .
mk − 1
. . .
Do m là căn nguyên thuỷ của p và k 6 .. p−1 nên mk −1 6 .. p, lại có (mk )p−1 −1 .. p.
Suy ra điều phải chứng minh.
Từ bài toán trên suy ra số n nhỏ nhất cần tìm là 2016.

Nhận xét. Bài toán trên được giải khá gọn gàng bằng cách sử dụng căn nguyên
thủy, tuy nhiên bạn đọc hãy tự tìm hiểu vấn đề ở mở đầu bài giải: "Mỗi số nguyên
tố đều có ít nhất một căn nguyên thủy". Đồng thời, bạn hãy thử tìm thêm cách
chứng minh khác bằng quy nạp.
2.4 Số học 113

Bài 5

Cho 9 số nguyên dương phân biệt d1 , d2 , . . . , d9 và đa thức P (x) = (x +


d1 )(x + d2 ) . . . (x + d9 ). Chứng minh rằng có số N nguyên dương sao cho
∀x ≥ N thì P (x) có ước nguyên tố lớn hơn 20.

Lời giải. Gọi d = max{d1 , d2 , . . . , d8 }. Chọn N = d8 .


Giả sử mệnh đề ở đề bài là sai.
Có 8 số nguyên tố nhỏ hơn 20. Khi x > N phân tích P (x) theo 8 số nguyên tố
đầu tiên:
P (x) = 2a1 .3a2 .5a3 .7a4 .11a5 .13a6 .17a7 .19a8
α
Do đó x + di = Π8j=1 pj i,j .
α
Mà x + di > d8 ⇒ x + di có ước p1 i,1 > d.
Có 9 số x + di nên theo định lý Dirichlet tồn tại i 6= j:

. α
x + di .. p1 i,1 > d
. α
x + dj .. p1 j,1 > d

. . .
Chọn α = min{αi,1 ; αj,1 } thì x + di .. pα1 và x + dj .. pα1 . Từ đó suy ra |di − dj | .. pα1 ⇒
|di − dj | > d vô lý.
Vậy ta có đpcm.

Nhận xét. Nhớ rằng số các số nguyên tố nhỏ hơn 20 là 8 số. Trong đề bài lại có 9
nhân tử của P (x), điều này có gợi cho bạn nhớ tới định lý Dirichlet? Sau khi có ý
tưởng thì cuối cùng chỉ là việc chọn sao cho phù hợp.

Bài 6

Cho P (x) ∈ Z[x]. Biết a1 , a2 , . . . , an là các số nguyên thoả mãn: ∀x ∈ Z, ∃i ∈


. .
{1; 2; . . . ; n} sao cho P (x) .. ai . Chứng minh rằng ∃j : P (x) .. aj ∀x ∈ Z.

Lời giải. Ta chứng minh bằng phản chứng.


.
Giả sử ∀i ∈ {1; 2; . . . ; n}, ∃x ∈ Z : P (x) 6 .. ai .
Ta có P (x + tpi ) ≡ P (x)(mod p)∀x ∈ Z, t ∈ Z, p nguyên tố. Với mỗi số ai ta lấy 1
ước nguyên tố, lấy lần lượt sao cho được tập {p1 , p2 , . . . , pk } gồm các số nguyên
114 HƯỚNG DẪN GIẢI

tố phân biệt.
Theo giả sử phản chứng, ta đặt:
.
P (xi ) 6 .. pi , i = 1, k

Và chọn được số x sao cho x ≡ xi (mod p)i theo định lí thặng sư Trung Hoa.
. .
Khi đó P (x) 6 .. pi ∀i = 1, k nên P (x) 6 .. ai ∀i = 1, k, trái giả thiết.
Vậy ta có đpcm.
Nhận xét. Điều phải chứng minh dễ dàng dẫn chúng ta tới ý tưởng phản chứng.
Nhớ rằng chúng ta có công cụ chọn số khá mạnh trong tay là định lý thặng dư
Trung Hoa, sau đó chọn thế nào cho đơn giản và phù hợp.

Bài 7

Tìm các số nguyên dương n sao cho với mọi số nguyên dương k, tồn tại số
tự nhiên a sao cho a3 + a − k chia hết cho n.

Lời giải. Bài toán tương đương với việc tìm n sao cho {a3 + a | a = 1, 2, . . . , n}
là hệ thặng dư đầy đủ modulo n.
Gọi p là một ước nguyên tố của n, ta sẽ chứng minh p = 3.
Do {a3 + a | i = 1, 2, . . . , n} là hệ thặng dư đầy đủ modulo n nên {a3 + a | i =
1, 2, . . . , p − 1} là hệ thặng dư thu gọn modulo p. Từ đó:

(13 + 1)(23 + 2) . . . ((p − 1)3 + (p − 1)) ≡ −1 (mod p).

Suy ra
(12 + 1)(22 + 1) . . . ((p − 1)2 + 1) ≡ 1 (mod p)
Với p nguyên tố ta có:

• (12 + 1)(22 + 1) . . . ((p − 1)2 + 1) ≡ 0 (mod p) với p ≡ 1 (mod 4).

• (12 + 1)(22 + 1) . . . ((p − 1)2 + 1) ≡ 4 (mod p) với p ≡ 3 (mod 4).

(Chứng minh điều trên bằng hệ thặng dư hoặc bằng định lý Lagrange cho số
học)
Do đó p chỉ có thể bằng 3.
Ta chứng minh các số n có dạng 3k đều thoả mãn bằng quy nạp.

• k = 1 thì mệnh đề đúng.


2.4 Số học 115

• Giả sử mệnh đề đúng đến k, ta chứng minh nó đúng với k + 1. Thật vậy
ta có {i3 + i | i = 1, 2, . . . , 3k } là hệ thặng dư đầy đủ modulo3k , cần chứng
minh {i3 + i | i = 1, 2, . . . , 3k+1 } là hệ thặng dư đầy đủ modulo3k+1 .
Ta viết i3 + i = 3k .qi + ri với mỗi i = 1, 2, . . . , 3k (ri < 3k ) và i = 3k + ti với
mỗi 3k < i ≤ 2.3k , i = 2.3k + ti với mỗi 2.3k < i ≤ 3k+1 .

– Với 1 ≤ i ≤ 3k ta có:
.
Nếu qi .. 3 thì i3 + i ≡ ri (mod 3k+1 ).
Nếu qi ≡ 1 (mod 3) thì i3 + i ≡ 3k + ri (mod 3k+1 ).
Nếu qi ≡ 2 (mod 3) thì i3 + i ≡ 2.3k + ri (mod 3k+1 ).
– Với 3k < i ≤ 2.3k ta có:

i3 + i ≡ t3i + ti + 3k (mod 3k+1 )

với ti = 1, 2, . . . , 3k .
– Với 2.3k < i ≤ 3k+1 ta có:

i3 + i ≡ t3i + ti + 2.3k (mod 3k+1 )

với ti = 1, 2, . . . , 3k .

Từ các điều trên, kết hợp với giả thiết quy nạp ta có được đpcm.

Nhận xét. Bài toán có sử dụng định lý Lagrange trong số học: Với đa thức P (x)
bậc n, phương trình P (x) ≡ a (mod p) có không quá n nghiệm trên modp.

Bài 8

Tìm đa thức P (x) hệ số nguyên biết với mọi số nguyên tố p, a, b nguyên


dương thì ab ≡ 1 (mod p) ⇒ P (a).P (b) ≡ 1 (mod p).

Lời giải.

1
• Đặt degP = n và Q(x) = xn P ( ) thì Q(x) ∈ Z[x]. Kí hiệu a−1 là số nguyên
x
để a.a−1 ≡ 1 (mod p) thì P (a)P (a−1 ) ≡ 1 (mod p). Từ đó an P (a)P (a−1 ) ≡
an (mod p).
116 HƯỚNG DẪN GIẢI

1
• an .P (a−1 ) ≡ an .P ( ) ≡ Q(a) (mod p). Do đó P (a)Q(a) ≡ an (mod p) với
a
mọi a sao cho (a, p) = 1.

• Từ đó theo định lý Lagrange thì P (x).Q(x)−xn chia hết cho mọi số nguyên
tố p nên P (x)Q(x) = xn . Mà deqP = n nên P (x) = xn , Q(x) = 1 hoặc
P (x) = −xn , Q(x) = −1.

Nhận xét. Bài toán có sử dụng định lý Lagrange trong số học và phần tử nghịch
đảo: Với mỗi số nguyên tố p và số nguyên a không chia hết cho p, tồn tại duy nhất
số a−1 sao cho a.a−1 ≡ 1 (mod p).

Bài 9

An và Bảo cùng nhau chơi một trò chơi: họ lần lượt viết các số tuỳ thích lên
bảng thành một dòng, mỗi người 3 số, An viết trước. Sau đó Bảo "nhường"
An điền dấu + hoặc − tuỳ ý vào giữa các số đã viết. An thắng nếu kết quả
trên bảng không chia hết cho bất cứ số tự nhiên nào từ 11 đến 18. Bảo
thắng nếu xảy ra trường hợp ngược lại. An nói rằng mình kiểm soát nhiều
hơn, nên chắc chắn chiến thắng. Bạn có đồng ý không? Tại sao?

Lời giải. Bảo mới là người có chiến thuật thắng.


Thật vậy, ở 2 lượt đầu Bảo điền số 11.12.13 . . . 18. Ta sẽ chứng minh Bảo có cách
điền ở lượt cuối cùng để đảm bảo thắng.
Gọi số Bảo cần điền ở lượt cuối là x.
Sau lượt đi thứ ba của An, bỏ qua các số Bảo đã điền (do chúng chia hết cho
11, 12, 13, . . . , 18), ta có 8 kết quả có thể thu được (do có 23 = 8 cách đặt dấu)
là a1 , a2 , . . . , a8 .
Do các số này cùng tính chẵn, lẻ nên chỉ có 2 loại số dư khi chia 8 số này cho 4.
Hơn thế, có đúng 4 số đồng dư với nhau theo mod 4.
Có 3 số đồng dư với nhau theo (mod 3) theo nguyên lý Dirichlet, giả sử là
a1 ≡ a2 ≡ a3 (mod 3). Không giảm tổng quát giả sử a4 ≡ a3 (mod 4).
Đến đây ta chỉ cần chứng minh tồn tại x sao cho a1 + x, a2 + x, . . . , a8 + x chia
hết cho ít nhất một trong các số 11, 12, 13, . . . , 18. Khi đó bài toán được chứng
minh do với mỗi i luôn tồn tại j sao cho ai − x = −(aj + x).
. .
Theo định lý Thặng dư Trung Hoa, ta chọn được: a1 + x .. 9; a2 + x .. 5; a4 +
. . . . .
x .. 16; a5 + x .. 11; a6 + x .. 13; a7 + x .. 17; a8 +, .. 7.
2.4 Số học 117

. . . . . .
Khi đó a1 +x .. 18; a2 +x .. 15; a3 +x .. 12; a4 +x .. 16; a5 +x .. 11; a6 +x .. 13; a7 +
. .
x .. 17; a8 + x .. 14.
Nhận xét. Bài toán trên là một sự kết hợp thú vị giữa số học và tổ hợp. Lưu ý
rằng các số từ 11 đến 18 có tổng cộng 7 ước nguyên tố, nên mục đích của việc chọn
số sẽ là làm cho các ước số nguyên tố đó xuất hiện.

Bài 10

Giả sử N∗ phân hoạch thành 3 dãy tăng {an }, {bn }, {cn } thoả mãn:

i. can = bn + 1

ii. an+1 > bn

iii. cn+1 cn − (n + 1)cn+1 − ncn chẵn.

Chứng minh rằng an = n2 .

Lời giải. Ta chứng minh được những điều sau:

• cn ≥ n.
• an < bn < can < an+1 < bn+1
• Trong đoạn [1; 2; 3; . . . ; c( an )] có:

n số hạng của dãy {an },


n số hạng của dãy {bn },
an số hạng của dãy {cn }.

Từ đó có can = an + 2n.
Với 1 vài biến đổi từ iii. ta có can −1 , can , can +1 cùng tính chẵn, lẻ. Khi đó
thấy được can −1 , bn , can , an+1 , can +1 là các số liên tiếp. Vì thế can = an+1 − 1.
Từ i, ii và iii, ta xét c1 = 1 hay b1 = 1 đều không được, suy ra a1 = 1.
Như vậy theo quy nạp ta có an = n2 .

Nhận xét. Sau khi đã nhận ra được một vài tính chất của dãy số thì điều phải
chứng minh là khá rõ ràng.
118 HƯỚNG DẪN GIẢI

Bài 11

Tìm các đa thức thoả điều kiện: a2 − b2 ∈ Q thì P (a) − P (b) ∈ Q.

Lời giải. Không giảm tổng quát, giả sử P (0) = 0.


√ √
• Với mọi số nguyên √dương n thì ( n)2 ∈ Z+ ⇒ P ( n) ∈ Q. Chứng minh
tương tự ta có P (− n) ∈ Q.

• Đặt P (x) = Q(x2 ) + xH(x2 ). Do có vô số x mà P (x) hữu tỉ nên tất cả hệ


số của P (x) đều√
hữu tỉ, suy ra tất cả hệ số của Q(x) và H(x) đều hữu tỉ.
Khi đó cho x = n với n nguyên và không là số chính phương ta suy ra
H(n) = 0 với vô số n. Vậy thì H(x) = 0 ∀x ∈ R.

• Từ điều trên ta có P (x)√= Q(x2 ) ∀x√∈ R. √ √


Lấy x > 0 bất kì có ( x + 1)2 − ( x)2 ∈ Z nên P ( x + 1) − P ( x) =
Q(x + 1) − Q(x) ∈ Q.
Nếu deg(Q(x + 1) − Q(x)) ≥ 1 thì tồn tại số nguyên tố p đủ lớn để có x

thoả |Q(x + 1) − Q(x)| = p (do Q(x + 1) − Q(x) liên tục trên R) là số vô
tỉ, vô lý. Vì thế Q(x + 1) − Q(x) là hằng số hữu tỉ. Hơn thế, Q(0) = 0 nên
Q(x) = ax với a ∈ Q.
Vì vậy P (x) = ax2 + b với a hữu tỉ.

Nhận xét. Bằng một số tính chất của đa thức, đặc biệt là tính chất liên tục, ta có
được lời giải hoàn chỉnh.

Bài 12

Tìm tất cả đa thức P (x) hệ số nguyên thoả mãn

2n | P (3n ) ∀n ∈ N∗

Lời giải. Nếu deg P > 0. Ta viết P (x) = (x − 1)k .Q(x) với Q(1) 6= 0 và k ∈ N.
Ta có 2n | (3n − 1)k .Q(3n ). Chọn n > k và n lẻ thì

n ≤ kv2 (3n − 1) + v2 (Q(3n )) = k + v2 (Q(3n ))

⇒ v2 (Q(3n )) ≥ n − k
2.4 Số học 119

.
Do đó Q(3n ) .. 2n−k với n > k và n lẻ.
Với n chẵn ta có
n ≤ k(v2 (n) + 2) + v2 (Q(3n )).
.
Chọn n đủ lớn và v2 (n) phù hợp, ta có v2 (Q(3n )) .. 2m với m ≤ n − k(v2 (n) + 2).
. .
Từ đó Q(1) .. 2m với m lớn đủ (vì Q(3n ) − Q(1) .. 3n − 1).
Khi đó suy ra Q(1) = 0 vô lý.
Vì vậy degP = 0 nên P (x) là hằng số, suy ra P (x) = 0 ∀x ∈ R.

Nhận xét. Bài toán khá khó kết hợp giữa bổ đề nâng lũy thừa LTE và cách chọn
số cho phù hợp. Kĩ thuật chọn số là một trong những kĩ thuật quan trọng nhất
khi tiếp cận các bài toán số học.

Bài 13

Tìm các đa thức P (x) hệ số nguyên, a, b nguyên dương và a > b sao cho:

P (n) | an − bn ∀n ∈ N∗

Lời giải. Trước hết xin phát biểu không chứng minh bổ đề dưới đây.
Bổ đề 4. Với đa thức P (x) hệ số nguyên và deg P > 0, tập {p ∈ P : ∃n ∈ Z+ : p |
P (n)} là tập vô hạn.
Chứng minh bổ đề trên dễ dàng xin giành cho bạn đọc.

Trở lại bài toán.

• Nếu deg P = 0 thì P (x) = d với d | a − b.

• Nếu deg P > 0, lấy p nguyên tố sao cho p | P (n0 ) với n0 nguyên dương và
p > a > b thì có p | an0 − bn0 và p | an0 +p − bn0 +p do P (n0 ) ≡ P (n0 + p)
(mod p).
Lại có an0 ≡ bn0 (mod p) nên p | an0 (ap − bp ). Mặt khác p > a > a − b nện
.
p 6 .. a − b. Nhưng ap − bp ≡ a − b (mod p), suy ra điều vô lý.

Vậy P (x) = d ∀x ∈ R và d | a − b.
120 HƯỚNG DẪN GIẢI

Bài 14

Tìm tất cả các đa thức P (x) hệ số nguyên sao cho

(P (n); P (2017n )) = 1 ∀n ∈ Z+

Lời giải. Nếu deg P > 0 thì tồn tại p nguyên tố là ước của P (2017m ) với m
nguyên dương nào đó (áp dụng bổ đề Schur).
Theo định lý thặng dư Trung Hoa, ta tìm được số n sao cho
(
n ≡ 2017m (mod p)
n ≡ m (mod p − 1))

Lúc đó P (n) và P (2017n ) cùng chia hết cho p, vô lý.


Do đó degP = 0. Vì vậy P (x) = 1∀x ∈ R hoặc P (x) = −1∀x ∈ R.

Nhận xét. Ta cần một giá trị đa thức cụ thể làm "mốc". Sau đó bằng cách áp dụng
định lý thặng dư Trung Hoa ta tìm được các số và đa thức tương ứng thoả mãn.

Bài 15

Cho số nguyên dương d. Gọi f (d) là số nguyên dương nhỏ nhất có đúng d
.
ước nguyên dương. Chứng minh rằng f (2k+1 ) .. f (2k ) ∀k ∈ N.

Lời giải. Với k nguyên dương bất kì, số ước của số nguyên dương f (2k ) =
Πni=1 pαi i là Πni=1 (αi + 1) = 2k . Khi đó αi = 2βi − 1 = 1 + 2 + . . . + 2βi −1 với mọi
i = 1, n.
Lúc đó ta viết
βi −1 2j
f (2k ) = Πm
i=1 Πj=1 pi .
m
Ta xét tập T = {p2 |p nguyên tố , m ∈ N}. Khi đó f (2k ) bằng tích của k phần tử
nhỏ nhất của tập T . Vì vậy dễ thấy đpcm.

Nhận xét. Bằng một cách viết khác, ta tìm được quy luật của số f (d).
2.5 Tổ hợp 121

2.5 Tổ hợp
Bài 1

23 người bạn muốn cùng nhau chơi bóng đá. Họ sẽ phải chọn ra một người
làm trọng tài và 22 người còn lại chia làm hai đội đá với nhau. Họ muốn
chia sao cho tổng cân nặng của mỗi đội là bằng nhau. Giả sử cân nặng của
từng người trong số 23 người là các số nguyên dương và với bất kì cách
chọn trọng tài nào thì họ cũng có thể chia thành hai đội mà tổng cân nặng
của mỗi đội bằng nhau. Chứng minh rằng 23 người này có cân nặng bằng
nhau.

Lời giải. Đặt a1 , a2 , .., a23 là cân nặng của những người này và S = a1 + a2 + .. +
a23 . Từ giả thiết, ta có nếu bỏ mỗi ai ra khỏi S thì ta đều chia được hai đội có
tổng khối lượng bằng nhau. Do đó, S − ai = 2P . Điều này chứng tỏ rằng S và
các số ai có cùng tính chẵn lẻ. Ta thực hiện tạo ra một bộ b1 , b2 , .., b23 mới cũng
thoả mãn yêu cầu của đề bài bằng cách:

ai
• Nếu tất cả ai đều chẵn thì đặt bi = 2

• Nếu tất cả các ai đều lẻ thì đặt bi = ai − 1

Nếu bi không đồng thời bằng 0 thì ta vẫn tiếp tục thực hiện quá trình trên, khi
đó, tổng S sẽ giảm sau mỗi quá trình và giảm tới 0. Từ đó, suy ngược lại thì ta
được các ai bằng nhau, do đó cân nặng của mỗi người là bằng nhau.

Nhận xét. Chúng ta vừa sử dụng mô hình đơn biến (một số tác giả gọi là phương
pháp xuống thang) để chứng minh bài toán trên, và đó cũng chính là mấu chốt
quan trọng nhất của bài toán, đưa số cân nặng giảm dần. Một số bài toán sau
đây cũng sử dụng mô hình này để giải nhưng mức độ phức tạp hơn, và cần thêm
những lập luận để đi tới kết quả chứ không đơn giản như bài tập này.

Ví dụ 1. Cho 2017 số nguyên dương đặt trên một vòng tròn. Biết rằng với 6 số
liên tiếp tuỳ ý, ta luôn có thể chia chứng thành hai nhóm ba có tổng bằng nhau.
Chứng minh rằng tất cả các số đã cho bằng nhau.

Hướng dẫn. Ta có, cứ 6 số liên tiếp thì chia thành hai nhóm 3 bằng nhau nên
.
a1 + a2 + a3 + a4 + a5 + a6 .. 2.
122 HƯỚNG DẪN GIẢI

.
a2 + a3 + a4 + a5 + a6 + a7 .. 2.
⇒ a1 ≡ a7 (mod 2)
Tương tự, ta suy ra được
ai ≡ ai+6 (mod 2)
Do 6 và 2017 có cùng tính chẵn lẻ nên tất cả các số trên vòng tròn có cùng tính
chẵn lẻ. Ta thực hiện thao tác sau liên tiếp:
ai
• Nếu các số ai trên vòng cùng chẵn thì thay ai bởi .
2
ai + 1
• Nếu các số ai trên vòng cùng lẻ thì thay ai bởi .
2

Khi đó, bộ các số ai mới vẫn thoả yêu cầu đề bài. Tuy nhiên tổng S = |a1 | + . . . +
|a2017 | sẽ giảm ngặt nếu có một số trong bộ bằng 1. Vì ta thu được một bộ các số
toàn các số 1 nên dãy ban đầu có các số ai bằng nhau.

Nhận xét. Bài toán trên có thể thay việc chia 2 nhóm thành 3, 4, 5, ... nhóm mà
vẫn giải được bằng cách tương tự. Ta xét các bài tương tự sau.

Ví dụ 2. Bộ 5 số nguyên là tốt nếu có thể đặt chúng là a, b, c, d, e để a−b+c−d+e =


29. Tìm tất cả các bộ 2017 số sao cho 5 số liên tiếp bất kì trong chúng đều tốt.

Hướng dẫn. Ở bài toán này, điểm khó là không biết các số đã cho có dương hay
không, vì thế, đại lượng tổng ở trên không xét tiếp tục được. Tuy nhiên, cách áp
dụng vẫn tương tự như sau:

• Từ tất các các số của bộ cho 29, ta thu được điều kiện tốt trở thành a − b +
c − d + e = 0.

• Tất các các số đã cho cùng là số chẵn.


ai
• Xét đại lượng S = Σ2017
i=1 | 2 | thì thông qua phép chia 2, tổng này phải giảm
ngặt. Từ đó suy ra tất cả các số này phải là 0 và tất cả các số ban đầu là 29.

Ví dụ 3 (VMO 2014). Tìm tất cả các bộ 2014 số hữu tỉ không âm sao cho nếu bỏ
đi bất kì số nào trong chúng thì các số còn lại có thể chia được thành 3 nhóm rời
nhau, mỗi nhóm có 671 số sao cho tích các số trong mỗi nhóm là bằng nhau.
2.5 Tổ hợp 123

Hướng dẫn. Bài toán này khó hơn các bài trước vì: các số cho trong đề bài là số
hữu tỉ mà không phải là số nguyên như các bài trước và phép toán đề cập trong
bài này là phép toán nhân mà không phải là cộng. Ta lần lượt giải quyết hai vấn
đề này như sau:

• Qui đồng mẫu để đưa về số nguyên.

• Xét số mũ của 1 ước nguyên tố để đưa về dạng tổng.

• Chú ý thêm trường hợp số 0 (nếu có 1 số thì phải có ít nhất 4 số).

Bài 2

Bạn An chơi trò chơi xếp hình với luật chơi như sau. Cho một hình vuông
4 × 4 chia thành 16 ô, có 15 mảnh ghép và một ô trống. Trong mỗi bước
chơi, An sẽ được phép trượt các mảnh ghép vào ô trống để thu được hình
mới. Bạn An sẽ thắng nếu sau hữu hạn bước trượt, An thu được hình như
sau:

Hỏi An có thể chiến thắng nếu hình ban đầu là hình sau hay không?

Lời giải. Ta đánh số các ô trong bảng ban đầu từ 1 đến 15 theo chiều từ trái
sang phải, từ trên xuống dưới. Ta cũng thực hiện đánh số các miếng ghép trong
124 HƯỚNG DẪN GIẢI

bảng ban đầu với thứ tự như vậy. Đối với một hình bất kì, gọi các miếng ghép
trong bảng tại vị trí i là ai . Khi đó, bảng ban đầu sẽ có ai = i,i = 1, .., 15. Ta sẽ
đếm số các cặp nghịch đối (i, j) sao cho i < j, ai > aj . Ban đầu thì số cặp (i, j)
này là bằng 0. Một phép trượt theo hàng sẽ không làm thay đổi số lượng cặp
này. Một phép trượt theo cột sẽ làm thay đổi lượng này một số lẻ. Mà vì ở hình
ban đầu và hình cuối cùng thì ô trống đều nằm ở góc phải dưới nên số phép
trượt theo cột là chẵn. Điều đó chứng tỏ rằng tính chẵn lẻ của số cặp nghịch đối
được bảo toàn. Do đó, ta không thể chuyển hình ban đầu về thành hình cuối
sau hữu hạn bước.

Nhận xét. Bài toán trên có đề cập tới khái niệm cặp ngịch đối, được cải biên từ
bài toán sau đây.

Ví dụ 4 (Pablo Soberon Bravo). Cho 2017 số nguyên dương xếp theo thứ tự. Ở
mỗi bước, ta thực hiện đổi chỗ 2 số bất kì, hỏi sau 2017 bước nào đó ta có thể thu
được dãy ban đầu hay không?

Ta có thể dễ dàng khẳng định được là không thông qua việc đếm số cặp nghịch đối
trong dãy.

Bài 3

Cho một dãy vô hạn a1 , a2 , .., an , ... với a1 = 1 và mỗi n > 1 thì:

• Nếu ước lẻ lớn nhất của n đồng dư với 1 module 4 thì an = an−1 + 1.

• Nếu ước lẻ lớn nhất của n đồng dư với 3 module 4 thì an = an−1 − 1.

1. Chứng minh rằng trong dãy số đó mỗi số nguyên dương xuất hiện vô
số lần.

2. Đặt
bn = min(ai = n)
i∈N

Tìm một công thức tính bn theo n.

Lời giải. Đề bài khiến ta tìm ước lẻ lớn nhất của một số tự nhiên bất kì, do đó,
mỗi số tự nhiên an , ta sẽ thực hiện phân tích an = 2m .pn với pn lẻ thì pn chính
là ước lẻ lớn nhất của a. Để có cái nhìn rõ ràng về sự tăng giảm của dãy số an ,
2.5 Tổ hợp 125

ta cần xét dãy số qn sau đây


(
1, nếu pn ≡ 1 (mod 4)
qn =
−1, nếu pn ≡ 3 (mod 4)

Ta dựa vào bảng sau để đoán qui luật của an .


q a
1 1 1
2 1 2
3 1 −1
4 1 2
5 1 3
6 −1 2
7 −1 1
8 1 2
9 1 3
10 1 4
11 −1 3
12 −1 2
13 1 3
14 −1 2
15 −1 1
16 1 2
17 1 3
18 1 4
19 −1 3
20 1 4

Ta có thể dễ dàng đoán được quy luật của dãy số này là tăng từ 1 lên 2, sau đó
giảm về 1, xong lại tăng lên 3, rồi lại giảm về 1,... Việc này được xảy ra vô hạn
lần nên mỗi số tự nhiên xuất hiện vô hạn lần trong dãy, và đây chính là điều
mà người giải hướng đến. Bài toán này có một điểm thú vị là dãy số đề cho có
rất nhiều tính chất. Do đó, mấu chốt là phải lựa chọn một số tính chất có lợi
cho việc chứng minh. Ở đây, mình xin phép đưa ra một lời giải cho bài toán này.
Một số tính chất mà ta sẽ sử dụng đến đó là

• q2m = 1, ∀m
• i, j đối xứng nhau qua 2k thì qi = −qj
126 HƯỚNG DẪN GIẢI

• a2m −1 = 1, ∀m

• Tồn tại một số hạng an0 với 2m ≤ n0 < 2m+1 sao cho un0 = m + 1

Bằng việc chứng minh 4 tính chất này và sử dụng các lập luận rằng trong mỗi
đoạn 2m − 1 đến 2m+1 − 1 thì dãy số an tăng từ 1 đến m + 1 rồi giảm xuống 1
theo bước nhảy là 1, ta sẽ thu được kết quả là dãy số an lặp lại vô hạn lần.

1. Xét dãy (qn ) với


(
1, nếu pn ≡ 1 (mod 4)
qn =
−1, nếu pn ≡ 3 (mod 4)

thì ta có an = an−1 + qn . Ta chứng minh 4 tính chất sau

(a) q2m = 1, ∀m. Tính chất này có được hiển nhiên do 2m ≡ 1 (mod 4)
(b) Nếu i+j
2
= 2m thì qi = −qj . Thật vậy, không mất tính tổng quát ta giả
sử i < j và i = 2k .p ≡ 1 (mod 4) thì j = 2m+1 −2k .p = 2k (2m+1−k −p)
nên ước lẻ lớn nhất của j là 2m+1−k −p ≡ −1 (mod 4) . Do đó qi = −qj
(c) a2m −1 = 1. Ta có a1 = 1. Ta xét các bộ đối xứng qua 2m là:

(1, 2m − 1), (2, 2m − 2), .., (2m−1 − 1, 2m−1 + 1)

nên theo tính chất 3, qn tại các bộ này là đối nhau. Hơn nữa, a1 =
a2m −1 = 1 nên từ 2 đến 2m − 1 có số lần xuất hiện 1 và −1 trong dãy
qn là bằng nhau, do đó u2m −1 = u1 = 1
(d) ∀m, ∃n0 , 2m ≤ n0 < 2m+1 : un0 = m + 1 (*). Ta chứng minh điều này
bằng qui nạp
(*) đúng với m = 1 do từ u2 đến u22 = u4 có u2 = 2.
Giả sử (*) đúng với m. Ta chứng minh (*) đúng với m + 1. Ta có:
un0 = m + 1 và u2m+1 −1 = 1 nên từ vị trí n0 + 1 đến 2m+1 − 1 thì
dãy qn có lượng số 1 nhiều hơn tổng số số −1 là m. Xét đối xứng
qua vị trí 2m+1 thì dãy số an tăng m đơn vị từ vị trí 2m+1 + 1 đến
2m+1 + 2m+1 − 1 − n0 = 2m+2 − 1 − n0 . Hơn nữa, q2m+1 = 1 nên từ vị trí
2m+1 đến vị trí 2m+2 − 1 − n0 thì an tăng m + 1 đơn vị. Do a2m+1 −1 = 1
nên a2m+2 −1−n0 = m + 2.
Vậy, (*) đúng với n + 1. Do đó, tính chất 4 được chứng minh.
2.5 Tổ hợp 127

Theo tính chất 3 và 4, ta có trong mỗi đoạn từ 2k − 1 đến 2k+1 − 1 thì an


tăng từ 1 đến n + 1 và giảm lại tới 1, do đó, các số tự nhiên được lặp lại
vô hạn lần trong an

2. Từ lập luận trong tính chất 4 thì ta có được rằng, vị trí đầu tiên của ai
bằng m sẽ có được từ vị trí aj bằng m − 1 bằng cách lấy đối xứng của i
qua 2m rồi cộng 1. Do đó, ta có được công thức truy hồi của bn là:

bn = 2n − 1 − bn−1

. Từ đó, ta có thể chứng minh công thức của bn bằng qui nạp như sau:
( 2n+1
b2n = 22n−1 + 22n−3 + .. + 21 = 2 3 −2
2n+2
b2n+1 = 22n + 22n−2 + .. + 20 = 2 3 −1

Vậy, công thức tổng quát của bn là:


(
22n+1 −2
b2n = 22n−1 + 22n−3 + .. + 21 = 3
22n+2 −1
b2n+1 = 22n + 22n−2 + .. + 20 = 3
128 HƯỚNG DẪN GIẢI

Bài 4

Cho lục giác đều ABCDEF cạnh 1 được điền các số như hình vẽ

0
F B
2 1

0
1 C
E

1
D

Có một con ếch ở vị trí A nhảy xung quanh các đỉnh của đa giác với độ dài
các bước nhảy nguyên. Gọi m là số cách nhảy của ếch sao cho tổng các số
nó nhảy qua là 2017. Chứng minh rằng m không là số chính phương.


Lời giải. Ta thấy 4ACE và 4BDF là hai tam giác đều có cạnh là 3 nên mỗi
lần nhảy, ếch sẽ nhảy từ một đỉnh của tam giác này sang đỉnh của tam giác kia.
Chia nhóm I = (A, C, E) tương ứng với các số (0, 0, 1) và II = (B, D, F ) tương
ứng với các số (1, 1, 2). Ta thấy {x + y|x ∈ I, y ∈ II} = {1, 1, 1, 1, 2, 2, 2, 2, 3}
chứng tỏ tổng các số trên hai bước nhảy liên tiếp của ếch có thể nhận được là 4
trường hợp bằng 1, 4 trường hợp bằng 2 và 1 trường hợp bằng 3. Nếu gọi sn là
số hành trình của ếch có tổng là n thông qua chẵn bước thì

sn = 4sn−1 + 4sn−2 + sn−3 (∗)

Một cách tương tự, gọi tn là số hành trình của ếch có tổng là n thông qua lẻ
bước thì công thức truy hồi vẫn là (*). Vì vậy, nếu gọi un = sn + tn là số hành
trình của ếch sao cho tổng các số nhảy qua là n thì

un = 4un−1 + 4un−2 + un−3 , với n ≥ 3


2.5 Tổ hợp 129

Ta có u0 = 1, u1 = 6, u2 = 28 và từ công thức truy hồi thì m = u2017 ≡ 2 (mod4)


nên m không thể là số chính phương

Nhận xét. Bài toán có thể giải bằng cách đặt 6 dãy truy hồi an , bn , cn , dn , en , fn
chỉ số hành trình của ếch có tổng bằng n và kết thúc tại A, B, C, D, E, F . Tuy
nhiên, cách tiếp cận đó khá phức tạp, đòi hỏi phải khác thác nhiều mối liên hệ
giữa các đường đi. Ta cũng có một bài toán tương tự
Ví dụ 5 (Saudi Arabia TST 2017). Người ta đặt các số 1, 2, 3, 4 trên vòng tròn
theo thứ tự đó. Một con kiến xuất phát từ số 1 và ở mỗi bước, nó s4 bò qua số bên
cạnh. Hỏi con kiến có bao nhiêu cách bò sao cho tổng tất cả các số mà nó bò qua
(kể cả số ban đầu) là 21.

Bài 5

Cho hai dung dịch A và B thoả mãn:

i. Số đo khối lượng của 1 lít A bằng số đo thể tích của 1kg B.

ii. p lít dung dịch A nặng bằng q lít dung dịch B với p, q là số nguyên tố
phân biệt.

Người ta chia các dung dịch A và B vào các bình giống nhau chứa 1 lít
dung dịch và vỏ bình nặng 1kg. Chứng minh rằng có duy nhất 1 cách để
ghép các bình cùng loại A hoặc B với nhau sao cho tổng khối lượng thuộc
(2017, 2018).

Lời giải. Trước hết, ta xét định lí Beatty với nội udng như sau: Cho hai số vô tỉ
dương α, β. Xét hai dãy số

A = ([nα])
B = ([nβ])
1 1
+ = 1 khi và chỉ khi A, B là phân hoạch của Z+ .
α β
Định lí này có thể chứng minh bằng cách sử dụng các bất đẳng thức về phần
nguyên. Ta sẽ áp dụng định lí này để giải bài toán trên. Gọi x,qy lần lượt
q là khối
lượng riêng của các dung dịch thì d x = 1 · y, px = qy nên x = p , y = pq . Khối
1 q
q q
lượng mỗi bình là α = 1 + pq , β = 1 + pq .
130 HƯỚNG DẪN GIẢI

1 1
Dễ thấy α
+ β
= 1, thoả mãn định lí Beatty. Suy ra hai dãy [mα], [nβ] là phân
hoạch của số nguyên dương nên ta có đpcm
Nhận xét. Ta có một số bài toán tương tự như bài toán này
Ví dụ 6 (APMO 2006). Với mỗi số nguyên dương n, gọi an , bn lần lượt là số cách
viết 10n trong hệ nhị phân, ngũ phân. Chứng minh rằng (an ), (bn ) là phân hoạch
của Z+ \{1}.

Để giải bài này, chú ý rằng: số chữ số của M trong hệ p phân là [logp M ] + 1. Ngoài
ra, α = log2 10, β = log5 10 thoả mãn điều kiện của định lý Beatty. Từ đó, ta cũng
có một nhận xét thú vị rằng: tổng số chữ số của 2n và 5n trong hệ thập phân là
n + 1.
Ví dụ 7 (VN TST 2000). Cho số nguyên dương k . Dãy số (un ) xác định bởi:
u1 = 1 và un+1 là số nguyên dương nhỏ nhất không thuộc tập hợp

{u1 , u2 , . . . , un , u1 + k, u2 + 2k, . . . , un + nk}

Chứng minh rằng tồn tại α vô tỷ dương sao cho un = [nα] với mọi n.

Đây là một kết quả √ có từ 1959.


√ Ta có thể phân tích cách tiếp cận như sau: Xuất
phát từ việc α =  √ 2, β = 2 + 2 thoả mãn điều kiện Beatty. Ta có hai dãy với
công thức an = n 2 , bn = an + 2n là phân hoạch của Z+ . Từ đó, để giấu
dãy bn đi, ta chỉ cần xét an + 2n . Để ý a1 = 1, a2 = 2, a3 = 4, b1 = 3, b2 =
6, b3 = 10 nên a4 có thể định nghĩa là số nguyên dương nhỏ nhất không thuộc
{a1 , a2 , a3 , a1 + 2, a2 + 4, a3 + 6} . Đó chính là cơ sở để có bài toán trên. Việc chứng
minh hai dãy trùng nhau chỉ cần dùng quy nạp, chú ý rằng α1 + α+k 1
= 1 chỉ có
một nghiệm vô tỷ dương duy nhất.
(Dãy Wythoff) Cho chuỗi S1 = 1 . Chuỗi Sn được tạo thành từ chuỗi Sn−1 bằng
cách thay 1 → 01 và 0 → 1. Các chuỗi S1 , S2 , S3 , . . . được ghép liên tiếp lại với
nhau thành một chuỗi vô hạn L . Gọi an là vị trí của số 1 thứ n trong chuỗi L.
Chứng minh rằng tồn tại α vô tỷ dương sao cho

an = [nα] , ∀n.

Ở đây ta có nhận xét rằng số 0 thứ n được sinh ra từ số 1 thứ n nên nếu gọi kn là
số các số 0 đứng trước số 1 thứ n và bn là vị trí của số 0 thứ n, ta sẽ có an = n + kn
1 1
và bn = 2n + kn nên bn = an + n. Do đó + = 1 hay α là tỉ số vàng.
α α+1
2.5 Tổ hợp 131

Bài 6

Có 2020 người đến một buổi tiệc được chia vào các phòng khác nhau sao
cho

i. Không người nào trong một phòng quen biết tất cả các người trong
phòng đó

ii. Trong nhóm 3 người bất kì thuộc cùng một phòng, luôn tồn tại ít nhất
2 người không quen biết nhau.

iii. Với bất kì một nhóm 2 người nào trong một phòng mà không quen
biết lẫn nhau, tồn tại đúng một người trong cùng phòng đó quen biết
cả hai người này.

1. Chứng minh rằng trong mỗi phòng, mỗi người có số người quen bằng
nhau.

2. Tìm số phòng lớn nhất.

Lời giải. Đề bài khiến ta nghĩ đến cách giải bằng đồ thị. Ta xét người ở trong
một phòng bất kì và biểu diễn mỗi người là một đỉnh, hai người quen biết nhau
sẽ tạo thành một cạnh. Do vai trò các phòng là như nhau nên để giải ý 2 ta cần
tìm số n là số người trong một phòng sao cho nó nhỏ nhất. Ta phát biểu lại bài
toán dưới ngôn ngữ của cạnh, đỉnh và các đa giác như sau:
Bài toán. Cho đồ thị gồm n đỉnh thoả mãn các điều kiện sau

1. Không đỉnh nào có thể được nối với các đỉnh còn lại

2. Không tồn tại tam giác trong đồ thị

3. Với hai đỉnh bất kì A, B mà không có cạnh AB, tồn tại duy nhất C sao cho
có cạnh CA, CB

Chứng minh rằng mọi đỉnh đều có cùng số bậc. Tìm số n bé nhất thoả mãn yêu
cầu.

1. Xét một đỉnh A bất kì, giả sử degA = m tức ồn tại đúng m đỉnh B1 , . . . , Bm
được nối với A. Do không có tam giác nên theo điều kiện 2, với mọi i 6= j
thì không tồn tại cạnh Bi Bj . Theo tính duy nhất của điều kiện 3, tất cả
132 HƯỚNG DẪN GIẢI

các đỉnh khác A đều không thể cùng nối tới được Bi và Bj . Do đó, số đỉnh
khác A được nối tới Bi là degBi − 1, giả sử các đỉnh đó là Cij . Lại theo
điều kiện 2 thì những đỉnh nối được tới đỉnh Cij thì không thể nối tới Bi ,
giả sử các đỉnh đó là các đỉnh Chk . Theo điều kiện 2 thì Cij không thể nối
tới Cik và theo điều kiện 3 thì với k 6= i bất kì, Cij không thể nối tới cùng
Ckh và Ckh0 (do Bk 6= Cij nối tới Ckh và Ckh0 ). Do đó, với k 6= i bất kì thì
Cij nối được tới nhiều nhất là một Ckh nào đó. Nhưng theo 3, tồn tại duy
nhất đỉnh X nối được tới Bk và Cij , mà các đỉnh nối tới Bk là A và Ckh .
Mà A không thể nối tới Cij nên với mọi k 6= i, tồn tại duy nhất Ckh nối tới
Cij . Vậy, Cij được nối tới Bi và thêm m − 1 điểm nữa nên degCij = m.
Ta tiếp tục quá trình với đỉnh Bi thay cho đỉnh A rồi lặp lại toàn bộ quá
trình, ta có được các đỉnh có cùng bậc.
Vậy, trong một phòng thì số người quen của mỗi người là bằng nhau

2. Gọi bậc của mỗi đỉnh là m. Khi đó, ta có được m đỉnh Bi và m(m − 1) đỉnh
Cjk nên n = m2 + 1. Với m = 1 thì không thoả 1. Với m = 2 thì n = 5 . Ta
thấy trường hợp sau thoả yêu cầu bài toán

A C

D
E

Vậy n nhỏ nhất là 5 nên số phòng nhiều nhất có thể chia là 2020 : 5 = 404
phòng.
2.5 Tổ hợp 133

Bài 7

Cho tập hợp A = {1, 2, . . . , 2n} với n nguyên dương. Một hoán vị các phần
tử của A được gọi là đẹp nếu như có ít nhất hai phần tử hơn kém nhau
n đơn vị đứng cạnh nhau. Chứng minh rằng số hoán vị đẹp nhiều hơn số
hoán vị không đẹp

Lời giải. Cách giải đơn giản nhất mà ta có thể nghĩ đến là dùng nguyên lí bù
trừ và đếm thẳng số hoán vị đẹp ra và so sánh nó với số hoán vị bất kì. Gọi Ai
là tập tất cả những hoán vị sao cho trong các hoán vị đó, hai phần tử k và k + n
đứng kề nhau. Khi đó, theo nguyên lí bù trừ thì số hoán vị đẹp là
n n n
n
X X X \
|Ai | − |Ai ∩ Aj | + |Ai ∩ Aj ∩ Ak | − . . . − (−1)n Ai


i=1 i6=j=1 i6=j6=k=1 i=1

= 2Cn1 C2n−1
1
(2n − 2)! − 2 2 n
2 Cn C2n−2 (2n
− 4)! + . . .
2n!(2n − 1)! 4n!(2n − 2)!
= .(2n − 2)! − 2
.(2n − 4)!
(n − 1)!(2n − 2)! (2!) (n − 2)!(2n − 4)!
4n(n − 1)(2n − 2)
≥ 2n(2n − 1)! −
4
(2n − 2)! 2n(2n − 2)!
= 2n! − 2n(2n − 1) +
4 16
(2n)!
>
2

Vậy, số hoán vị đẹp nhiều hơn số hoán vị không đẹp.

Nhận xét. Ngoài cách sử dụng nguyên lí bù trừ (mang nặng tính toán) thì ta vẫn
còn một vài hướng tiếp cận khác, ở đây chúng tôi sẽ trình bày hướng thứ 2, đó là
sử dụng ánh xạ. Việc phải chứng minh số hoán vị đẹp nhiều hơn số hoán vị không
đẹp khiến ta nghĩ đến việc liệu có tồn tại một ánh xạ là đơn ánh nhưng không toàn
ánh được xây dựng từ tập các hoán vị không đẹp vào tập hoán vị đẹp hay không?
Xét một hoán vị không đẹp {x1 , x2 , . . . , x2n } thì tồn tại duy nhất k ≤ 2n − 2 sao
cho |xk − x2n | = n. Ta xét phép đặt tương ứng hoán vị không đẹp trên với hoán vị
đẹp {x1 , x2 , . . . , xk , x2n , xk+1 , . . . , x2n−1 } thì phép này là ánh xạ. Ta kiểm tra được
ánh xạ này là đơn ánh và không là toàn ánh (bạn đọc tự kiểm tra), từ đó suy ra
số hoán vị đẹp nhiều hơn số hoán vị không đẹp.
134 HƯỚNG DẪN GIẢI

Bài 8

Cho n là một số nguyên dương và S là tập hợp các điểm (x, y) trên mặt
phẳng với x, y không âm và x + y < n. Các điểm trong S được tô màu đỏ
hoặc xanh theo qui luật, nếu (x, y) là màu đỏ thì (x0 , y 0 ) được tô màu đỏ với
x ≤ x0 và y ≤ y 0 . Đặt A là số cách chọn n điểm xanh mà hoành độ x của nó
khác nhau và đặt B là số cách chọn n điểm xanh mà tung độ y của chúng
khác nhau. Chứng minh rằng A = B

Lời giải. Ta có hình minh hoạ với n = 6 như sau:


A
5.
O C
4.
N P D
3.
M Q R E
2.
L S T U F
1.
K J I H G B

0 1. 2. 3. 4. 5.
A A
5. 5.
O C O C
4. 4.
N P D N P D
3. 3.
M Q R E M Q R E
2. 2.
L S T U F L S T U F
1. 1.
K J I H G B K J I H G B

0 1. 2. 3. 4. 5. 0 1. 2. 3. 4. 5.

Gọi a0 , .., an−1 là số điểm xanh có hoành độ bằng 0, 1, .., n − 1. Gọi b0 , .., bn−1 là
số điểm xanh có tung độ bằng 0, 1, .., n − 1. Khi đó, ta có A = a0 ...an−1 , B =
b0 ...bn−1 . Ta cần chứng minh A = B hay nói cách khác a0 ..an−1 = b0 ..bn−1 .
2.5 Tổ hợp 135

Ta nhận thấy rằng nếu trên đường chéo x + y = n − 1 có một điểm màu đỏ thì
A = B = 0. Do đó ta chỉ nhìn trên trường hợp mà tất cả các điểm trên đường
chéo đều màu xanh. Tuy nhiên, khi trong trường hợp này, ta lại đưa bài toán
về việc đếm bài toán ban đầu với n − 1, đáp án sẽ có được bằng cách cộng
thêm vào mỗi a0i , b0i với 1 (Với a0i là số điểm xanh có hoành độ bằng i trong hình
(x, y), x + y < n − 1 va bi cũng như vậy). Phân tích này dẫn đến việc chúng ta
phải giải bài toán này bằng qui nạp. Ta sẽ tìm cách tính A, B với n từ việc tính
A, B với n − 1. Quan sát thêm một chút, việc tô màu như đề cho sẽ cho chúng
ta các hình tam giác vuông cân, từ đó nảy ra ý tưởng rằng a0 , .., an là các hoán
vị của b0 , .., bn (*). Ta sẽ chứng minh kết quả này bằng qui nạp.

• Với n = 1 thì hình chỉ còn một điểm, khi đó a0 = b0 = 0 hoặc a0 = b0 = 1


nên (*) đúng với n = 1.

• Giả sử (*) đúng với n = k, tức a0 , .., ak−1 là hoán vị của b0 , .., bk−1 . Ta
chứng minh (*) đúng với n = k + 1. Gọi ck là số điểm xanh có hoành độ k
trong trường hợp k + 1, dk là số điểm xanh có tung độ bằng k. Ta xét hai
trường hợp

– Nếu tất cả các điểm trên đường x + y = k đều là màu xanh thì ta có
ci = ai + 1, di = bi + 1, i = 0..k; ck = dk = 1 nên ta có đpcm
– Nếu tồn tại một điểm (m, k − m) màu đỏ thì phần (x, y) : x ≤
m, y ≤ m đều màu đỏ, đặc biệt cm = dn−m . Xét hai phần P =
{(x, y) : x < m, y > k − m} và Q = {(x, y) : x > m, y < k − m} thì ta
có ai , ai+1 , .., am là hoán vị của b0 , b1 , .., bk−m tương tự ta suy ra được
đpcm.
136 HƯỚNG DẪN GIẢI

Bài 9

Cho một dãy n tấm bìa đặt sấp ở trên bàn được đánh số từ 1 tới n. Mỗi lần
cho phép thay đổi trạng thái của k tấm bìa liên tiếp từ sấp thành ngửa và
ngược lại

1. Chứng minh rằng có thể chuyển hết tấm bìa từ sấp sang ngửa khi và
.
chỉ khi m .. k

2. Nếu n không chia hết cho k, tìm số bìa tối đa có thể chuyển sang
ngửa.

Lời giải.

.
1. Chiều thuận là chiều hiển nhiên vì nếu n .. k thì n = kp, ta thực hiện đổi p
lần, mỗi lần k tấm theo thứ tự thì các tấm bìa từ sấp được chuyển thành
ngửa. Ở chiều đảo, giả sử n không chia hết cho k, tức n = pk+r, 0 < r < k,
khi đó, ta thực hiện tô màu các tấm bìa thứ 1, k, k + 1, 2k, . . . , pk, pk + 1
thì số tấm được tô màu là một số lẻ. Tuy nhiên, một lần chọn k tấm liên
tiếp và đổi trạng thái sẽ luôn làm thay đổi trạng thái của 2 tấm, do đó ta
không thể chuyển các tấm trên về ngửa nếu n không chia hết cho k.

Hình 2.1: Trường hợp n = 13, k = 5

Vậy 1 được chứng minh

2. Nếu n không chia hết cho k, tức n = kp + r, 0 < r < k. Ta để ý đến trường
hợp n = 13, k = 5 ở trên và tìm cách để đổi được nhiều tấm ngửa nhất. Ta
thực hiện như sau:

• Đổi các tấm từ 1 đến 5 thành ngửa


• Đổi các tấm từ 6 đến 10 thành ngửa
• Đổi trạng thái các tấm từ 9 đến 13, tức 9, 10 thành sấp, 11, 12, 13
thành ngửa.
2.5 Tổ hợp 137

Cách đổi này giúp ta thu được 11 tấm ngửa. Tuy nhiên, nếu n = 11 thì
ta sẽ không thực hiện bước cuối cùng, vì nếu thực hiện như vậy, ta sẽ chỉ
làm thêm 1 tấm ngửa nhưng lại mất đi 4 tấm, tức là đã lỗ 3 tấm. Trong
trường hợp này, ta chỉ thực hiện hai bước đầu và thu được 10 tấm ngửa.
k
Với những phân tích đó, ta sẽ đưa ra lời giải cho hai trường hợp, r ≤ và
2
k
r>
2
n
(a) Nếu r ≤ , ta chứng minh rằng số tấm bìa được đổi sang ngửa tối
2
đa là kp. Ta thực hiện các bước sau:
i. Tô màu 1 các tấm bìa ở vị trí 1, k, k + 1 . . . , kp, kp + 1 thì ta có số
lẻ các tấm bìa, mà mỗi lần đổi sẽ làm thay đổi trạng thái của 2
tấm bìa nên tồn tại 1 tấm bìa không thể đổi sang ngửa.
ii. Tô màu 2 các tấm bìa ở vị trí 2, k − 1, k + 2, . . . , kp − 1, kp + 2 thì
ta có số lẻ các tấm bìa mà mỗi lần đổi sẽ làm thay đổi trạng thái
của 2 tấm bìa nên tồn tại 1 tấm bìa không thể đổi sang ngửa.
Hơn nữa, các tấm bìa này khác các tấm bìa ở lần trên nên ta suy
ra có 2 tấm bìa không thể đổi sang ngửa

Hình 2.2: Trường hợp n = 16, k = 7

k
Tương tự như vậy đến r < thì ta được có ít nhất r tấm bìa không
2
thể đổi sang ngửa, tức có tối đa kp tấm bìa đổi sang ngửa, và cách
xây dựng để kp tấm sang ngửa thì ta đã biết ở trên. Cách tô màu như
k
trên sẽ không thể áp dụng trong trường hợp r > vì sẽ bị trùng các
2
k
màu với nhau. Điều đó dẫn đến kết quả khác cho trường hợp r >
2
k
(b) Nếu r > , ta thực hiện tô màu như trường hợp trên nhưng chỉ đến
2
bước k − r, ta thu được có ít nhất k − r tấm bìa không thể đổi sang
ngửa, tức có tối đa kp + 2r − k tấm bìa được đổi sang ngửa, cách đổi
thoả mãn có kp + 2r − k tấm bìa thành ngửa như sau: Đổi lần lượt các
tấm từ 1 đến kp thành ngửa theo lượt, mỗi lượt k tấm. Sau đó, đổi k
138 HƯỚNG DẪN GIẢI

tấm cuối cùng, ta thu được số tấm ngửa là n − (k − r) = n + r − k =


kp + 2r − k
k
Vậy, nếu r ≤ thì số tấm được đổi sang ngửa tối đa là kp, trường hợp
2
ngược lại thì số tấm được đổi sang ngửa là tấm kp + 2r − k tấm.

Bài 10

Trong một bảng vuông n × n, ta đặt những chiếc đèn lên các ô của bảng,
mỗi ô một đèn. Ở mỗi lần thay đổi, ta được phép chọn một đèn làm gốc và
thay đổi trạng thái của đèn đó và tất cả các đèn khác cùng hàng cùng cột
với nó từ tắt sang bật và ngược lại. Với trạng thái ban đầu là bất kì, ta có
thể đưa tất cả đèn về trạng thái bật được hay không với

1. n = 6.

2. n = 2017.

Lời giải. Để tiện cho việc mô tả, ta qui ước một ô có trạng thái bật nếu ô đó
có chứa dấu + và trạng thái tắt là dấu -. Ta có nhận xét rằng, việc bất kì trạng
thái nào ta cũng có thể đưa về trạng thái tất cả các đèn đều bất tương đương
với việc ta có thể thay đổi trạng thái của một đèn mà không làm thay đổi trạng
thái của các đèn còn lại. Ta xét tới các trường hợp nhỏ trong bài

1. n = 6. Ta sẽ xem xét rằng có thể đổi trạng thái của một ô bất kì mà không
làm thay đổi trạng thái của các ô còn lại hay không?

Hình 2.3: Trường hợp n = 6


2.5 Tổ hợp 139

Giả sử ô cần đổi trạng thái là ô được mang dấu - ở vị trí như hình trên. Ta
cần chọn những ô sao cho sau những lần lấy các ô đó làm gốc, thực hiện
thay đổi trạng thái, ta được ô dấu - trở thành ô dấu +, các ô còn lại trong
bảng giữ nguyên trạng thái, hay nói cách khác, ô dấu - được tác động lẻ
lần và các ô còn lại được tác động chẵn lần. Ta nhận thấy rằng, những ô
có thể làm thay đổi trạng thái của ô dấu - là những ô được tô màu. Những
ô không được tô màu chỉ có thể làm thay đổi trạng thái với những ô cùng
hàng hoặc cùng cột với ô dấu - . Do đó, trường hợp đơn giản nhất mà ta
nghĩ tới sẽ là thay đổi trạng thái các ô tô màu, sau đó mới tính tiếp tới
chuyện thay đổi trạng thái của các ô không tô màu để cho phù hợp. Ta xét
việc chọn lần lượt các ô tô màu làm gốc, ta thu được

• Ô dấu − được tác động 11 lần (11 ô tô màu)


• Các ô được tô màu mà khác dấu - được tác động 6 lần (bởi các ô cùng
hàng hoặc cùng cột)
• Các ô không được tô màu được tác động 2 lần (một lần bởi ô cùng
hàng, một lần bởi ô cùng cột)

Như vậy, ô dấu - đổi thành + và các ô còn lại giữ nguyên trạng thái, do
đó, đối với bảng 6 × 6 thì ta có thể chuyển hệ bóng đèn ở trạng thái bất kì
về trạng thái bật.

2. Ta để ý rằng, với n bất kì việc tác động như trên sẽ luôn làm ô dấu cộng
được tác động lẻ lần 2n − 1, các ô không tô màu được tác động 2 lần và
các ô tô màu được tác động n lần. Như thế, việc có thể đổi được theo cách
này hay không phụ thuộc vào tính chẵn lẻ của n. Với trường hợp n = 6
này thì câu trả lời là có. Tuy nhiên với, n = 2017 thì chưa chắc. Ta sẽ mong
rằng việc đổi được hay không chỉ phụ thuộc vào tính chẵn lẻ của n nên
ta sẽ xét trường hợp nhỏ hơn để có cái nhìn trực quan cho bài toán. Xét
trường hợp n = 3 và dấu ta cần đổi vị trí như sau:

Hình 2.4: Trường hợp n = 3


140 HƯỚNG DẪN GIẢI

Dấu − ở góc phải dưới và cần đổi dấu ˘ này thành + mà không làm thay
đổi trạng thái của các ô còn lại. Xét bộ ô được tô màu như hình trên, khi
chọn một trong các ô trong bảng làm gốc và thay đổi trạng thái thì có 4
hoặc 2 ô tô màu bị thay đổi trạng thái, tức tổng số các dấu – trên các ô tô
màu không bị thay đổi tính chẵn lẻ. Mà việc chỉ đổi trạng thái của ô (3; 3)
lại làm thay đổi tính chẵn lẻ của tổng số ô - trong các ô tô màu, tức không
thực hiện được. Vậy, ta không thể đưa bảng 3 × 3 bất kì về bảng chỉ có
dấu + được. Từ đây, ta thấy rằng việc chứng minh đối với n = 2017 cũng
tương tự như trường hợp n = 3 vì chứng minh của ta chỉ phụ thuộc vào
tính chẵn lẻ của n. Do đó, với n = 2017 thì không thể kết luận rằng có thể
đổi một trường hợp ban đầu bất kì về trạng thái tất cả các bóng đều bật

Bài 11

Cho một bảng 5 × 5 được tô trắng đen xen kẽ, các ô ở góc được tô đen. Trên
mỗi ô đen có các đồng xu đen và trên các ô trắng có các đồng xu trắng.
Các đồng xu có thể di chuyển đến các ô bên cạnh. A và B cùng chơi một
trò chơi như sau: Đầu tiên, A khởi động trò chơi bằng cách lấy một đồng
xu đen ra khỏi bảng rồi di chuyển một đồng xu trắng vào ô trống. Sau đó,
B di chuyển một đồng xu đen vào ô trống. Các lượt sau đó, A sẽ di chuyển
một đồng xu trắng vào ô trống và B di chuyển đồng xu đen vào ô trống.
Trò chơi kết thúc khi một trong hai người không thể di chuyển được theo
luật trên, và người còn lại là người chiến thắng. Hỏi có chiến thuật thắng
hay không, nếu có hãy chỉ ra ai là người thắng?

Lời giải. Để ý rằng cứ đến lượt chơi của A thì ô trống là ô đen và A sẽ luôn
phải bỏ đồng xu trắng vào ô đen, đến lượt chơi của B thì ô trống là ô trắng và
B sẽ phải bỏ đồng xu đen vào ô trắng. Do đó, trò chơi sẽ kết thúc sau 24 lượt
chơi. Ta có thêm nhận xét rằng sau khi khởi động trò chơi, A đã lấy một đồng
xu đen ra khỏi bảng, và do đó các ô còn lại được phủ bởi các hình domino 2 × 1.
Lượt đầu A di chuyển, tức là đã lấy một đồng xu ra khỏi một miếng domino
nào đó. Nếu B tiếp tục thực hiện như A nhưng đối với phần còn lại của miếng
domino đó thì ta thấy rằng B luôn luôn thực hiện được thao tác của mình sau
khi A thực hiện thao tác. Vậy, B là người chiến thắng
2.5 Tổ hợp 141

Bài 12

Cho lưới tam giác đều như hình vẽ, trong đó mỗi cạnh có chứa n điểm
(không tính hai đầu mút của cạnh), các đoạn thẳng song song với cạnh
tam giác lớn được nối với nhau. Đếm số tam giác đều có đỉnh là các điểm
trong lưới đã cho

B C

Lời giải. Ý tưởng đơn giản nhất được đưa ra ở đây là đếm truy hồi, . Gọi bậc
của tam giác đều này là số điểm có trên mỗi cạnh, không chứa đầu mút. Ta sẽ
tìm công thức truy hồi của dãy Sn với Sn là số tam giác đều trong tam giác lớn
bậc n
A

5 1
4 2
3 3
2 4

1 5
B C

5 4 3 2 1
142 HƯỚNG DẪN GIẢI

Ta thấy rằng, các tam giác trong tam giác bậc n sẽ có thể nằm trong một trong
các trường hợp sau:

• Nằm trong 1 trong 3 tam giác bậc n − 1 lần lượt chứa các đỉnh A, B, C

• Là một trong n tam giác chéo được đánh số như trên hình.

Tuy nhiên, các tam giác nhỏ trong 3 tam giác bậc n − 1 này có thể trùng nhau
nên để không đếm trùng lặp, ta phải sử dụng nguyên lí bù trừ. Gọi X,Y ,Z lần
lượt là tập hợp các tam giác nằm trong tam giác bậc n − 1 chứa đỉnh A, B, C.
Khi đó, ta có:

Sn = |X| + |Y | + |Z| − |X ∩ Y | − |Y ∩ Z| − |Z ∩ X| + |X ∩ Y ∩ Z| + 3
= 3Sn−1 − 3Sn−2 + Sn−3 + 2

Như vậy, ta có được công thức truy hồi của dãy là

(
S0 = 1, S1 = 5, S2 = 15
Sn = 3Sn−1 − 3Sn−2 + Sn−3 + n

Tìm công thức tổng quát từ công thức truy hồi này, ta được

4
Sn = Cn+4

Nhận xét. Chúng ta cũng có thể thử sức với một bài toán khác, cũng tương tự
kiểu đếm này như sau:

Ví dụ 8 (Olympic Sinh viên 2016). Xét lưới 16 × 16 tạo thành từ dãy các điểm
như hình sau (khoảng ách giữa các hàng và các cột là 1).
2.5 Tổ hợp 143

1. Đếm số hình vuông với đỉnh nằm trên lưới và có diện tích bằng 4. Đáp án:
196

2. Đếm số hình vuông với đỉnh nằm trên lưới và có diện tích bằng 25. Đáp án:
283

3. Đếm tất cả hình vuông có đỉnh nằm trên lưới. Đáp án: 5440

Bài 13

Cho 2018 bóng đèn đang ở trạng thái sáng được xếp liên tiếp nhau trên
một đường thẳng. Hai người cùng chơi trò chơi như sau: Ở mỗi lượt chơi
của mình, mỗi người sẽ chọn một bóng đèn sáng, sau đó đổi trạng thái của
bóng đèn đó cùng với 4 bóng đèn phía sau nó.

1. Chứng minh rằng trò chơi sẽ dừng lại sau hữu hạn bước.

2. Ai có thể luôn là người chiến thắng, hãy đưa ra chiến thuật thắng ấy.

Lời giải.

1. Ta chứng minh bằng quy nạp.


Bài toán đã cho đúng với n = 5 thật vậy: Người thứ nhất có hai khả năng
chọn bóng đèn: Một là chọn 4 bóng sau đổi trạng thái thì chỉ có bóng đầu
144 HƯỚNG DẪN GIẢI

sáng người thứ hai phải chọn 4 bóng đầu đổi trạng thái sau đó bóng đầu
tiên tắt, lúc này người thứ nhất phải chọn bốn bóng sau đổi trạng thái và
trò chơi kết thúc. Hai là người thứ nhất đổi trạng thái bốn bóng đầu, khi
đó người hai chọn bốn bóng sau và trò chơi kết thúc.
Giả sử bài toán đúng tới n = k. Khi n = k + 1 ta thấy rằng nếu trong lúc
chơi không ai chọn bóng đầu tiên thì chỉ có k bóng phía sau được chọn
trò chơi sẽ dừng theo giả thuyết quy nạp sau đó người chơi tiếp theo phải
chọn bóng đầu tiên nếu muốn tiếp tục chơi, sau khi chọn bốn bóng đầu
tiên thì bóng đầu tiên tắt, ta không chọn nó được nữa. Trò chơi quy về
trường hợp n=k cũng kết thúc.

2. Ta đánh số các bóng sáng đèn là 1 không sáng đèn là 0. Tổng các số ở vị trí
chi hết cho 4 lúc ban đầu là 504 là chẵn. Mỗi lần chơi chọn bốn bóng liên
tiếp nên tổng này thay đỗi tính chẵn lẽ một lần. Khi trò chơi dừng tổng là
0 vẫn là số chẳn. Nên tổng số lượt chơi là chẳn. Vậy người sau thắng.

Bài 14

Cho 33 điểm khác nhau nằm bên trong một hình vuông có√cạnh là 4. Vẽ 33
đường tròn nhận các điểm này làm tâm, có cùng bán kính 2. Chứng minh
rằng tồn tại một đường tròn trong số chúng chứa ít nhất 3 điểm trong số
33 điểm nói trên.

Lời giải. Bài toán này là một bài toán Dirichlet khá đơn giản, công việc của
chúng ta là chia hình vuông √ này thành 16 hình sao cho đường kính của mỗi
hình này nhỏ hơn hoặc bằng 2. Và tất nhiên, cách chia đơn giản nhất là chia
hình này thành 16 hình vuông đơn vị. Khi đó, theo nguyên lí Dirichler, phái có
một hình vuông đơn √ vị chứa ít nhất 3 điểm A, √
B, C nên khoảng cách lớn nhất
của A đến B và C là 2, do đó đường tròn (A, 2) sẽ chứa B và C.

Nhận xét. Đây là một bài tập về Dirichlet khá đơn giản bởi vì cách chia hình
không đem lại nhiều khó khăn cho người giải, tuy nhiên, đối với những bài toán
sau đây thì việc phân chia không phải là dễ dàng

Ví dụ 9. Chứng minh các bài toán sau (Ngô Minh Phương - Các chuyên đề tổ hợp
bồi dưỡng HSG)
2.5 Tổ hợp 145

1. Bên trong tam giác đều ABC có cạnh bằng 6, cho 13 điểm phân biệt. Chứng
minh rằng tồn tại hai điểm
√ trong số 13 điểm đã cho mà khoảng cách giữa
chúng không vượt quá 3
2. Bên trong hình chữ nhật 3 × 4, cho 6 điểm phân biệt, chứng minh rằng tồn
√ điểm trong số 6 điểm đã cho mà khoảng cách giữa chúng không vượt
tại hai
quá 5
Đối với các bài toán ở ví dụ này, ta nhận xét rằng việc đơn giản hơn là ta chỉ
cần chứng minh tồn tại 2 điểm có khoảng cách nhỏ hơn một số nào đó, ít ra thì
cũng dễ hơn 3 điểm. Tuy nhiên, về số phần cần chia là thực sự khác biệt và không
dễ nhìn như bài toán trên. Ở mỗi bài toán, ta phải chia hình ban đầu thành các
phần sao cho mỗi phần đều có đường kính nhỏ hơn một số cho trước. Và sau đây
là từng cách chia cho mỗi bài toán
Ta cần chia tam giác này thành 12 phần, mỗi phần có đường kính nhỏ hơn
1. √
3:


2. Ta cần chia hình vuông 3 × 4 thành 5 hình có đường kính nhỏ hơn 5. Hình
được chia như sau sẽ thoả mãn đề bài
146 HƯỚNG DẪN GIẢI

Tuy nhiên, việc nghĩ ra cách phân chia cho những hình trên là không đơn giản, đa
số là "mò". Ở đây, chúng ta sẽ đưa ra một cách chia tương đối tổng quát. Mục tiêu
chia hình của chúng ta ở mỗi bài toán là chia thành các đa giác (để dễ trình bày
đáp án) sao cho diện tích mỗi phần là lớn nhất, và đảm bảo đường kính mỗi phần
luôn nhỏ hơn một con√số cố định. Chẳng hạn trong câu a, ta cần chia 12 phần có
đường kính nhỏ hơn 3. Nếu bỏ đi mục tiêu đa giác, ta√có thể sẽ nghĩ đến tìm
các hình có diện tích lớn nhất và có đường kính nhỏ hơn 3 để phủ lên tam giác
đều có cạnh là 6 này. Và hình mà chúng ta sẽ dùng ở đây√ chính là hình tròn. Ta
sẽ phác thảo sơ bằng cách vẽ các hình tròn đường kính 3 và phủ chúng lên tam
giác cạnh bằng 6.

Và chỉ cần "cắt tỉa" gọn gàng, ta đã thu được kết quả như trên. Thậm chí, nếu sắp
xếp gọn gàng, ta chỉ cần sử dụng 10 hình tròn, và do đó, đề bài có thể sửa lại chỉ
cần 11 điểm thì ta vẫn có thể giải được
Cũng một công đoạn tương tự như câu 1, ta có thể phác thảo cho câu b và thực
hiện "cắt tỉa" để thu được đáp án của bài toán
2.5 Tổ hợp 147

Có một vấn đề nhỏ ở đây là phần còn lại không thể phủ
√ bởi một hình tròn, nhưng
may thay vì đường kính của hình này vẫn nhỏ hơn 5 và ta có được đáp án.

Bài 15
Có 2010 que diêm trên bàn. A và B cùng chơi trò chơi theo lượt như sau:
Đến lượt của mình, họ sẽ lấy đi 1, 3, 4, 5 hoặc 7 que diêm. Người lấy que
diêm cuối cùng sẽ chiến thắng. Nếu A chơi trước, hỏi người nào sẽ có chiến
thuật thắng?

Lời giải. Ta có nhận xét rằng không được phép lấy đi 2, 6 que diêm. Nên nếu
A lấy a que thì B có thể lấy 8 − a que để được tổng số que lấy đi là 8. Do đó,
nếu người nào ép người còn lại vào thế chia hết cho 8 thì người đó sẽ thắng.
Do 2010 chia 8 dư 2 nên A với lượt chơi đầu tiên không thể đưa B về trạng thái
chia hết cho 8. Nếu A bốc 4 que diêm, thì B lại rơi vào tình trạng không thể đưa
A vào trạng thái chia hết cho 8, tuy nhiên, nếu B tiếp tục bốc 4 que và liên tiếp
như thế thì cuối cùng chỉ còn lại 2 que diêm tại lượt của A và do đó B là người
chiến thắng. Nếu A bốc số que diêm khác 4 thì B sẽ đưa được A về trạng thái
chia hết cho 8 và do đó B chiến thắng.
148 HƯỚNG DẪN GIẢI
Tài liệu tham khảo

[1] Võ Quốc Bá Cẩn, Phạm Thị Hằng, Chuyên đề Bất đẳng thức hiện đại.
http://www.mediafire.com/file/334rrf4ib3f8msj/
[2] Võ Quốc Bá Cẩn, Trần Quốc Anh, Sử dụng AM-GM để chứng minh bất đẳng
thức.
[3] Nguyễn Văn Huyện, Về một bài toán trong "Hello IMO 2007".
http://www.mediafire.com/file/stharkh32qqpcyb/
[4] Trần Nam Dũng, Về một dạng phương trình hàm đa thức.
https://diendantoanhoc.net/index.php?app=core&module=attach&
section=attach&attach_id=20496.
[5] Nguyễn Chí Trung, Phương trình hàm đa thức một biến.
[6] Lê Phúc Lữ, Nguyễn Tuấn Anh, Một số vấn đề về đa thức bất khả quy.
[7] Lê Phúc Lữ, Các bài toán về nghiệm của đa thức.
[8] Lê Xuân Đại, Công thức nội suy Lagrange.
[9] Nguyễn Đình Toàn, Nguyễn Đức Anh, Tính chất số học của đa thức.
[10] Diễn đàn Art of Problem Solving https://artofproblemsolving.
com/community/ và Diễn đàn Toán học Việt Nam - VMF http://
diendantoanhoc.net/home/.
[11] Lê Phúc Lữ, Sự kết hợp giữa hình học và đại số trong các bài toán về phân
giác, Kỷ yếu Gặp gỡ Toán học 2011.
[12] Nguyễn Văn Linh, Khai thác một bài toán hay từ đề thi Olympic Hình học
Sharygin năm 2017.
[13] Tạp chí Mathley.

You might also like